Sunteți pe pagina 1din 147

Chapter 1: Pervasive Issues

Professional Responsibility concerns are often different than personal ethics judgments. Thus, the purely
moral lawyer will still have some ethical dilemmas.

Consider in light of lawyer in Sprung and in our initial Problemthe lawyer has arguable grounds for winning
the appeal, however, the lawyer also concludes that the adversary's claim will fail on a procedural issue,
because of failure to comply with judicial decisions relating to party status on appeal.
A
If you don't tell the other lawyer, the error may or may not be caught.
A
If you do tell, you (and your client) might still win.

What happened in Sprung,?


A
Make sure students understand that the Procedural rules in force at the time of this case are NOT the
same as the federal rules.
1
Under Missouri practice, when a default was given, because the defendant failed to answer, the
defendant could get back into the case fairly easyas is the case in the current Federal Rules
usually merely indicating that there was a meritorious defense was enough.
1
BUT, when a Default Judgment was obtained, as in the case because proof of damage was put
into evidence before a judge, it was infinitely more difficult for the defendant to reopen the case
in most cases, not only would the defendant have to show a meritorious defense, but also that the
defendant was free from neglect in presenting that defensethe real stumbling block here.
1
Here, then, it is critical to the plaintiffs ultimate success that the time limit wherein the
defendant can get back into the case relatively easily expires.
A
Ask someone to recite the basic facts.
1
When lawyer talks to client about client options, have students act out this part.
1
The point of this case is almost always better made when the students use words they can
attribute to both the lawyer and the client.
a
The students playing the lawyer will almost certainly articulate possible results with
caveatsthe court could let the defendant back into the casethey could have a defense on
the meritsbut nothing is certain.
a
The client, on the other hand, will definitely want to win the case and keep the $1.5
million judgmentwhich students should be able to point out is probably a little excessive
in light of the facts.
A
This is a good case to acclimate oneself to the world of professional responsibility.
A
Ask if there is anything in the case to suggest that the lawyer was a bad lawyerone known for taking
shortcuts, lying to the court or to adversaries?
1
In fact, there is nothing.
1
Indeed, this lawyer had been active in lawyer disciplinary enforcement and was involved in
numerous Supreme Court Committees.
A
Then ask if the students think this lawyer knew the "Rules" of the Profession.
1
In fact, the lawyer seems to struggle with the rules
1
Wanting to follow the clients direction.
1
While not wanting to act contrary to the interests of a colleagues.
a
Here you can determine if the other lawyer was just a colleague.
a
In fact, he was a friend.
A
The lawyer followed civil procedure rules for the client, obtained valid default judgment, then learned a
lawyer-friend actually represented the defendant.
A
Lawyer sought client's advice on whether to permit the defendant in the case, it being anticipated that
defendant might have a legitimate defense.
A
Lawyer took client's directive to not tell defendant or attorney.
A
Did Lawyer do the "right" thing?
1
How do you define "right" in this context?

1
1
1
1
I

As Supreme Court of Missouri finds, lawyer did the "right" thing in terms of compliance with
purely procedural law.
Additionally, lawyer followed client's express direction, thus doing the "right" thing from the
client's perspective.
Several of the judges, however, state or imply that the lawyer has a higher obligation.
Consider in light of readings:

Croft, and Notes following:


A
Largely a historical piece dealing with professional rules in this country.
A
Note development:
1
Series of essays like those of Sharswood and Hoffman.
1
Developed into Canons in 1908 and by 1920 all but 13 states had them.
a
Canons not enforceable rules, but evidence of practice norms.
a
Note illustration from Canon 32--Can the views of some of the Sprung judges arise from
the language this Canon?
1
Canons were interpreted by advisory opinions and developed into a body of enforceable
obligations.
1
Changed into Code of Professional Responsibility with Canons, Disciplinary Rules and Ethical
Considerations in 1969.
1
Further development began in late 1970's and resulted in Rules of Professional Conduct (Model
Rules).
a
Note the last paragraph of this piecethese rules are not ethics but instead a set of
regulatory guidelines
a
If they are regulatory guidelines, what does that tell us?
a
It ought to tell us that the rules should not be violated.
1
They are thus a statement that below this level, we wont go.
1
Use that in conjunction with Preamble Note [1]The lawyer is a representative of clients; an
officer of the legal system; and a public citizen.
1
And Preamble Note [9]virtually all difficult issues of professional responsibility involve conflicts
between or among those roles.
1
In resolving those conflicts, if one of the resolutions takes you below the standards set out in the
rules, what do you do?
1
Select another alternativeWhy?
1
Because the rules are regulatory guidelinesnot statements of ethics.
1
Also note in that same last paragraph of Croft, the Restatement nature of the Rules of
Professional Conduct.
a
And here you can look at any of them.
a
Rules format is to give you Black letter law, followed by Comments.
1
In many states, the Comments are also part of the law.
1
Like a Restatement, then, never read the Rules without reading the Comments to that Rule.
1
Note following Croft: Unstated in this Note is that the Current law in many jurisdictions are the
pre-2002 ABA Model Rules of Professional Conduct; In Missouri, for example, these provisions
are the CURRENT LAWthese Rules are the second set of Rules listed in most statutory
supplements.
1
Beginning at the end of the last century, however, two things happened.
a
First, in 1997, the ABA appointed a new committee to look at the Rules to determine if
changes are necessary.
a
That committee was known as the Ethics 2000 Committee and their recommendations are
now before most statestheir report is in your supplement as the current or 2003 Model
Rulesthey are first set of Rules listed in most supplements.

These rules are the CURRENT ABA Ruleshaving been approved in both 2002 and as a
result of amendments in 2003. They have been forwarded on to the states for adoption, but
few states have yet to complete this process.
1
Here is the problem; beginning with 2004, the Multi-State Professional Responsibility
Examination tests the 2003 Rules exclusively. Thus, while you will need the 2001 Rules for your
current practice in many states, the actual test will be exclusively on the 2003 Rules.
1
There is a further complication in MissouriAs of January, 2005, Missouri follows the 2002 (the
former) Model Rules, with some variations.
a
BUT, a Missouri bar committee is studying the CURRENT Model Rules and will suggest
amendments to the Missouri rules.
a
The changes that have been so far suggested DO NOT track anything that has been done
with the CURRENT rules.
a
You can find the Missouri Bar proposed revisions on our class websiteI recommend that
you consult it often.
1
Our study here, then, as is the casebook, concentrates on the 2003 Rulesyou will simply be in the
forefront of your firms.
a
The Second thing that happened at the end of the last century is in Note 2 following
Croft;, the American Law Institute appointed a group to come up with a Restatement of
the Law Governing Lawyers

That group has finished their work and the ALI Restatement is contained in some
supplements.

While it is highly instructive, it is NOT the law in any state and is not likely to be
soon.

But, the provisions of the Restatement will have an affect on state courts and you
really need to look at its provisionsparticularly where they differ with the current rules.
Not mentioned in Croft, but a further defining element for us are Ethics Opinions, both formal and informal
adopted by ethics committees of both ABA and statesthey are not the lawbut are highly influentialthey are
mentioned in Note 3 following Croftyou can see they are adopted by the ABA, state bar associations, even
some local and specialized bar associationssuch as the patent bar.
Let me make my point again; for our purposes, then, and for purposes of this course, the Rules currently
governing lawyers according to the American Bar Association, and therefore this class are called The Rules of
Professional Conduct or The 2003 Model Rules--The Rules of Professional Conduct are written in a
Restatement form, and contain BLACK-LETTER MANDATORY LAW and Comments.
A
The provisions of the ALI Restatement are only influential at the moment.
A
Our focus will thus be on the 2003 Rules, but we will always look at where there are differences in the
following:
1
The 2001 Rules
1
The ALI Restatement.
A
On the test(s) in this course, we will cover the following:
1
The 2003 or CURRENT ABA Rules; AND
1
Any changes suggested for Missouri, as indicated on the course website.
When current Rules are viewed as a regulatory mandate," what are lawyers obligation in either Sprung or
our problem?
A
Review of the Notes following Croft, beginning with Note 4 help you work your way through this.
1
First, look at Rule 1.6is information learned in the course of representing the employer likely to
be considered confidential within the meaning of Model Rule 1.6?
1
To reveal that information to the other side then would require that one of the listed exceptions
exist--and none do.
1
Then, look at Rule 1.2(a) requiring substantive decisions within the relationship to be made by
the client, but only requiring the lawyer to consult with the client regarding the means of
accomplishing these objectives; Is the decision to tell the lawyer on the other side means,
particularly if it means the client will likely lose?

Former Canon 32 indicates that the lawyer "advances the honor of the profession and the best
interests of" the client when the lawyer provides advice that suggests "exact compliance with the
strictest principles of moral law;" BUT that only requires providing Adviceit does not require
the lawyer to disavow the clients wishesWell talk more about morality in just a moment.
Questions 7 and 8 at the end of the Croft article lead into the Wasserstrom article; Where does your
conscience fit into what you do on behalf of your client and, if your conscience dictates that you do
something, can or should you use your forensic ability to convince the client that your objectives should
be those of the client? Suppose you simply cannot stand what the client wants to do?

Wasserstrom article and Notes following,


A
Wasserstrom offers two criticisms of practice of law.
1
What are they?
a
Role-Differentiated position of attorney.
FIRST CRITICISM: The Amoral Lawyer

A lawyer's role differentiated professionalism renders moral considerations irrelevant.


Things that an ordinary person need not and should not do. Lawyer's goal is to provide
competence that the client lacks, regardless of the client's goals. Laywer has a duty to
make his or her expertise fully available to a client whom they've sought to represent.

Most attorney's find this amorality simple - few exceptions, such as representing
unpopular clients and pushing "unfair" laws

Attorney is Non-Moral--so long as client's position is legal, lawyer's job does not include
passing on client morality.

Thus, unique nature of lawyer's role requires lawyers to sanction conduct they would not
otherwise sanction.

Wasserstrom appears to blame part of his criticism on the legal profession, primarily
because rules operate within the Adversary System.

Does Wasserstrom think this system has a place in any area of the practice of law?
Wasserstrom sees this system working well in criminal defense, where the
Constitutional guarantee of counsel may legitimately require the lawyer to be roledifferential.

In other cases, however, the Adversary System only perpetuates role differentiated
behavior and, because it takes place in a forum uniquely populated by lawyers, this role
differentiation is virtually impossible to control; the result is that the client does not have
the ability to understand what is going on in a matter in which a lawyer is used.

SECOND CRITICISM: Lawyer-Client Relationship is Inherently Unequal/Defective


This leads to Wasserstrom's second criticism of the profession--that the lawyer-client
relationship is flawed from the outset because the lawyer, as a professional, is trained in
areas unfamiliar to the client, that the lawyer is a member of a small, specialized group
that enjoys considerable social prestige, and that the lawyer therefore possesses power and
influence that the client cannot match.

This makes the client inherently unequal.


It allows the lawyer to manipulate the client.

Professional relationships are inherently unequal


Professionals have expert knowledge and jargon
Clients cannot evaluate professional performance (but judges and other
professionals can, thus malpractice)

"The professional often interacts with the client in both a manipulative and
paternalistic fashion."

First, do any of Wasserstroms criticisms find their way into the current Rules?
A
Note 1 following Wasserstromlook at Rule 2.1is this an attempt to add a morality element to an
otherwise amoral activity? Would this appease Wasserstrom?
A
And consider again Rule 1.2(a)s provision that the client controls the objectives of the representation,
while the lawyer controls only the meansis this the influence of Wasserstrom? Attempting to prevent
the lawyer from using the lawyers forensic ability against the client?
A
Rule 2.1: Advisor: requires attorneys to use judgment to give honest advice and permits attorneys to
refer to matters outside the law, including moral concerns, in advising clients.

Consider Wasserstrom in conjunction with the excerpt from Simon, Note 2, following Wasserstrom.
A
Simon says that most critics see but two models for the profession.
1
The Libertarian model.
a
Favors procedure over substance favors form over purpose.
a
Allows lawyer to take advantage of loopholes in the law that benefit client.
a
Does not favor or permit law-breaking; rather allows unfettered use of laws arguably legal,
but sometimes contrary to legislative intent.
1
The Regulatory model.
a
Favors substance over procedure.
a
Basic duty of lawyer is to assure decisions on merits of disputes.
a
Thus discourages use of "technicalities" to thwart decision on merits.
a
Examples: duties to share information and correct misunderstandings
A
Both approaches, according to Simon, do not permit discretion.
A
Simon advocates a third, "discretionary approach;"
1
Lawyer can consider all relevant circumstances of a particular case and then take those actions
most likely to promote justice.
a
This allows lawyer to consider the relative merit of any claim.
a
Both of the particular claim itself and in terms of how allocating time on that claim will
prevent the lawyer from allocating time to other clients.
1
Simon's discretionary model would attempt to redefine the "right" approach by not automatically
making the legally correct outcome the "legal" one and the other the "morally correct" one.
1
Consider in light of his example under Internal Merit, that of plaintiffs lawyer who is unaware
that contributory negligence has been statutorily abolished and insurance defense counsel who
is. In Simons discretionary approach, insurance defense counsel should be concerned with
global fairnesswould a settlement be fair if the plaintiffs lawyer recommended less to the client
but still thought the case could be lost on contributory fault?
A
To support this view, what does Simon say about the Rules of Professional Conduct?
1
He advocates that they become rebuttable presumptions that the lawyer could disregard if the
value to be served by the rule was not served.
1
It is necessary for Simon to approach the rules this way because Rule 1.6 would prevent the
insurance defense lawyer from telling the plaintiffs lawyer without the defendant insurance
companys consentsomething the insurance company almost certainly would not permit.
A
In his discretionary approach, isnt Simon really acting more like a judgeconcerned with the ultimate
outcome of the caserather than an advocate for one of the partiesHold this thought....

Consider Wasserstrom and Simon in light of the lawyer in Sprung.


A
Wasserstrom and Simon address different issues for that lawyer.
A
Wasserstrom is concerned with the model of the relationship between attorney and client--that is the
relative position of attorney to client.

Simon is concerned with the attorney's performance within that relationship--nothing in Simon
addresses the issues of lawyer dominance within the relationship--in fact Simon seems to presume that
dominance.
In Sprung, some of the dissenting judges' criticism of lawyer are stated in terms of violating
professional "practice norms," these judges then seem to suggest a greater obligationbut from where?
Is the question the dissenting judges in Sprung are asking: "Can, or perhaps should, these regulatory
measures take into consideration the moral precepts of the earlier Canons and Code?" Isnt this like
Simons discretionary approach?
Look also at Note 3 following Wasserstrom; that Note suggests various models available to a lawyer
whose personal conscience is at odds with what the lawyer perceives as the lawyers professional
obligation.
1
In fact, the Note suggests that lawyer has multiple options when the lawyer does not want to
engage in role-differential behavior.
a
Recognize lawyering as non-moral and go onis this much of an option?
a
Integrate moral imperatives of profession within own conception of self and deal with
moral disputes as with other life experienced moral problems; thereby accepting
responsibility for the resolution, including being ready to leave profession if necessary.
1
Wasserstrom would seem to prefer the option of integrating the "demands of moral point of
view" within a concept of self and dealing with those moral imperatives.
1
Is that the same as a "discretionary approach" as seen by Simon?
a
It might consider the same factors; but
a
Simon does not seem to be concerned with the lawyer imposing the discretionary
approach on the client, regardless of what the client wants--the second of Wasserstrom's
criticisms that professional relationships are inherently unequal.
And if you again consider Rules 2.1 and 1.2(b), you will see that both Wasserstrom and Simon are
finding their way into the Rules.
1
Again, Rule 2.1 permits lawyer to tell client moral, social, economic, political consequences of
the client's actions. The rule is discretionary because compulsion could result in lawyer
manipulation of the free will of the client--creating the "oligarchy of lawyers" to which
Wasserstrom refers; but it does appear to allow the lawyer to advise the client of the kinds of
relative merit issues mentioned by Simon.
1
Rule 1.2(b) allows lawyer to represent a client without endorsing the client's views on issues.
This would seem to differ with Simon, who would arguably want the lawyer to include
consideration of relative and internal merit; but it does appear to satisfy Wasserstrom's notion of
client control.

The Sprung Lawyer is not said to have failed to comply with the ideals expressed in Note 3 following
Wasserstrom.
A
In fact, once the lawyer told the client of the moral, social aspects of case (Rule 1.2), the lawyer followed
client's direction of concerning the "objectives" of the representation (Rule 1.2(a)).
A
Thus, what the lawyer did in Sprung appears to be in accord with both of Wasserstrom's criticisms.
A
It is not known if lawyer considered the relative merit of the case, although the lawyer's testimony in the
opinion makes clear that the lawyer did have some thoughts on the issue.

If lawyer complied with Wasserstrom, possibly considered the discretion of Simon and did not violate any
rules, how can the Sprung dissent criticize the actions?

Part of answer involves discussion of Adversary System. [Is it a necessary evil? Or just an evil?]
A
Aronson helps us by giving us several definitions of the adversary system; he sees the lawyers function
as doing the lawyers best to advance that system, regardless of the lawyers personal morality;
1
Traditional definitions of that system:
a
First, is it a "truth-oriented" system?

Under it, partisan advocacy is valuable only so long as it leads to a correct


adjudication of the facts.

The lawyer subordinates the clients interest to a higher personal moral value.

This sounds very similar to Simons internal merit;


a
Second, is it an "adversary-oriented" system?

Probably the system we know best; the parties investigate, introduce and argue
the evidence most favorable to their side.

Prevents the lawyers from advocating a personal opinion of the facts, instead
presuming the clients set of facts and arguing those.

This should sound similar to Simon's libertarian approach.

As Note 1 following the article reminds you, this is generally seen as the bedrock
of our legal value systems.
1
Aronson's "innocence-oriented" system is similar but functions only in criminal cases; Like
Wasserstrom's exception, Aronson recognizes that criminal defense does require the lawyer to
engage in adversary-challenge to state evidence.
1
Aronson sees the "Adversary System" as a process for resolving disputes.

As a process, it presumes that each side has equal adversaries and that each side has an
equal opportunity to present its arguments.

If that presentation is roughly equal, and it can be weighted by case law, statute, or rule to
help one side or another, the system has worked.
Consider in light of Shaffer, in Note 2 following Aronson; Shaffer sees the Adversary System as
producing the kind of criticism Wasserstrom noted; a "market-morality;"
1
Shaffer completes a full circle back to the Croft article earlier in the Chapter.

Croft told us about rule-making by the profession.

In the words of one radio commentator, Shaffer gives us the rest of the story behind
enactment of these rules.

He points out how lawyers like David Dudley Field used the very Field Code of civil
procedure as a method for thwarting the law; and here again, Simons libertarian model
comes to mind.

He also points out how lawyers moved more to a law firm rather than individual practice
norm and also moved into more formal bar associationsboth of which produced a need
for more regulationboth of which produced a bar that was not mindful of a
consideration of the common good;

He argues that the adversary ethic is a recently invented rationalization.

He would ask lawyers, "Does your practice make people better-- not just your client, but
other people in the community?"

If the opposing advocates are of unequal talent and experience, will the adversary system
lead to the correct result?

Shaffers point then is that rule-making by the profession has caused this market-morality to
become an ethic of its own.
a
In that regard, Shaffer would support Simon's apparent opposition to the Regulatory
approach.
a
Shaffer says that making the client a better citizen is not irrelevant; thus supporting
Simon's discretionary approach.
Whether you use Simon, Aronson, Shaffer or some other model: How you define the system in which
lawyers represent clients will determine which of the professional "models" you use.
1
But dont take too much comfort in that.
1
David Dudley Field and his colleaguesthe lawyers for the robber-baronshave won the day
when it comes to professional regulation.

1
1

Which has been coupled with a professional view of the adversary system that equates it with
some Constitutional concepts.
As a result, what we have is a regulatory scheme that does not view the rules as rebuttable,
and which you must therefore follow.

With that in mind, however, lets return to our problem, and think about what we have read in light of
question, Note 4 following Aronson.
A
If you call attention to the error, who is served?
A
If your adversary is not smart enough to figure something out, how is that your responsibility?
A
As the Note points out, if that is true, doesnt the responsibility then fall to the court to sort out when
counsels error should be excused to the detriment of the side that did not err?
1
And as you rethink Simons discretionary model, or Aronsons innocence model; or Shaffers
point, dont you have to ask...
1
Does this model help the court decide the case more than it helps my client advocate their case?
And if the answer is yes, what is my real job? And that is the point of Note 5 following Aronson.

In addition, Terrell & Wildman partially help.


A
Note distinction between "Tradition" and "Traditionalism,"
1
Tradition is a faith in a system.
1
Traditionalism is "dead faith of the living," a longing for the way things use to be.
1
Which of these are the dissenting judges in Sprung actually exhibiting?
1
If it is Traditionalism, then they are upset that lawyer did not do things the way they used to be
done.
1
If it is Tradition, then they are upset that lawyer did not keep up with the spirit of certain
essential, timeless principles upon which all lawyers can, and should be judged.
A
these authors suggest the following "traditions;" 6 Values
1
Excellence.
1
Ethic of integrity.
1
Respect for the system.
1
Respect for other lawyers.
1
Accountability.
1
Adequate distribution of legal services.
A
In Note 1 following this excerpt, are Judge Blackmars thoughts those of tradition or traditionalism?
Is he longing for his perception of the practice of a bygone era? Or is he arguing that a lawyer under
Rules of Professional Conduct must sacrifice the interests of a client in favor of assisting a fellow lawyer
who erred?
A
Before finally concluding any thoughts on the lawyer in our problem, or the lawyer in Sprung, consider
them again in light of Morris Dees, who is virtually universally respected as a good lawyerand you
can see what he did to help clients in the passage in the Note 2 following Terrell and Wildman.
1
Note 3 following Terrell and Wildman asks students to assume lawyers for the Klan have
committed a procedural error akin to that committed by the lawyer in Sprung. It further asks
students to assume that Dees meets with his clients and tells them about the error and the
consequences and wants to know if the clients want him to tell the Klan lawyer so they might get
back in the case.
1
One can only assume that even the judges in Sprung would be hesitant to criticize Morris Dees
for doing exactly what the lawyer did in Sprung.
a
If those judges would agree with Dees actions, ask students why?
a
Certainly, the Klan is not as likeable a client as the parties in Sprung, even though the
erring party there was a corporation.
a
But, if the students think the judges would agree with Dees conduct, while still
disagreeing with the conduct of the lawyer in Sprung, dont we have our answer to the
tradition/traditionalism question? Arent the judges hoping the lawyer in Sprung would

not treat the judges the same way as he treated his adversary if the judges were
representing a client, had a bona fide case and made an error? And isnt that
traditionalism?
At the end of this chapter, students almost always say: I cant outline this material and I am unsure what is
going on;
A
It is probably therefore worth raising that this material is not designed to be assimilated like normal
cases and readings from other cases.
A
The material in Chapter 1 is called pervasive because it will be raised in specific contexts as students
go through the material.
A
I usually tell students to make notes about the principal readings and just hang-onthis material will
have relevance as we move forward.
Chapter 2: The Legal Profession and the Practice of Law

Section A. Judicial Regulation and Doctrine of Inherent Power.


A
The Problems Question asks who will win a suit in which a public entity tries to discharge a lawyer,
without cause, in spite of a law that provides for termination only upon good cause.
1
Generally, hiring and firing of public employees is a matter for the legislature
1
As a result, usually controlled by statute.
1
So start by asking who students thought would win when they read only the problem and had
not yet read Wallace?
A
Succession of Wallace appears to reject this general rule when dealing with lawyers.
1
State statute permitted a person to designate, in their Will, an attorney for the estate and then
prevented the Estate from discharging that lawyer without good cause.
1
This was in some conflict with rules governing lawyers, Rule 1.16 which requires a lawyer to
withdraw when discharged.
1
The state statute was a legislative response to prior decisions refusing to allow a Will to speak on
the issue without legislative permission.
A
Court sees its power as coming from three sources:
1
Louisiana State Constitution's provisions for separation of powers.
1
The inherent power of any court; and
1
The power the court has been given to discipline members of the bar.
A

The power to reject this statute as applied to attorneys is said to be "inherent;" that is, it is necessary to
the court's ability to fulfill its role as a separate branch of government and its role as superintendent of
the state's judicial system.
1
The power the court finds is the "exclusive" power to control the practice of law; nothing the
legislature does can touch on such an area.
1
Thus, in areas where Court has also issued a rule, any legislative action that would appear to
override the rule would violate separation of powers principles because the legislature would
invade areas within the court's province.
a
This is so even though the legislature has the power to enact such laws, under its police
power jurisdiction.
a
Thus, if Court had NOT adopted a rule, the issue would be whether the legislature could
legislate in the area, and bring lawyers within that law, at least until the Court declared
the law inapplicable to lawyers.
Extent of the "inherent" power to control attorneys, questions, Note 2 following Wallace.
1
Inherent because it is not express, but arises from the overall Constitutional authority of the
Court.
1
Applies not only to lawyers who appear in court, but to all lawyers--primarily because lawyers
are considered generalists and any of them could be called upon to represent a client in court.

1
1
1

Inherent: often unstated in a state's constitution, but necessary to a supreme court's ability to
fulfill its role as a separate branch of government.
Exclusive: when court has acted, legislature cannot.
Universal: applies not just to lawyers who appear in court, but to all attorneys (because all
attorneys could appear in court.)

Ask students to test themselves by what we mean exclusive control over the practice of law, using the
questions in Note 2 following Wallace.
1
Does the legislature have any power to enact laws touching on the practice of law?
a
Of course they do.
a
The legislature has police power to enact any legislation that touches on public health,
safety or welfare.
1
The court is not limiting police powerthat is not the point.
1
The court is saying that, under the constitution, it is the court that has the power to control the
practice of lawbecause lawyers are the courts officersas a result, if legislative and judicially
enacted provisions are in conflict, then those of the court will controlit is a matter of separation
of powers.
1
If the court approves a legislative provision, it can adopt it as its own.
1
But, it is not acquiescing in the legislative powerrather it is enacting the provision for itself
using the legislative provision as its text or context.

Can the court overall the legislature??? Can legislature supersede state Supreme Court's [Model Rule]
1.16(a)(3), requiring attorneys to withdraw when discharged by clients.
HOLDING: "This court has exclusive and plenary power to define and regulate all facets of the practice
of law."
Obviously, if the court can enact provisions regardless of what the legislature does, then exclusive
control over all lawyering activities can produce public concern about the "club" nature of the
profession and can further the notion of an "oligarchy of lawyers," as mentioned by Wasserstrom.
1
This is the concern of Wolfram, Note 3 following Wallace: Wolfram sees "inherent power" in two
ways:
a
Affirmative "inherent power" is the power the court has to affirmatively act in an area that
touches upon the practice of law. Generally, the exercise of "affirmative" inherent power is
not of concern to Wolfram.
a
Negative "inherent power" is the claimed right of a court to say that no other branch of
government can "legislate" in areas relating to the practice of law.
1
Both the problem and Wallace are "negative" inherent power cases.
a
The court does not try to reconcile the legislative provisions with its own rules; the court
simply strikes the legislation, as it might apply to lawyers.
a
This seems to negate the power of the legislature to enact "police power" legislation.
1
This can be a source of controversy, particularly when the Court, all of whose members are
lawyers, determines that laws designed to apply to all citizens do not apply to lawyers.
1
And while our problem and Wallace are both inherent power cases involving legislation, the
doctrine will also prevent the executive branch from interfering with the judicial branchas noted
in Note 6 following Wallaceeven though it would be good investigative technique for the
prosecutorwho is an executive officerto interview the party represented by counsel, this lawyer
is still a lawyer, subject to the judicial branch and Rule 4.2.
What about the fact that the problem involves a public employee?
1
Note 5 following Wallace mentions the N.J. case upon which our problem is based.
1
Students may ask about Comment [5] to Rule 1.16 which indicates that whether appointed
counsel can be discharged is a matter of applicable law.
1
At that point, some discussion of the notion appointed needs to take place to draw a
distinction between a truly appointed attorneyone appointed by a judge to represent a client

A
A

A
A

in a given case and for whom the applicable law statement of Comment [5] to Rule 1.16 applies
and one who is appointed by a government agency, to whom Comment [5] does not apply;.
1
The N.J. case is present for completion and I do not cover it because I dont want to create an
unjustified expectation that such an exception exists in all states.
The extent of inherent power regulation can be seen in Note 7 following Wallace; all of these provisions
would be permissible judicial regulation under either affirmative or negative inherent power.
This inherent power is usually found in the section of most state court rules that deal with the Supreme
Courts general power or power over the bar. They are not found with other civil or criminal procedure
rules.
If you take Wolframs concern to its logical extreme, there would be no limit to the extent to which a
state supreme court could "regulate" the practice of law and could prevent other governmental entities
from "regulating" in that area.
Students should be sayingwell thats true, but the federal courts could always step in because of federal
supremacy.

Section B. Federal Intervention in State Regulation


A
Problem
1
Question (a) Court adopts rule restricting admission to those scoring highest scores on bar
examination.
a
Rule is challenged on basis that it is anticompetitive, in violation of the antitrust laws.
a
The court moves for summary judgment
-> suit fails because Supreme Court is immune from antitrust liability under the state-action
doctrine enunciated in Parker v. Brown. Here the Illinois SC was acting in a legislative capacity
and therefore in the same position as a state leg, so that the activities in question are exempt from
Sherman Act liability. (BOTTOM OF PG. 52)
1

Question (b) students are asked to assume the rule was adopted by the Board of Bar Examiners
acting on their own, although all of the members are appointed by the Supreme Court and
although the court's rules require new board rules to be filed a year in advance, which was not
done and which is rarely done.
-> with regard to court committees, the SC "has required a showing that the conduct is pursuant to
a 'clearly articulated and affirmatively expressed state policy' to replace competition with
regulation." (INSENT, 6., PG 58)

Question (c) expands question a and asks students to assume there is evidence that members of
the court knew the new rule was discriminatory and took the action anyhow.
-> Equal protection suit against racist board of bar examiners and state supreme court the court
is a state actor and is thus subject to suit on Equal Protection grounds under leg. Immunity, the
judges cannot be sued for damages under civil rights law. Under sovereign immunity, the court
and state may be immune from a damages suit. Injunctive or other equitable relief may be obtained
if Eq. Pr. Violation can be established.

Question (d) raises a related matter; an attorney who spoke out against the court's adoption of
the rule is charged with unethical conduct before a bar ethics committee and files a federal suit
alleging the speech is protected by the First Amendment.
-> suit seeking injunction preventing enforcement of state disciplinary rules, based upon first amd
rights under abstention doctrine, federal courts won't enjoin an ongoing state disciplinary
proceeding if it 1) is a judicial proceeding, 2) involves important state interests and 3) permits
constitutional challenges. (PGS. 59-60)

Question (e) asks students to assume that the disciplinary case went to a hearing and that Hinds
was able to present the First Amendment defense, but that defense was unsuccessful. Can she
now file suit in the federal court on the basis of some federal denial of civil rights?
-> A federal district courtlacks subject matter jurisdiction to entertain appeals from state courts.

Question (f) is entirely hypothetical. There is no such provision of the Civil Rights Act and
students need to be so informed. It is here only for teaching purposes. It asks students to assume
a specific federal law requiring states to have a bar that reflects the diversity of the state. Will that
law prevail over the state Supreme Courts assertion of exclusive power over the practice of law?
-> The McDade Amendment makes state professional responsibility rules applicable to
federal prosecutors (PG 61). The Sarbanes-Oxley Act requires reporting by attorneys of
federal securities law violations, despite state PR rules prohibiting disclosure of client
information. (PG 61)
Will state or federal law control?

Lawline v. ABA,
1
The American Bar Association is a voluntary organization that adopted a rule relating to the
"unauthorized practice of law" and providing definition to that rule; the rule was then sent on to
states and the ABA urged State Supreme Courts to adopt the provision.
1
This rule was adopted by the Supreme Court of Illinois and by the United States District Court
for the Northern District of Illinois.
1
Plaintiffs, who run a business that tries to provide people with alternatives to lawyers, sues
alleging the rule is anticompetitive and violates the antitrust laws.
1
The court rules as to each of the defendants:
a
As to the two Courts and their members:

When a governmental entity adopts legislation that is anticompetitive, that legislation


does not violate the antitrust laws, because that was not the kind of activity these laws
were designed to protect--as to the Supreme Court of Ill., this is the Parker v. Brown stateaction exemption.

The antitrust laws only protect commercial activity, not governmental activity, even where
the governmental activity is decidedly anticompetitive in nature.
a
As to the Bar Associations:

They didn't engage in any anticompetitive activity, other than adopting and
recommending passage by the government defendants.

Mere advocacy of an anticompetitive rule is not, of itself, anticompetitive.


1
It is also alleged that the rules violate certain Constitutional rights and are adopted by "state
actors" sufficient to sustain a civil rights action.
1
This sounds like a possible cause of action particularly since the court has already been
determined to be a state actor for Parker v. Brown purposes.

However, judges are judicially immune from damage actions for activities conducted
within the scope and course of their position as judges, as discussed in Note 6 following
Lawline.

Rule-making is clearly such an activity.

But, suppose they acted in defiance of the law--can an argument be made for saying the
actions of the judges were "ultra vires," therefore outside the scope of their duties and not
covered by the immunity?

Even when judges are immune from damage liability--could, however, be liable for
injunctive relief and attorney fees.

As to Bar Associations.

They are not state actors merely because they recommended the rule.

As to Bar Committee.

The court indicates they are agents of the Supreme Court and thus enjoy judicial
immunity.

Whether the bar committee is an agent will depend on how much control the court
exercises; note reference to Hoover v. Ronwin, Note 6 following Lawline

Ronwin took the Arizona Bar Examination and failed.

The Arizona bar examination is administered by the state board of bar


examiners.

Bar examiners appointed by the state Supreme Court.

The bar examiners have the power to adopt rules, including a grading formula,
so long as rules are filed with the Supreme Court.

Ultimately, the Supreme Court must pass on applicant, through a review


procedure for any who fail the exam.

Action alleges a conspiracy to control the number of admits to the Arizona bar in
violation of the antitrust laws.

Having determined that lawyers are protected by Constitution for such things as
speech (Bates v. State Bar of Arizonacommercial speech), this case asks if lawyers
are protected by federal antitrust legislation.

Supervision issue:

The majority held the bar committee enjoyed judicial immunity if the Supreme
Court had to receive the rules before they went into effect and if the Court kept
ultimate control over admission to the practice.

Minority would have preferred antitrust exemption only if the agency action is
required by the Court

How does this relate to question (b)?

Is there enough supervision in our problem?

What is clear from Ronwin is that the monopoly created by State


Supreme Court control over the practice of law is federally permissible, even
over claims that the activity impedes competition, so long as the Supreme
Court of the state is exercising direct supervision over the agency adopting
the anti-competitive rules.
What about Question (d)?

That is Note 9 following Lawline.

If Hinds believes she has a legitimate First Amendment right to make the kangaroo
court comment, your immediate reaction should be that this is a federal question and
that we can get better relief going to federal court.

What we should do then is prevent the local ethics committee from actingthrough
injunction in the federal courtand have the First Amendment portion heard in federal
court.

As Note 9 indicates, this raises the Younger abstention doctrinemade applicable to ethics
proceedings by Middlesex County in the Note (the case upon which the problem is
based)the doctrine applies when there is a pending state proceeding.

Thus, in Question (d), there is a pending state case--federal court should dismiss in the
absence of some showing that the state proceeding cannot get it right.

And here noteby getting it right we mean that the ethics committee must ONLY have the
ability to hear the First Amendment claimit is not necessary that the committee decide
the claim in favor of the party raising it;.

The likelihood is that under Younger/Middlesex County, the state would need to
demonstrate through several cases that it will not properly consider the First Amendment
claim before there would be the kind of exception viewed by Younger.
Question (e)

This is Note 10 following Lawlineonce Hinds loses in the state ethics proceeding, the
temptation is to say o.k., Ive complied with the Younger doctrinenow Im going to
federal court;

That, however, cannot be done under the Rooker/Feldman Doctrine--lower federal courts
cannot generally review decisions by highest court of state.

Thus, the only way to get federal review of the First Amendment claim is to proceed
through the appellate process in state court and then apply for certiorari to U.S. Supreme.
Question (f)

There could be some federal action.

Under Article VI of the federal Constitution, the laws of the United States are the supreme
law of the land.

Thus, if Congress adopts legislation, and that legislation affects lawyers, it can be
considered supreme over state law.

You can see this very problem in Note 12 following Lawline; in a decision of first
impression, a federal court considered federal prosecutors not subject to certain state
ethics rules.

But also see what Congress didit returned control of federal lawyers to the states, making
federal prosecutors subject to the ethics rules of the states.

Look then at Note 13 following Lawline; Following the Enron and other corporate
scandals, Congress enacted the Sarbanes Oxley Act and put into it provisions requiring
the SEC to adopt rules for lawyer conduct; If these provisions conflicted with a state ethics
rule, which would prevail?

You should say the federal rule.

The ABA adopted rules attempting to respond to the Sarbanes Oxley Act in the summer of
2003 and we will get to them under confidentiality.
Lets review.

Look at Note 11 following Lawline.

At end of this section, while it is clear that Federal Courts can assert supremacy over state
court regulation of the practice of law, we have seen that the actual instances of the federal
court intervening are few and far between and can only exist:

Where there is no pending state proceedingbecause a pending state proceeding would


prevent federal court review under Younger.

To obtain prospective relief in the overwhelming number of cases, although this can
include attorney fees, because damage relief would not be permitted under judicial
immunity.
That leaves the state largely undisturbed in regulating

The Unauthorized Practice of Law.

Admission to Practice.

Discipline.
And we study those regulatory matters in succeeding sections.

10

11

1
I

Section C. Admission to Practice


A
The state Supreme Court controls admission to the practice from the moment someone enters law
school.
A
As a result, students often complete, either before or in the first year and often again in the third year,
some type of application to be a law student or for admission to the bar--here insert local rules.
A
This is part of "character and fitness" inquiry.
A
Character and fitness review is part of the court's regulatory power.
1
It is limited by Federal Court only to the extent indicated in cases like Schware, Note 1 following
Mustafa.
1
Any inquiry must have a "rational relationship" to the qualities necessary to the practice of law.

Problem, question (a) involves 2 issues


1
First, is either the plagiarism or the "drunk and disorderly" charge arising out of the football
incident "rationally related" to the practice of law?
a
This is a factual question.
a
You can see the kind of matters considered by the court in In re Mustafa, where student
had

Outstanding law school record.

Intent to pay money back.

Immediate admission and contrition.

No criminal conviction.

Yet was denied admission to the bar.

Was this student admitted? Why not?

He stole money.

Attorney would normally be disbarredor at least suspended for at least 5 years


for such conduct.

Cannot allow this student in.


2
Second issue is that covered in Question (b): who must prove character.

As Mustafa suggests--burden is upon applicant to demonstrate good moral character;


court uses language "Mustafa has failed to establish that he has the ...;"

It could be important who has the burden in a close case. Can bar here prove that Mustafa
has bad moral character?

Does this mean, then, that applicants should not admit such matters on their bar
application?

Model Rule 8.1 makes it a disciplinary offense to fail to report "material" information to
the licensing authorities.

Whether something is "material" is a fact question that the applicant cannot truly answer.

Thus, the better position is to report anything that could remotely be considered
"material."

There would be no statute of limitations.

Note the interplay of this Rule and the protections of the Fifth Amendment,
Comment [2] to Rule 8.1.
3
In most instances, when an applicant files the application, no problems are indicated and the
matter is approved.
4
When an application is questioned, because of information on it, the application is, in Missouri,
often sent to a Circuit Bar Committee (to be discussed in Discipline section) for investigation.
5
The normal practice is for the Circuit Bar Committee to contact the applicant.
6
Recently, the Missouri Supreme Court has started using the services of the National Conference
of Bar Examiners to do some of the character and fitness investigation.
7
Whenever or however the applicant is contacted, the Burden is on the Applicant to prove his/her
character and fitness.
8
Missouri Rules now envision certain presumptions.

Missouri Supreme Court Rule 8.05: Qualifications to take the bar examination.

If plead or found guilty of felony:

not eligible until later of

five years after the plea, OR

Five years after completion of sentence or period of probation.

If denied registration as a law student.

Five years after denial unless the Court or board specifies shorter period.

If admitted in another state and complaint pending there or suspended there,

not until complaint resolved, OR

Suspension is over.

So how would the problem turn out?

Several courts have declined to find an isolated drunk driving conviction rationally
related.

If that conviction showed a pattern of such conduct, the result would likely be different.
1
You should take from Mustafa the notion that almost anything can be reviewed in the character
and fitness inquiry:

Confidential juvenile material.

Confidential military records.

Civil cases.

Ordinance violations.

Hospitalizations
1
This should convey the notion that the Practice of Law is not so much a right as it is a privilege.
Mustafa and question (a) relates to defined conduct committed by an applicant. What about question
(c), which does not involve conduct necessarily, but primarily, involves thought.
1
First, consider Sarah Baird, Note 4 following Mustafa.

Baird refused to answer a question about being either a communist or being a member of
an organization that advocates overthrow of the United States government.

At oral argument in U.S. Supreme Court, her counsel gave as an example the story of an
avowed racist.

And, perhaps as a result, the Court held that views and beliefs are immune from inquiry.

I think we have to assume that views and beliefs are immune when they are
unaccompanied by action in support of those views and beliefs.
1
Then, consider the case of Matthew Hale, Notes 5-7 following Mustafa.

Hale is the avowed racist forecast by Sarah Bairds lawyer almost 30 years earlier.

Yet, at least at the time of his initial hearing, he had not apparently acted on any of his
avowed racist beliefsand here you cannot consider things that the bar committee thought
had no significance, as indicated in Note 5 following Mustafa.

Illinois turned him down. At this stage, was there anything different between Hales case
and Bairds case? Presumably not, but the facts changed.

While Hale appealed to the Illinois Supreme Court, one of Hales followers went on a
shooting spree in Illinois, killing others, including the Northwestern University basketball
coach, before killing himself.

Note 6 following Mustafa details the relationship between Smith, the shooter, and
Hale.

Is this relationship enough to take the case from beliefswhich are immune
from inquiryto action which is not?

One can only presume so as the U.S. Supreme Court denied certiorari.

A postscript:

In January, 2003, Hale (PG 66) was arrested in Chicago for soliciting the murder
of a federal judge who sought to hold Hale in contempt in a civil proceeding. Matt
OConnor, Hale held in plot to kill judge, Chicago Tribune, Jan. 9, 2003, 2003 WL 3741956.
The allegedly threatened judge later sanctioned Hale and his Church $200,000 for
violating an injunction. Matt OConnor, Judge imposes sanctions on Hale, Chicago
Tribune, Oct. 26, 2003, 2003 WL 66437120. Prosecutors later amended the charge against
Hale to include a charge that Hale, while being held in jail on the murder plot,
instructed his father to lie to a grand jury apparently investigating the relationship
between Hale and the Chicago shooter Smith. Matt OConnor, U.S. files new charge
against Hale, Chicago Tribune, Aug. 29, 2003, 2003 WL 62154769.

The case was tried in April, 2004 and Hale was convicted of two different counts
of soliciting to have a federal judge killed and three counts of obstructing justice in
conjunction with covering up Hales connection to Smith. Hale guilty on 4 counts; Jurors

say e-mail, tapes sealed verdict in plot to kill judge, Chicago Tribune, April 27, 2004, 2004
WL 77186928.
7
Finally, consider admission and federal preemption; please refer to Note 9 following Mustafa.
1
The Americans with Disability Act does apply to the bar admission and application process.
2
A court cannot disqualify someone with a mental illness, but:

can inquire about it; AND

Can disqualify if the illness results in conduct that is inconsistent with the duties of a
lawyer.
Section D. Unauthorized Practice
A
Problemquestion 1 asks you to prepare a preliminary reportcertainly that report will include a
discussion of what is Unauthorized Practice;
MODEL RULE 5.5 outlines the unauthorized practice of law
-> Restatement Section 4: A person not admitted to practice as a lawyer (see section 2) may not
engage in the unauthorized practice of law, and a lawyer may not assist a person to do so.
--- this language is shaky, because then if a person is not admitted to the practice, then they can
engage in the unauthorized practice of law, well no, that's not true everyone knows what this
means.
--- there are criminal and misdemeanor actions by state for people who are not lawyers to deal
with their unauthorized practice of law
1

United States v. Johnson.


a
Facts.

National Legal Professional Associates is an Ohio firm that provides consulting services in
criminal cases.

Its owner/operator (Robinson) is a disbarred lawyer.

NLPA apparently markets itself to criminal defendants either through their families or
through some sort of direct contact with jails.

However, note who hires themthey are only hired by the lawyerwhy?

This is an attempt to insulate NLPA from unauthorized practice.

By attempting to serve as an employee/consultant to the lawyer and not an


independent actor.
a
What do they do?

It is a little unclear, but they seem to be recommending strategic and procedural


recommendations to criminal defendants.

If a family member or friend makes such recommendations, is that the unauthorized


practice in every case?

No, and the court indicates that the performance of activities requiring the legal
skill and knowledge of an attorney, by a non-attorney, are o.k. IF performed under
the direct supervision of an attorney.

The real key here is that the recommendations being made by NLPA are being done in
a way that undermines the attorney-client relationshipinstead of being done under the
supervision of a lawyer, the NLPA activities are being done in spite of the
recommendations of the lawyer.

That means that NLPA is essentially acting on its own.


a

How does court define the practice of law?

Services performed in court proceedings.

The courts Note 10 adds to this definition:

Services performed incident to court proceedings, even if performed out of court.


Providing any service requiring the use of any legal knowledge or skill.

Conveyancing.

Here, then the court finds that the absence of any meaningful attorney supervision
renders the activities of NLPA the unauthorized practice of law.

Preparation of all legal instruments whereby a legal right is secured.


Giving opinions on title to realty.

You will note how indefinite the courts definition of unauthorized practice is; should there be
a uniform definition?
a
This is Note 2, following Johnson.
a
There have been attempts and even some states have adopted a uniform definition, but
most places continue with a more indefinite understanding of the concept.
a
Many states also have criminal legislative provisions defining the unauthorized practice
and making that activity a crime.

Obviously, those definitions are not binding on a court exercising inherent power,

But many courts look at those definitions.

In most states, however, prosecution for unauthorized practice, under the criminal laws, is
very rareleaving most of the work in this area to discipline by the bar or independent
action to enjoin non lawyers from unauthorized practice.
Before going further, lets look at Note 1 following Johnson.
a
What is the point of unauthorized practice rules?
a
It would be easy to see them as just more guild legislation.
a
But look at Florida Bar v. Schramek, Note 1 following Johnson.

Supreme Court has supervisory control over judicial system.

Exercises that control through attorney regulation.

As a result of exercising that supervisory control, Court is able to keep jurisdiction over
attorneys.

If anyone, like Schramek, is able to tell people he has the power of an attorney, the Court
may not be able to exercise control over that person as easily; this the Supreme Court
cannot allow.
Now look at the activities in the problem:
NOTE: selling of kits, forms and manuals with general advice is NOT the practice of law but providing
advice specific to a particular document/form or issue IS the practice of law.
a

Divorce Kits, Note 3 following Johnson.

Selling kits and providing formseven filling out formsparticularly as part of another
business like a real estate businessis not the unauthorized practice, generally if:

The forms were originally prepared by a lawyer.


The person selling the form does not give advice about the effect of the form.
The person filling out the form does not render a separate charge for filling out the
form.

Thus, when we look at Grants work in the living trust seminars, we need more
information.

Is he providing advice?
Is he charging a separate fee for filing out the forms?
If the answer is yes to either, it is more of a problem than just selling the divorce
kits;

Here, we must then look at the conduct of Angela Hazendorf, who is an attorney and who
is allowing Grant to sell the living trust kits at seminars she sponsors.

This is Note 7 following Johnson: The pre-2002 version of Rule 5.5(a) made it
impermissible for a lawyer to aid another in the Unauthorized Practice; In
the post-2002 version, Comment [1] makes it clear that the revised language
also applies to the lawyer assisting another; This is different from
performing services under the supervision of an attorney; Lawyers aid in
unauthorized practice if they assist another and dont supervise them.

This would also include lawyers aiding disbarred or suspended lawyers as

noted in Juhnke, Note 7.


Activities for Landlords.

First, obtaining power of attorney;

Having a power of attorney does not allow a person to practice law.


It generally authorizes the attorney in fact to sign papers in place of another person,
but it does not allow the giving of advice.***

Second, appearing for landlords.

As power of attorney, Grant could not represent landlords in proceedings, but


he could appear if the landlords could appear.

That raises the pro se exception, Note 6 following Johnson.

A person can always represent themselves.


But corporations and other legal entities are not persons, and the law does
not allow those entities to appear in person or to be represented pro se;

Thus, those landlords that are corporations would have to be represented by


an attorney at lawand Grant with his power of attorney would not be
one.

A couple of other Notes.


1
First, Note 5 following Johnson.
a
What about somebody who maintains a web site that excoriates local judges and gives
general advice about what to do to get around judicial proceedings?
a
Initially, there must be a First Amendment concern.
a
Here, the Ohio opinion, however, focuses on the fact that the advice is very general and
could be found in almost any over-the-counter book about the lawwhich anyone could
check out from a library or buy in a bookstore.
a
This kind of generality the decision is unwilling to find as the unauthorized practice.
a
When would they be willing to find unauthorized practice?

When the advice is tailored to a specific person.

As long as the advice is general, using hypos, not case and person specific or
specific question with specific answer, then it is not legal advice and is therefore not
unauthorized.
1
Second, look at Note 4 following Johnson.

When law students are appearing for clients, that is the unauthorized practice of law.

Unless they are appearing under some authorized student practice ruleat which point it
is not the unauthorized practice.

Thus, if a law student were to call up a creditor for one of the students parents and tell the
creditor that the law student was a student and that what the creditor was doing was
illegal, that would likely be the unauthorized practice.
Multijurisdictional Practice.

1
1
1
1

As lawyers, we are admitted to practice in places where we have either taken the bar examination
and been admitted, or.
Where we are admitted by some sort of reciprocity; or
Where we are admitted to for one or a couple of mattersadmission pro hac vice.
The Problem introducing this section is very simple.
a
None of the lawyers are licensed in California.
a
Is what they did the practice of law as we have defined it? Of course, they advised on a
legal dispute the corporation had in California.
a
Under the statute, then, these lawyers are not entitled to recover a fee; and that is exactly
what happened in the case on which the problem is basedBirbower, Note 1 following
Richards and ONeil.
a
Problem, question (a) is the real issue we want to talk about; If Birbower is the law, how
can lawyers practice in a modern world where many transactions take place over state
lines.
Richards and ONeil v. Conk.
a
Facts.

Conk sold Day Dream to Cullman.

Conk and Day Dream were from Indiana.

Cullman was from New York and retained Richards and ONeil, New York lawyers, to
represent them.

Following the sale, a dispute arose between Cullman and the selling shareholders of Day
Dreaman arbitration proceeding was filed in New York and was settled.

Another lawsuit was filed in Indiana by Cullman and Day Dream against ConkRichards
and ONeil appeared pro hac vicethis lawsuit was dismissed.

Yet another lawsuit filed in Indianaby Conk against the Chief Financial Officer of Day
Dream and against Richards and ONeil.
a
Richards and ONeil move to dismiss claiming an absence of personal jurisdiction.

This motion is about contacts;

Richards and ONeil has no office in Indiana.

No advertising and no solicitation there (although they did appear pro hac vicethe
assumption is that a pro hac vice would be limited to appearing in that case and would
not be an appearance for jurisdiction purposesif that is even possibleand well discuss
that shortly).

Court therefore dismisses case.

Judge Najams concurrence.

Look at Rule 5.5(c).

That rule clearly allows lawyers admitted to one jurisdiction to appear in other
jurisdictions, under the circumstances mentioned in the ABA Commission Report and
contained now in Rule 5.5(c)(1-4).

Judge Najams thus finds that the activities of Richards and ONeil were not the
unauthorized practice; He notes that Indiana does not have the current version of Rule 5.5,
but that does not seem to bother him much.

But then look at the final paragraph of concurrence.

Judge Najam is warning lawyers about dangers of practicing in foreign


jurisdictions.

Because of potential of establishing minimum contacts there.

But if there are no minimum contacts established by Rule 5.5 practice in another
jurisdiction, isnt that a bad thing for consumers of legal services? That is, should
Conk have to go to New York to sue Richards and ONeil for activities that took
place in Indiana?

As this opinion should make clear, the issue of multijurisdictional practice is a relatively
new one (at the time of the writing of this edition) that is likely to continue to develop.

Please look at Note 2 following Richards and ONeil; in the first litigation in Richards and
ONeil, the law firm appeared pro hac vice; what does that mean?

According to my version of Blacks Law Dictionary, it means for this one particular
event, and that certainly is the way courts interpret it.

When you know you are going to be in a state in which you are not admitted, you seek
admission from the local court to appear pro hac vice for the one particular case you are
working with.

You must do so through a local attorney; Why?

So court has somebody to go after in the event things go wrong.

There is certainly personal jurisdiction over local counsel.

If an out of state lawyer is admitted in another state, and is in good standing in


that other state, that is generally a good barometer of what the lawyer will do in the
admitting state; If the attorney is in good standing, court will generally admit pro
hac vicebut this is clearly discretionary, particularly as the Note indicates where
there are instances of procedural or other abuse.

As contained in Note 3 following Richards and ONeil, pro hac vice admission is difficult
in cases where there is no litigationbecause no court to whom to apply.

And see Note 4 following Richards and ONeil, in a criminal case, failure to admit a
particular lawyer may implicate the 6th Amendment by refusing a lawyer of a defendant's
choice.
Section E. Discipline
A
Generally, state Supreme Court exercises "exclusive control"
1
This applies to lawyers who appear in court.
1
To lawyers in office practice who never appear in court.
1
To lawyers in their non-practice life--that is lawyer can be disciplined as a lawyer for acts
committed as a private citizen.

Disciplinary Structure--using Missouri as an example.


1
The entire procedure is controlled by the Supreme Court of Missouri and appears as part of Rule
5 of the Missouri Supreme Court Rules.
1
Missouri, in keeping with recommendations from the American Bar Association, and following
the lead of other states, has a two-agency structured disciplinary procedure.
a
Bar Committees

Fact finders appointed in each circuit.

Advisory Committee has statewide jurisdiction, usually to render opinions, but can also
hear cases.

Members appointed by Supreme Court, not local bar.

Find facts and make determination of Probable Cause that misconduct occurred.

In Missouri, as most states, must contain non-lawyer members.

Issue information to Supreme Court.

Court normally, but is not required to, appoints special master, to find facts and
make recommendation of discipline.

Ultimate fact finding and discipline are province of Court.


a
Chief Disciplinary Counselcreated to separate prosecutorial function from fact-finding
function.

Investigates complaints against lawyers.

Prosecutes those cases, normally before circuit bar committees.

The process is ongoing.

In 2002, appointed an Ethics Consultant, who will write advisory opinionsremoving that
function from OCDC and will consult with lawyers about ethics matters.

Supreme Court, in 2003, adopted an intervention program, that allows some ethics
matters to be processed in a slightly different fashion, to avoid the stigma of a violation.
Consider the problem
Look at Question (b), relates to sanctions--the general range of sanctions for disciplinary violations and
the factors to be considered in determining the applicable sanction is contained in Note 2 following In re
Advisory Panel.
1
Also consider the process that is due; Note 3 following In re Advisory Panel
a
Disciplinary proceedings are quasi-criminal.
a
Lawyer gets some procedural due process.

Notice

Opportunity to be heard.

Prohibition against amending charges without giving lawyer an opportunity to respond.


But some criminal procedure does NOT apply.

No presumption of innocence.

No proof beyond a reasonable doubt.

No right of confrontation.

NOTE: 5th amendment is there if the answer to a question is incriminating, the attorney could be
criminally used against him (incriminating) then he can refuse to answer the question on grounds
of self-incrimination under the 5th amendment.***

Some students will ask about discipline in relation to criminal proceedings.

Where a criminal trial results in a conviction, the conduct is subject to issue preclusion in a
subsequent ethics proceeding because the standard of proof in the ethics proceeding is
rarely beyond a reasonable doubt, and even if it is, that standard has been met.

As a result, following criminal conviction, and in most states, even while the case
is pending appeal, there can be a summary disciplinary proceeding, usually in the
Supreme Court, in which a disciplinary sanction is imposed for the violation.

The disciplinary proceeding cannot be used as a collateral attack of the criminal


conviction.

Where the criminal case results in acquittal, the acquittal does not subject the disciplinary
proceeding to issue preclusion unless in the disciplinary case, the standard of proof is
beyond a reasonable doubt;

If the standard in the disciplinary proceeding is lower than beyond a reasonable


doubt, the criminal case did not decide if the lower standard could be met.

At that point, a full ethics hearing can be held so as to assess the evidence under
the lower standard.
Question (a)--Obligation to turn in another lawyer.

A
A

What rule governs reporting professional misconduct of another lawyer or judge - RULE 8.3
NOTE: privilege is not a rule of prof resp, it is a rule of evidence.
Also NOTE: in quasi-criminal discipline proceedings, the lawyer is "entitled to [limited] procedural due process,
which includes fair notice of the charge."
1

In re Ethics Advisory Panel


a
Facts:

Attorney X settled a case on behalf of a client.

The settlement funds were given to Attorney X.

Attorney X stole the funds.

Inquiring Attorney, whose client is entitled to the funds, asked the client about turning
Attorney X over to authorities.
Client assumed that such action would compromise the clients ability to get anything and
would not authorized Inquiring Attorney to turn Attorney X in.
Inquiring Attorney wants to know if there is any duty even in the absence of client
consent.
Court looks at Rule 8.3.
Under Rule 8.3(a), is there an obligation?

Yes, the lawyer knows

Here look at Note 1 following opinion which requires a firm factual basis to
support knowledge.

There does not appear to be a need for personal knowledge, but must be
more than any information;

Contrast that with Rule 1.0(f) apparently rejecting this standard in favor of
an actual knowledge that can be based on circumstantial evidence standard.

In addition to the lawyer knowing, the offense is also one that raises a substantial
question of Attorney Xs trustworthiness.
So why does the court find no duty to report?

Rule 8.3(c)no duty to report if doing so would reveal information protected by


Rule 1.6.

Rule 1.6 [Confidentiality] includes more information than would be covered by


"privilege;"

Rule 1.6 will not authorize release of information if client will not consent. Thus,
rule 1.6 of confidentiality trumps 8.3( c)'s duty to report

Because client will not consent, lawyer has no duty, even though conduct of
Attorney X violates the criminal law.

Rule 8.3 shifts the focus from the lawyer as officer of the system back to lawyer
as advisor of clients; one can only suspect that our detractorsthose who see the
practice as being part of a guild will not see this rule in this light.

Question (c)What duties do judges have? Note 5 following In re Advisory Panel


1
Canon 3(D)(1) and (2) of the Code of Judicial Conduct are very similar to Rule 8.3, but does not
have the exception for client confidential information.
1
Judges have duty to turn in other judges as well.
1
And here consider the lawyers duty to turn in judges, Note 4 following In re Advisory Panel.
a
Lawyers have same duties to turn in judges as they do to turn in other lawyers.
a
The Note details what happened to at least one lawyer who refused to do so.

Question (d)Reciprocal disciplinary rulesNote 7 following In re Advisory Panel.


1
Rule 8.5(a) says lawyer can be disciplined in any jurisdiction where lawyer is admitted,
regardless of where conduct took placeeven if the lawyer is not admitted, the lawyer can be
disciplined in a state if the lawyer provides any legal services in that jurisdiction
1
Rule 8.5(b) deals with which states rules apply.
a
If misconduct takes place before a tribunal, the rules of state in which the tribunal sits,
unless local court rule provides otherwise.
a
If the misconduct takes place out of court.

The rules of the jurisdiction in which the conduct took place apply; HOWEVER.

If the predominant effect of the conduct takes place in another jurisdiction, then that
states rules apply.

Question (e)Reciprocal disciplineNote 6 following In re Advisory Panel.

1
1
1
1
1

When disciplined in one state, most states require lawyers to report that discipline to all other
states in which lawyer is admitted.
Done by local court rulenot by Rules of Professional Conduct.
Failure to report is an additional violation.***
Standards of discipline encourage, but do not require a hearing.
Most states impose the same discipline and make it concurrent with the early disciplining states
discipline.

And finally take a look at Note 8 following In re Advisory Panel.


1
When a lawyer is admitted pro hac vice for the purpose of doing one case in a foreign
jurisdiction, discipline normally comes from the court admitting that lawyer, rather than the
disciplinary authority of the state in which the lawyer has the temporary practice. Under Rule
8.5(a), however, the out of state lawyer does subject him/her self to the disciplinary of the state.
a
Pro hac vice:
is a legal term usually referring to a lawyer who has not been admitted to practice in a certain
jurisdiction but has been allowed to participate in a particular case in that jurisdiction.
1
Discipline generally comes from the court revoking pro hac vice status, but note.
1
The lawyer must be given notice and an opportunity to defend the charge that is causing the
status to be revoked.

Section F. The Lawyer as Member or Employee of a Law Firm


A
Problem, Question (a)--transforming the firm into one of the statutory limited-liability models.
1
Rule 5.4(d) permits lawyers to practice in a "professional corporation or association" so long as
only lawyers own interests and serve on the board thereof.
a
Whether this rule would extend to the newer form of "limited liability company"
authorized by statute in some states is an open question, but Rule 1.0(c) defining firm
seems to anticipate that lawyers will be organized in all kinds of legal forms.
a
Henderson v. HSI Financial helps.

Facts

One lawyer in a firm represented a client and was to remit funds to that
client.

Evidently the lawyer did not and client sued lawyer and firm.

Firm was incorporated under one of Georgias limited liability statutes


and sought to have case against the firm and the other two lawyers dismissed.

Court overrules First Bank & Trust Co. v. Zagoria, in which the court had reviewed
the applicability of limited liability statutes for lawyers on an inherent power basis finding
that legislature cannot exempt lawyers from liability if the court does not want that to be so; Court
found that trust in profession requires all lawyers in a firm to be individually liable, regardless of
the firms business structure.

What article does it appear the Georgia Supreme Court read between Zagoria and
Henderson?

It looks like they read Wolfram.

The court appears to move away from the negative inherent power by
holding that it is the legislature that determines the ability of lawyers to insulate
themselves from personal liability.

In fact, a reading of this paragraph should give you the idea that the
court is no longer using negative inherent power.

But is that really what the court does?

No, the next paragraph is much more consistent with traditional


affirmative and negative inherent power.

The court says: Exercising our regulatory power,

So look at Note 1 following Henderson.


Isnt the court saying:
We can act, but we are not going to do so because we believe
what the legislature did is satisfactory;

In terms of firm structure, however, this opinion acknowledges the opinion of the ABA
and allows lawyers to form limited liability partnerships.

The liability of members is then set out in Note 3 following Henderson, looking at the
provisions of the Restatement.

If a firm is traditionally organized as a partnershipeach of partners is liable for


the conduct of each of the other partners.

If organized as a limited liability entity.

An entity is not generally liable for acts of individual members.

The non-involved individual members are not generally liable.

The involved members are liable.

Here, see Rule 1.8(h) which prohibits a lawyer from making


any agreement prospectively limiting the lawyer's liability for malpractice,
unless.

The client consents after being independently represented in


making the agreement.

Most clients would not agree to such an arrangement.


Question (b)Law Related servicesStart with Rule 5.7

What are law-related services?Rule 5.7(b)

Services that are related to legal services but which if performed by a non-lawyer would
not be the unauthorized practice of law.

Law related services are services related to legal services but are not actually legal
services so if performed by a non-attorney, not be the unauthorized practice of law
"services that could be provided in conjunction with and in substance are related to legal
services."

Title company; real estate; financial services; sports agenthere look at Note 5 following
Formal Opinion 2003-1 and see the kinds of law-related services about which inquiry was
made in Florida.

So, if they can be performed by non-lawyers, why do we care?

Because of Rule 5.7(a).

Under 5.7(a), if a lawyer opens a magnetic resonance imaging center (Note 5


following Formal Opinion 2003-1), the lawyer is still subject to the Rules of Professional
Conduct in operating that magnetic resonance imaging center if:

The lawyer operates the magnetic resonance imaging center in a way


that is not distinct from the operation of the lawyers office; OR

Even if the lawyer does operate the imaging center distinct from the law
office and runs it with others, if the lawyer does not take reasonable measures to
make sure the clients of the magnetic resonance imaging center know that the
lawyer-client relationship does not apply.

And note that Rule 5.7(a)(2) envisions the lawyer operating a law-related
service with others, presumably including non-lawyers, if you look at the kinds of
businesses about which Florida received inquiries.

*** confidentiality may be assumed to apply unless you make it clear


that the services are not legal services and therefore protections of the client-lawyer
relationship do not exist!

Think about that in relation to Rule 5.4(a) and (b).

Lawyer may only share legal fees with a non-lawyer under very limited
circumstances.

Lawyer cannot form a partnership with a non-lawyer if any of the


activities consist of the practice of law; Why?

Probably because the court will not have control over those nonpractitioners.
Thus, under the Rules, the only time a lawyer can be in business with a nonlawyer is either in a non-law business, in which it is clear that the lawyer is not acting
as a lawyeror in a law-related business and then the lawyer must be mindful of Rule
5.7.

Formal Opinion 2003-1 helps us understand some of those situations.

Lawyer represents seller of business and wants to help locate a buyer and then
charge a fee based on a percentage of the transaction price (essentially a brokers
fee).

Here, can provide such a service, but subject to the Rules of Professional
Conduct.

As a result, any fee will have to be reasonable in light of factors in Rule 1.5
(Ohio does not discuss this, but an essentially contingent fee based on the
transaction is not prohibited to lawyersbut it will have to be reasonable in
light of the Rule 1.5 factors).

Lawyer represents buyer and wants to locate a seller and charge buyer a transaction
fee.

Again must be reasonable (Rule 1.5).


But must also make sure that seller does not think lawyer represents seller
under Rule 4.3

Both of these requirements are because the lawyer is doing the law-related
service as a lawyer and is subject to Rules.

Lawyer wants to use a business broker to locate a buyer and then split the fee with
the broker.

Because lawyer is a lawyer and governed by Rules of Professional Conduct,


Rule 5.4 applies.

No portion of 5.4 will allow this type of fee split.

The lawyer wants to refer a business entity to a lender and receive a fee from the
lender.

Committee has trouble with this because it does not know the role of the
lawyer.

The lawyer exercises judgment in selecting a lenderpart of that judgment


will be impaired by the lender who gives the lawyer the most money.

If the business is relying on the lawyers judgment, that judgment must be


independent (Rule 2.1) and if the judgment of the lawyer is compromised by
the lawyers own dealings, there is a conflict of interest under Rule 1.7(a)(2);
If, however, the business is also a client, there is a conflict between the
lawyers one clientthe bank, and the lawyers other client, the business
entityalso under Rule 1.7(a)(2).

Lawyer wants to find a seller on behalf of an investment group and then represent
the seller.

The Committee sees this the same as the previous problem.

If the lawyer is being asked to provide independent professional judgment


for the investment group, that independent judgment is compromised by
representing the seller; if the lawyer is representing both, there is a client
conflict.

Lawyer wants to open an ancillary business to locate buyers and sellers of


businesses (essentially a business brokerage firm) and charge a transaction fee.

Here, Rule 5.7(a) controls.


There is nothing wrong with lawyers opening businesses.
But to keep from having to operate under the Rules of Professional Conduct,
the business must be separate from the lawyers law practice, and the
opinion adds to Rule 5.7(a) by telling us:

Neither the lawyer nor the business can require legal representation.
The lawyer must disclose ancillary business ownership.

Notes following this opinion are mostly historicalas they must bethis area of practice is

just beginning to unfold and you will see lots of information on all these topics in the
future.
Question (c)--What can an Associate do when a senior partner directs the Associate to do what
the Associate believes is unethical conduct?
a
First, the Senior Lawyers are ethically responsible, under Rules 5.1 & 5.3 (this includes
Managing partners and others with managerial responsibility) for the conduct of
associates and non-lawyer assistants.
---> keep in mind, its not a course in ethics, its in lawyer conduct! So therefore don't
say associate is being asked to do something "unethical" but rather something
against attorney conduct rules

This is different than liability for negligence, which attaches through tort doctrine of
respondeat superior.

Ethics responsibility attaches to supervising lawyer if:

Supervisor ordered the unethical conduct, OR


Supervisor knew of unethical conduct in time to do something about the conduct,
and did nothing.

This is a standard more protecting of the employer than the tort standard of
respondeat superior.

Look at Note 4 following Jacobson; Supervising Lawyer must take reasonable measures
to monitor the activities of supervised lawyers and non-lawyer assistants.

Associates are in a difficult position under Rule 5.2.

Generally, the Associate feels that it is necessary to follow orders or get fired,
The Rules permit a subordinate lawyer to follow an order of a superior lawyer if the order
to be followed involves a questionable one of ethical duty.

MR 5.2 holds an attorney responsible for complying with the MR even if someone ordered
an attorney to engage in misconduct in question (under subsection (a))--- difficult position
because do you blow the whistle on your firm/boss as a young/new attorney???

Where there is no question that conduct is not permitted, the lawyer cannot follow
orders.

Rather, the Associate is obligated to follow the rules.

This is the problem in Jacobson v. Knepper & Mogathis may be the most stringent outcome
of any case that we read all semester.

Facts.

This can present a problem if the firm then fires or threatens to fire the lawyer.

An Associate discovers that firm is filing pleadings that violate federal law.
The Associate makes three attempts to tell Senior Partnerwho promises Associate
that the practice will stop.

After the third attempt, Associate is fired.


The Associate files suit for wrongful or retaliatory discharge.

The Illinois court gives us the basic law.

Most lawyers are "at will" employees so that they can be hired or fired without
cause.

The court recognizes that there are some exceptions to the at will doctrine.
Where discharge is in retaliation for conduct and in contravention of a clearly
mandated public policy, then.

The court says that such exceptions are necessary to assure that societal interest in
public policy is carried out, while still allowing an employer to run a business and
an employee to earn a living.

You should have thought: Is there a more clearly mandated public policy than preventing
a lawyer from violating the Rules of Professional Conduct?

But the court says no.


The Associate had a duty under Rule 8.3 to turn in the Senior lawyer.
Because of that duty, the public is fully protected and no need to allow this cause of
action.

The analysis of this case is best summed up by the points in Note 2 following the case.

To which can be added that not only is the Associate still discharged.
But, he also faces discipline for not turning in his Senior partner.
Something no Associate is going to do.

Not all cases are as stringent.

Wieder v. Skala, Note 3 following Jacobson, changed the at will doctrine, at least in
N.Y.

When one is hired as a lawyer, an implied part of the agreement is to act ethically.
Preventing the lawyer from acting ethically frustrates the agreement and is
actionable.

This case, and others, recognizes an implied obligation on part of both parties to act
in good faith.

The Restatement does not take a position on this issue, finding that lawyers are
subject to local law in dealing with the employment relationship, Restatement
(Third) of the Law Governing Lawyers, 56 (Reporters note k).

Note 5 following Jacobson gives the similar resolution to the problem for lawyers who are
Inside or House Counsel.

Illinois denies a claim for retaliatory discharge.


Other states have allowed a retaliatory discharge claim.
Even where such a claim is allowed, however, the lawyer may still be required to

withdraw from representing the corporate clientthis because the lawyer has been
discharged and withdrawal is mandatory under Rule 1.16(a)(3).
Question (d)--restrictions on post-departure practice; here, review the problem to see what
is being discussed.
No advertising that can be viewed within 10 miles of the main office of the firm; (this
provision may fail under the general law because of its absence of time constraint and we
should probably assume a reasonable period of timelike two years).

Cannot accept a client you worked for while at the firm for two years without the written
consent of the firm.

If you stay 5 years and leave, you get.

2 months salary as a leaving bonus.


1 additional month times the number of years worked if you do not practice law in
the state for two years.

If you do practice law, you lose the 1 additional month for not practicing, but still
get the 2 months as a leaving bonus.

First, look at Rule 5.6.

It prevents lawyers from making partnership or employment agreements or settling


litigation in a way that provides any restriction on the ability of a lawyer to practice.

It does allow an agreement that restricts the lawyers right to practice after
retirement.

Jacob v. Norris, McLaughlin & Marcus

Lawyers left one firm and started another.


Lawyers took clients with billings of about $500,000.
Lawyers had participation agreements with the firm that provided for certain
compensation if lawyer left firm.

Return of equity interest in firm.


Purchase of shares of member at a reasonable price
Additional compensation that could be, among other things, either
competitive voluntary or non-competitive voluntary departure.

Distinction between competitive voluntary and non-competitive voluntary.

Under competitive, the member receives no additional amount.


Competitive voluntary departure is that which either provides legal services
to firm clients or encourages members or employees of the firm to join in the
departure.

There is no question in this case but that the departure of the lawyers was a
competitive voluntary departure.

But lawyers want compensation based on non-competitive, claiming that the


provisions of the participation agreements violate Rule 5.6.

Court here tells us that Rule 5.6 applies not only to direct prohibitions on practice,
but to indirect ones as welland that includes taking members of the staff; why
does the Rule exist?

To allow the client freedom of choice.

If lawyer leaves, client cannot be penalized.


Indirect provisions force lawyer to choose between continuing to serve their
clients (which is their job) and moving.

Defendant here claims provisions are reasonable economicallyand court does not
disagree.

But instead reminds us that the practice of law is not consistently a


commercial enterprise.

And the real issue is protecting client choice.

Can firms do anything to prevent wholesale leaving?

Yes.
Court allows the firm to take into consideration the economic impact of a
lawyer (or staff) leaving in deciding the value of the leaving lawyers
ownership interest.

You can see the universality of the ban on restrictive covenants in Note 2 following Jacob.
Note 1 following the case mentions Blackburn v. Sweeney, which is another indirect
restriction caseand answers our question about advertising. It holds that an agreement
restricting the right to advertise violates the provision.

Note 3 following Jacob discusses the retirement exception;

Under it, would the provision in the agreement in our problem be permitted?
Probably, the lawyer could be forced to lose the distinct amount for retirement;
But not the amount for longevity;

Unless the provision was a sham and the court viewed it like the court in Gray v.

Martin, Note 3.
Beyond what is permitted by Rule 5.6, there are no general restrictions on leaving; look at
Note 5 following Jacob and the cited provisions of the Restatement.

When you are getting ready to leave a firm, you cannot solicit clients for your new firm
UNLESS you advise your existing firm (You do not need permission under the
Restatement).

After you leave, you are free to solicit clients within the limits of the advertising and
solicitation rules we will discuss in Chapter 3.

NOTE: This is a Restatement provision ONLYthere is nothing in the Rules that says this
but there is nothing in the Rules that says anything contrary either.

You should also look at Rule 1.17 on selling the law practiceit provides some restraints on

the lawyer and makes sure that a lawyer selling a practice is NOT selling the clients to the
buyer.
Chapter 3: The Attorney-Client Relationship
Section A. Attracting Clients: Advertising and Solicitation
A
The Profession of Law traditionally shunned advertising and solicitation, believing that the relationship
of attorney and client is a uniquely personal one that cannot be developed through the use of
advertisements.
A
The U.S. Supreme Court has approved, however, the business aspects of the profession by clearly
holding that truthful, non-deceptive advertising by lawyers is protected commercial speech under the
First Amendment.
A
This then creates a tension within the profession.
1
Advertising and solicitation are permitted to some extent, however many continue to believe that
neither should be used.

As a result, those who believe that neither should be used have tried to restrict the amount of
permissible advertising and solicitation within the rules,
1
This has led to a number of cases in which the Courts have struck down rules that tend to too
narrowly restrict an attorney's first amendment rights.
Problem,
1
You obtain an "800" number and advertise using "1-800-LAWSUIT"--is there anything wrong
with this?
1
Florida Bar v. Went For It, Inc..

Advertising by lawyers is protected commercial speech, although of recent


vintage.

Test of protection stems from earlier Court decision in Central Hudson on an


"intermediate scrutiny" basis.

Government can regulate commercial speech that concerns unlawful conduct or


is misleading; As a result, that which is either inherently deceptive or proves deceptive in
fact can be regulated.

Permitted commercial speech can also be regulated, if state meets tests set out
in Central Hudson.

The state must demonstrate substantial interest; here state has two.

Protect people traumatized by injury who might be in a state of


mind to be unable to objectively determine if they need counsel.

Protect personal privacy of persons who might be grieving.

Aren't these legitimate state interests?

Certainly survey shows the concern.

The state must show that regulation directly advances the state interest

Here is where bar's two year survey helps.

Prior cases have merely speculated on this issue.

This case offers concrete evidence.

The regulation must be narrowly drawn

30 days may not be the best, or optimum regulation.

But it is commensurate with and in proportion to the


government interest involved.
1
Would there be similar interests involved in banning an "800" number? What?
1
How about the 2nd idea, sponsoring youth athletic teams and putting your telephone number on
the team's shirts? Isn't the result going to be the same?
a
Are these communications that require the name of at least one attorney responsible for the
content within the meaning of Rule 7.2(c)?
a
Maybe, and this could be a potential problem.
1
The 3rd and 4th ideas: Can you write to people you find in the newspaper who have been injured
in accidents or who need a criminal lawyer?
a
What constraints are contained in Model Rule 7.1-7.3?

7.1 deals with communication generally-


Prohibits all false or misleading communications about the lawyer or the
lawyers' services and defines false or misleading to include statements materially
misrepresenting facts or omitting facts needed to make the statement "as a whole not
materially misleading."

Must tell the truth.

Defines false and misleading in terms of overall impression--thus "I have


never lost an antitrust case" is a factually correct statement that becomes deceptive
when coupled with the fact that I have never had such a case.

7.2 deals with advertising.

7.2(a) subject to the requirements of rule 7.1 and 7.3 a lawyer may advertise
services through public media, such as telephone directory, legal directory, newspaper,
outdoor advertising, radio or tv or through written or recorded communication.
No restriction on medium used.
2002 Rule eliminates need to keep a copy for 2 years and changes
comments.
7.2 (b) a copy or recording of an advertisement or written communication shall
be kept for two years after its last dissemination along with a record of when and where it
was used.
7.2(c ) a lawyer shall not give anything of value to a person for recommending
the lawyer's services

Can have reciprocal referral arrangements but cannot be exclusive


and client must be informed of the existence of the arrangement
7.2 (d) any communication made pursuant to this rule shall include the name
of at least one lawyer responsible for its content.

7.3 deals with solicitation.

Solicitation is generally targeted advertising.


Like the letters here.
Cannot communicate in-person, live telephone or real-time electronic contact BUT
Can write letters so long as (Rule 7.3(b))

Prospective client has not communicated a desire not to be contacted.


So long as the solicitation does not involve coercion, duress or harassment.

These letters would not survive in Florida because of Went for It, at least not in civil
cases, but they might survive in other jurisdictionsthere is nothing in Rule 7.3 that
bans them generally.

Even in Florida, what about letters to person accused of crime?

This is Notes 1-2 following Went for It.


Justice Kennedy dissented in Went for It.
His dissent goes to the essence of advertising.
The restriction in Went for It may prevent those who need lawyers the most
the victims of tragedyfrom actually getting them.

The 4th Circuit weighed in on this debate in Ficker, Note 2 following Went
for It.

There, a ban similar to the one in Went for It could not be sustained where it
prevented lawyers from contacting those accused of crime or traffic
violationsthose people need lawyers, not time to grieve.

Rule 7.4 deals with field of practice advertising and is not covered by the problem; It
allows lawyers to indicate they practice in certain fields, but prohibits mentioning that
lawyer has been certified as a specialist unless the lawyer in fact has been so certified by an
appropriate state authority or one accredited by the ABAand then, the ad needs to
mention the certifying agency.

What insight does Florida Bar v. Went For It, Inc. give you on these rules.

Bar cannot speculate.

Must be real problems with real proof.

This is what distinguishes this case from earlier ones.

Solicitation involving personal contact, however, is different.

Note, Rule 7.3(a) refers to in person solicitation as including telephonic or real-time


electronic solicitation.

Why?

Here, court has said solicitation is inherently deceptive, because there is no


opportunity to control what the lawyer says.

If in writing, can require a copy be kept

If an ad, can require lawyer to keep a copy.

1
a
a

Look at the Notes:


Note 3 applies the Went for It decision to other types of advertising and advertising-like cases.
Note 4 mentions Television and Radio, facebook and twitter now as wellthe rules do not
specifically mention these media, but it is clear they are covered by the permission on
advertising.

Note 5 mentions Firm or Trade Names ***SEE MR 7.5 Names like the Suit Shop; These are
not protected as commercial speech, however the rules deal with them in much the same fashion,
banning those that create deception.

Note Rule 7.5(d) prohibiting lawyers from using a firm name that gives the impression the
lawyers are a partnership when they are not.

This arises where common advertising strategy has lawyers sharing office; or secretary;
but who are actually separate entities.

If you advertise together, you may be held to be a firm for purposes of the conflict of
interest Rules.
Note 6 deals with class actions and part b of our problem.

Class counsel in class actions are governed by Rules 7.1-7.5, but are also governed by Rule
23(d) of the Federal Rules of Civil Procedure.

Generally, court cannot ban class counsel from soliciting, but can set rules to protect all.
Note 7 deals with Lawyer Referral Services.

Lawyers can solicit within Legal Service PlansYou can see how they are defined.

Lawyer can participate in such a plan even though it solicits liveas noted by Rule 7.3(d)
Finally Note 8do the advertising rules really have any meaning today? At least one author
concludes they do notand note that the ALI Restatement of the Law Governing Lawyers does
not cover advertising at allthe ALI allows general law to govern.

Section B. Nature, Establishment and Scope of the Relationship


A
Problem Question (a.) --Smith has called and indicated that he can't find another lawyer. Do you have
any obligation?
1
When do lawyer's ethical obligations begin?
1
Look at Scope Notes, Paragraph [17]; "Most of the duties flowing from the client-lawyer
relationship attach only after the client has requested the lawyer to render legal services and the
lawyer has agreed to do so.
1
In other words, the obligations under the rules begin when the attorney client relationship
begins.
Prospective clients may not become an actual client but MR 1.18 defines a "prospective client" in
subsection (a) and places confidentiality restrictions on attorneys

Also, under 1.18 (c ) says an attorney who met with a prospective client cannot represent
someone with "materially adverse" interests "in the same or a substantially related matter."

Restriction on this however is that if the attorney obtains potentially harmful info,
except as allowed under MR 1.18(d) and also provides other attorneys in the firm
cannot represent the client with adverse interests

a
a
a
a
a

Togstad v. Vesely, Otto, Miller & Keefe, is instructive on this issue (normally I ask students to play
the parts of litigant and lawyer in this casewhen the students act like a client, it is much easier to
understand how the client actually believed that she had an attorneyand vice versa).
Facts

Some 14 months following her husband suffering overly severe consequences of a medical
procedure, Mrs. Togstad was referred by a friend to attorney Miller for discussions about a
possible medical malpractice case.

Mrs. Togstad had no prior relationship--directly or through a family member--with Miller


or the firm.

Mrs. Togstad and Miller met for about 45 minutes in his office and discussed the
possibility of a medical malpractice action.

No fee was charged.

Miller kept no papers of Mrs. Togstad related to her husbands case.

Mrs. Togstad said that Miller indicated that he did not think that Mrs. Togstad had a case,
but that he would discuss the matter with his partners and get back to her if he changed
his mind.

When Mrs. Togstad did not hear, she assumed there was no case and did nothing further.

She was later advised there might have been a case.

By this time, statute of limitations had run on original action.

She filed legal malpractice action against the law firm.

Because this is a Malpractice--rather than Disciplinary--action, Privity is required--thus, as


indicated, there must be an attorney-client relationship between Miller and Mrs. Togstad for
the action to proceed.
The Court finds that when an attorney is asked to render advice and does so in a situation in
which it is reasonable for the other person to rely on that advice, there is created either a duty,
for tort law purposes, or a request coupled with reliance and acceptance, for contract law
purposes.
Look at Note 2 following Togstad, it repeats the teachings of the Rules, but then mentions the
Restatementwhich essentially adopts the Togstad holding.
In looking at whether the facts create an attorney-client relationship, facts must be viewed from client's
perspective.
Who decides these facts? Note 3 following Togstad, the "trier of fact" trier of fact at trial is the
jury.
Client likely to think attorney-client relationship when money is paid.
In Togstad,

When attorney indicated he did not think Mrs. T. had a case, but would get back to her if
his partner thought differently.

And he did not get back to her.

Reasonable client will assume that if lawyer does not think there is a case, there is no case.

Client will not then go to another lawyer.

In any such cases, lawyer must "break" the attorney-client relationship or limit it in scope--this is
Note 4 following Togstad.

Write person who might reasonably think she is a client.

Indicate no attorney-client relationship.

Indicate that major events, like statute of limitations, could affect the persons right to sue
if no action is taken.

Advise to get another lawyer.

Indicate whether anything more will be done for person, and the limitations of what will
be done.
NOTE: whether an attorney-client relationship existed, it can be a question of fact. If a jury is the
trier of fact, the lawyer is less likely to be sided with!

1
1
1
1

Would it apply to internet advice? Like our question (b) in our problem? Note 5 following
Togstad;.

If lawyer receives unsolicited email and does virtually nothing in response, there is clearly no
attorney-client relationship.
Note in DC the creation of attorney-client relationships through internet chat rooms (how would
this comport with Rule 7.3(a)? Perhaps DC had not yet considered that rule) and also note that if
lawyer manifests an interest in receiving inquiries over the internet, as in our problem, even the
Arizona State Bar might differ in its earlier opinion.

These rules apply regardless of whether client is looking for trial lawyer or is seeking advice
from office practitioner.
Thus would apply to a lawyer/investment analyst if the client came looking for legal advice.
Would apply to attorney for lending bank if the borrower innocently asks that lawyer for advice
and receives it.
Would apply to advice on joining a business venture by a lawyer/entrepreneur

Please consider Note 1 following Togstad; Under Rule 1.18, a lawyer owes duties to a
prospective client, including the duty to keep the clients information confidential and a duty
not to represent conflicting clients.

Question (c.) In our Problem--Assume no representation of Smith, but Judge calls you and asks you to
represent Smith in the civil action, for no fee. What are your obligations?
1
Rule 6.1
1
There are two versions at this point. The original version, set out in notes to 2001 version simply
makes lawyers aware of their obligation--the currently adopted version encourages lawyers to
render at least 50 hours of service per year. The 2002 version adds a sentence at the beginning of
the 2001 version making it clear that it is the Professional Responsibility of lawyers to render
pro bono services.
1
In considering the answer to our problem, students should assume that any type of informal
process for resolution of a landlord/tenant dispute of the type Smith has is either unavailable or
will not work.
1
This then means that Smith will likely have to go to court to obtain the relief he needs, and to
which Albion concludes Smith is entitled.
1
If a lawyer is involved on Smith's behalf.
a
The system of justice benefits from having that lawyer articulate Smith's position, rather than
having Smith cause turmoil with his largely irrelevant opinions.
a
Smith will benefit from having his valid legal claim presented in the best light.

1
a
a

Does this mean that a lawyer is obligated to take the case? See Note 6 following Togstad.
Model Rule 6.1 is more encompassing than were former ethical considerations EC 2-25 and 2-26.
All of these rules encourage lawyers to accept responsibility for representation of those unable to
afford legal services.
a
Making legal services available to all is an obligation that all lawyers undertake.
a
How that obligation is met is largely left to the discretion of the individual lawyer.
a
Rules only say this indirectlyRestatement, as the Note indicates, does a better job of saying this
directly.

Should a lawyer be allowed to discriminate in accepting clients?

California adopted a new provision of its rules prohibiting discrimination.

One author argues that discrimination against persons with disabilities is


impermissible apparently because the law office is a place of public accommodation
under the Americans with Disabilities Act; and a Massachusetts lawyer was found
liable for impermissible discrimination after refusing to represent men in divorce
proceedings.

At the moment, these are not the law in most states and the ABA has yet to give
an opinion on the issue.
a
If the lawyer decides to accept the case, the lawyer can look to the provisions of Model Rule
1.2(b) which make it clear that the lawyer's decision to represent a client is not an endorsement of
the client's views on any topic.
a
What if the lawyer cannot stand the clientsuppose the client is even more offensive than Smith
seems to be:

Rule 1.7(b) helps in that it recognizes as a conflict of interest personal interests of the
attorney that would materially limit the representation.

Thus, if the clients personality or views were so repulsive that the lawyer knew
that the lawyers independent professional judgment would be materially limited
by the lawyers own personality or views, then the lawyer should declineor
terminatethe representation.

This is probably the real meaning of Rule 6.2(c) dealing with acceptance of
appointments.

Rule 6.2(a) and (b) reiterate our current discussion; but

Rule 6.2(c) seems to permit the lawyer to refuse when the clients cause is so repugnant
as to impair the client-lawyer relationship;

This is clearly a conflict of interest analysis and requires the lawyer to balance.

The need to provide legal services to all against.

The extent of impairment of the relationship caused by the lawyers beliefs in the
clients repugnance.
1
The bottom line, however, is that, in response to question (c) of our problem, in the absence of
some judicial appointment, the lawyer need not take Smith's case.
1
Today, many states continue to debate the need for mandatory public serviceyou can see that
debate in Note 10 following Togstad.
1
Most would probably not require representation of Smith, but would give the lawyer options as
to how to meet any imposed commitment.
Suppose you read question (c) and (e) of the problem as the court seeking to compel the lawyer to
represent; Then, some different considerations apply.
1
On one hand, the lawyer's obligation does not require an acceptance of each case that walks in
the door.
1
When requested or appointed by the court, however, it is more difficult to refuse.
1
What do the Notes say: In Criminal Cases, (Problem, question (e))Note 8 following Togstad.
a
Because courts have obligation to fulfill a criminal defendants 6th Amendment right to counsel.
a
And because the courts have the power to regulate the practice of law.

a
a

1
a
a
a

As the Restatement indicates, an attorney client relationship is created when the court appoints
an attorney to represent a criminal defendant.
Such appointment is not a taking of the attorneys services, because the practice of law is a
privilege, not a right; additionally, attorneys gave up some of their rights when they elected to
practice law.
While courts can compel lawyers to represent criminal defendants without fee, or to serve as
standby counsel to criminal defendants who have elected to proceed pro se, the courts cannot
compel the attorney to spend the lawyers own money.
Civil cases are more difficultNote 9 following Togstad.
Look at the reference to In re Amendment to Fla. Rules.
Court holds that it can compel, through its inherent power, lawyers to represent indigents in civil
cases.
There is no 13th Amendment problem if the lawyer is not threatened with jail for refusing.

Court says that discipline is not jail.

Some question as to whether this would withstand federal Constitutional challenge.


Federal cases such as U.S. Supreme Court decision in Mallard v. United States District Court, also
mentioned in the note do not change this result.

Mallard dealt with discretionary appointment of lawyers under in forma pauperis


statutes.

The statute gave the court discretion to appoint and the court held that the court could not
"compel" lawyer service under the statute.

Mallard expressly left open the possibility of the court's use of inherent power to compel
representation.
The cases in this line hold that there is no 5th Amendment "taking" of the lawyer's services
because lawyers gave up a portion of their "services" when they became lawyers.

But note the disagreement that exists in civil cases.

Some courts hold compelled representation in civil cases does amount either to a taking or could
amount to a taking depending on the factual circumstances.
Certainly, what courts are trying to do here is reconcile the attorneys right to make money with
the clients right to have counselthe last part of Note 9 gives the balancing that at least one court
does.

First, look at whether the indigent has made an effort to obtain counselincluding
presumably an attorney who would take the case on a contingent fee.

Second, the court can then attempt to enlist private counsel; the refusal to order private
counsel to assist can only be seen as an abuse of discretion if a balancing of the five factors
mentioned in the note weigh in favor of such an appointment.

Question (d) of the problem raises the issue of limited representation; you are attempting to represent
Smith for a limited purpose. This is Note 11 following Togstad, along with Rule 1.2(c).
1
Rule 1.2(c)--what is required to limit the scope of the relationship?
1
This, in part, is Lerner v. Laufer, Note 11, the New Jersey case.
a
Facts.

Following mediation of their divorce dispute, and a written agreement as to what to do


with marital property, the mediator recommended that wife consult a lawyer about the
agreement.

When the lawyer saw this agreement, dont the facts seem to indicate that the lawyer did
not like the agreement?

But what do you suppose the conversation was between the wife and the lawyer (Get that
wife wanted lawyer to just skim over the agreement and really did not want the lawyer to
do a full service review)?

Wife later found out the agreement was not any good and sued

Please note that whether there is a limited representation is a question of factbut even with that,
the court found this agreement to be limited.
1
Wouldnt Wasserstrom like this result?
a
The client is getting to make decisions, even bad ones.
a
The lawyer is not getting to exert the lawyers extraordinary knowledge over the client.
a
Note this is a factual question and you can see that appellate court is not likely to overturn trial
court finding.
Note 12 following Togstad asks if law students should be required to do pro bono legal work, as an
introduction to the issues in Rule 6.1? What do you thinkin light of the fact that some schools already
require it?

Section B(2)--Identifying the Client


A
Once it is established that an attorney-client relationship can arise by implication; and once it is agreed
that the obligations of the lawyer run to the client, it becomes important to understand exactly who is
the lawyer's client.
A
In most situations, the answer is easy--the person who hired the lawyer; but in other situations, the
answer may not be quite so clear.
A
As we saw in the last section, an attorney client relationship can begin by a judge appointing the lawyer
to represent the client.
A
What about in the context of entity representationthat is, where the lawyer represents a corporation
obviously, corporations only act through peopleso who is the client?
1
The corporation only?
1
The president who hired the lawyer?
1
The board of directors who approved the lawyers hiring?
1
All of the above?
1
Some of the above?
1
None of the above?
A

Problem, --Athletic conference is represented by Page and LaRussa law firm. Two member schools are
represented by Young and Winslow firm. When an athlete sues the conference over eligibility, Young
and Winslow represent the plaintiff; the issue then is whether representation of two of the five schools
in the conference suggests representation of the defendant conference so that Young and Winslow
would be appearing on opposite sides of the same lawsuit. That result, students should presume at this
time would be an impermissible conflict of interest under Rule 1.7.
1
a
a
a
a

Greate Bay Hotel v. City of Atlantic City helps


Sands and Boardwalk Regency Casinos in Atlantic City want city to condemn certain property so
as to make a public walkway.
The property to be condemned is property in which Square Brighton has an interest.
In the litigation, Clapp & Eisenberg will represent Square Brighton.
C&E also represent five business trusts that operate super casino games.

Both Sands and Regency are members of the trust

If, in representing the trusts, C&E also represents Sands and Regency, then C&E
represents a client (Square Brighton) in an action against another client (Sands and
Regency); and this is prohibited by the conflict of interest rules.
Rule reference is Model Rule 1.13 (and there have been some changes in Rule 1.13 as a result of
ABA action in Summer, 2003well cover those when we get to Confidentiality and Conflict of
Interest)--lawyer who represents entity represents entity, not members of it.

It is not impermissible to represent members of the entity.

But mere representation of entity does not imply representation of members.


Issue then in Greate Bay is whether these trusts are separate entities, separate from their
members so that the conflict of interest provisions of Rule 1.7 are not offended;.

a
a

Here, court finds the entities separate.


As a result, there is no conflict for the law firm to represent the two entities at the same time.

1
a
a

In our problem, what does that tell us?


It may answer the court's question.
The facts of the problem are very similar to those in Greate Bay; in the problem, the plaintiff's
lawyer represents two member entities of the defendant.
If the member entities are considered defendants in the action, then plaintiff's lawyers would also
represent defendants.
What kind of a question is the determination of who the entity lawyer represents?

a
1
a

Note 1 following Great Bay Determination of whether partners in a limited partnership were
represented by the partnerships lawyer is a factual one that can be answered by an
examination of all the facts surrounding the relationship between the law firm and the
partnership members.

In our problem, then, will have to look at the relationship between the law firm and the member
schools and the member schools and the conference.
As Note 2 following Great Bay tells you, the same issues can arise even before the entity is
createdthus where individuals went to lawyer and asked lawyer to form a corporation, and only
represent the corporation, the lawyer did not represent the individualsUnder Rule 4.3, however,
the lawyer needed to advise the non-represented individuals they were not being represented.
A related issue:
There are situations in which individual members of a corporation will have their
communications with corporate counsel protected by the corporations attorney-client privilege-but that is different than our discussion here.
That issue is covered in next chapter.
Corporations are not the only examples of the issues discussed in Great Bay; another example is
that of insurance defense.
When insured buys liability insurance policy, policy provides that insurance company will
provide defense.
Assume insured does something that results in liability suit against insured; Insurer hires a
lawyer and directs the lawyer to represent the insured under the policy.
Who is the lawyers client?

The insured?

The insurer?

Both
As Note 4 following Great Bay tells you, the answer to this question can implicate conflict of
interest issues under Rules like Rule 1.7
Other examples:
Guardian and ward.
Other principal/agent relationships where agent hires lawyer for principle.
Parent hires lawyer for child.
In dealing with these examples, please also note the provisions of Rule 5.4(c)
Under that rule, the lawyer is not permitted to allow someone who is paying the lawyer to
influence the lawyers independent professional judgment on behalf of the client.

Thus, it seems clear that the way to identify the client in a difficult situation is to
determine whose interests the lawyer is advancing.

Regardless of who is paying the bill.

1
a

a
1
a
a
a

a
1
a
a
a
a
a

Section C--Authority within the Attorney-Client Relationship


A
Problem.

What legal theories allow an adversary to seek to enforce a settlement approved by the lawyer
who thought there was authority, but rejected by the client? Has lawyer done anything to violate
Rules by taking certain procedural actions in case?
New England Educ. Training Servs. v. Silver Street, helps answer the two questions posed.
Facts; Attorney had authority to settle for up to $10,000, but apparently settled the case for more-client does not want to pay third party and third party is seeking to enforce the settlement.
Issue is whether 3rd party can rely on the lawyer's statements to bind the client to a settlement to
which client did not consent.
The Restatement of Agency sees 2 classes of agency, with one of them having 2 subclasses.

1
1
1
1
a

Actual authority is discussed in Note 1 following Silver Street and is that possessed by the
agent as a result of "the principals manifestation of consent [to the agent];" generally
speaking, that which is part of the principal-agent agreement between them.

Subclass 1--When the principal prepares specific instructions for the agent, the
authority is called Express Actual Authority.

Subclass 2--When the authority arises from the assigned task, but is not specifically
part of it, the principal assume that agent will figure out what needs to be done as a
result; this authority is called Implied Actual Authority.

Implied because it is not stated.

Actual because it arises from the necessary implications of the tasks assigned to
the agent.

Apparent Authority is the second major class of agency and is discussed in Note 2 following
Silver Street and arises without regard to the actual authority given the agent.

It arises as a result of manifestations of consent given by the principal to third parties with
whom the agent deals.

It may thus arise without regard to the fact that the agent possesses no actual authority.
In Silver Street Partnership, 3rd party first raises the possibility there is "implied actual
authority," defined as actual authority that is "circumstantially proven from the facts and
circumstances attending the transaction in question;"

It can be proved from words used,

From customs

From the relations of the parties.

But there must be proof; thus, merely because attorney had authority to settle for a
different figure does not create an inference that attorney has authority to settle for any
figure

For what would there be implied authority upon which third party could rely?

Authority to negotiate

But not to settle--why the difference?

Consider in light of Rule 1.2(a)and this is discussed in Note 3 following


Silver Street.

Lawyer must abide by client's decision concerning the objectives of the


representation.

As settlement is an objective, the lawyer must respect the client's decisions.


Thus, lawyer can do only if the lawyer has authority; this is clearly
Wasserstromor his disciplestalking.

Rule 1.2 does give rise to some implied actual authority

Normally, actions concerning the means by which the objectives are


accomplished can be taken by the lawyer alone, so long as the client is
ultimately consulted

Generally no implied authority to settle arising merely by the relationship of


attorney and client.
Second way in which there could have been authority in Silver Street Partnership could be
apparent authority.

apparent authority does not depend on the agency agreement between lawyer and client
(principal and agent)

Instead, it relies on manifestations between client and third party.

If the client does something that makes the third party think that the lawyer has the
authority, then the client is bound.

Here, consider Navajo Tribe of Indians, Note 2 following Silver Street.

Settlement between Indian Tribe and General Motors and car dealership over cars.

Facts.

There were extensive discussions between attorney for tribe and GM/car dealer.

Settlement was reached by attorney.

There was a court hearing at which a spokesperson for the tribe testified that the
settlement was o.k..

At that point, the tribe, through its representatives, had told both the court and the
other party that they could rely on whatever the attorney had done in settling the
matter.

The situation was different in Silver Street Partnership.

Merely hiring an attorney is not enough.

Merely allowing attorney to appear is not enough.

As a result, apparent authority will be difficult to show in most lawyer-client


settings because Rule 4.2 forbids the adversary lawyer from communicating
directly with the client in the absence of specific state law or court order.

Thus, all negotiations would have to go through lawyer.

In the absence of some outward manifestation of authority to the adversary,


however, there is no apparent authority to settle.

As a matter of Evidence Law, how will this be handled? This is the tenor of Note 4 following
Silver Street.

If authority to settle is not implied, then it must be proved.

Party that needs the authority must prove it.

Thus, defendant would be required to prove that plaintiff's lawyer had authority from
plaintiff to settle.

This would seem to be a difficult task.

The court in Silver Street Partnership indicates that the decision may make settlement more
restrictive, BUT the decision does not discourage settlement.

When coupled with the method of proof, however, the Silver Street result impedes
settlement because it places on anyone who deals with an attorney the burden of
determining the attorney's authority.

Some states, Missouri, for example, use a slightly different approach to this
problem and, while it does not conform to the rigid structure of agency law, does
make some procedural sense.

Missouri attempts to resolve the difficulty of having the defendant in our problem
prove the authority of the plaintiff's attorney as follows.

Upon the attorneys actual or implied representation of authority to settle,


the courts will presume authority.

As a result, a prima facie case exists allowing opposite side to make a


submissible case on authority.

The party whose attorney's authority is questioned can, however, prove no


authority, however, that proof is by way of affirmative defense.

As an example, see Barton v. Snellson, 735 S.W.2d 160 (Mo. App., E.D. 1987). For the
type of proof needed to oppose the prima facie showing, see Pettus v. Missouri Ins.
Guar. Ass'n., 731 S.W.2d 527 (Mo. App., E.D. 1987).
Authority under the Rules of Professional Conduct.

TYPES OF AUTHORITY
Express: authority actually given by the client to the attorney
Implied: authority implied from the circumstances

Retention of an attorney to represent one's interest in a dispute, with instructions to conduct

settlement negotiations, without more, does not confer implied authority to reach an agreement
binding on the client."
Apparent: conduct of the principal that reasonably leads the third party to rely on the agent's
authority.

Authorizing an attorney to conduct negotiations "could not reasonably be relied on as the basis for
apparent authority to settle the case."
*never make a settlement commitment without your client's authority to do so. If there
are limitations on your authority, make them know to the opposing counsel.

Rule 1.2(a)--Client controls objectives of representation

Settlement
In criminal cases.

Whether to plead
Waiver of jury
Whether the client will testify.

Alteration of terms of contract


It would thus be impermissible for lawyer to do something in these areas without
the authority of the client.

Rule 1.2(a) generally allows the lawyer, with consultation with the client, to control the
means whereby the objectives of the representation are accomplished.

This is because of lawyer's superior knowledge of the procedural aspects of the


representation.

Authority over these areas is "implied" from the hiring of the lawyer; such matters
as:

Whether to cross examine a particular witness.


How to plead a specific defense.
The caveat would be that the lawyer's actions cannot compromise or negate

the client's claim or cause of action; if the means eliminate the objectives, the
means are within the clients control.
Thus, in our Problem,

Question (a): If in a Silver Street Partnership jurisdiction (which is the majority), there
would be no settlement because the defendant could not prove authority.

Question (b): Under Rule 1.2,

Decisions on means for accomplishing objectives of representation can be made by


attorney, after consultation with client.

Two final thoughts.


1
First, Note 6 following Silver Street.
1
Rule 1.14client with a disability.
1
The lawyer does not assume more authority simply because the client is under a disability
(whether caused by age, infirmity, mental or psychological condition).
1
As Rule 1.14(a) makes clear, the lawyer is to maintain as normal an attorney client relationship as
possible with one who is under a disability.
1
It is only when the client is unable to make adequate decisions that the lawyer may breach
confidentiality and seek appointment of a guardian.
1
If the client is under so much of a disability as to need a guardian, does the client really have the
mental capacity to appoint an agent in the first instance? That is the point of the Devine article in
Note 6.
1
a
a
a
a
a
a

Decision on means should not, however, be allowed to destroy the client's lawful
cause of action.
Short of that, however, tactical decisions, like in this problem, are generally said to
be within the implied actual authority of the lawyer.

Second, Note 5 following Silver Street; A test of authority.


The court found the lawyer had apparent authority to compromise the clients rights.
Apparent authority will only exist when there is no actual authority.
Apparent authority exists without regard to what is actually in the contract between the parties.
Actual authority is that contained in the attorney-client contract.
In this case, regardless of what the court found on the issue of apparent authority, the contract
between attorney and client did not authorize the attorney to take the action taken.
As a result, a decision by the court that there was enough to create a presumption of authority
vis--vis the other party was not a finding that the attorney had actual authority under the
contract.
Because the concepts of actual and apparent authority are different and depend on different
facts, there was no determination of the actual authority issue and preclusion did not apply.

Section D. The Trust Account


A
Missouri Rule 1.15 is similar to the Model Rule. It requires lawyers to keep money in an interest bearing
account and allows lawyers to opt out of the interest bearing account provision (but not out of the trust
obligation); you need to look at this rule.
A
The real obligation of Rule 1.15 is in paragraph (d):
A
Lawyer is required to:
1
Notify client or third party of receipt of money in which client or third party has an interest.
1
Promptly deliver to client or third party money or funds to which they are entitled.
a
Except where otherwise required by rule or law

Here, look at part (e) of rule

If there is a dispute, the funds must be kept separate from funds of others until
an accounting.

If there is a dispute only over a portion, that portion must be kept separate.
a
Or by agreement with client.
1
If requested, provide prompt accounting
A
These obligations are not substantially different from the obligations of a regular trustee.
1
What is different is where these funds are kept
1
That is part (a) of Rule 1.15.
a
Funds in which client or third persons have an interest must be kept

Separate from the lawyer's own funds; BUT

Can generally be kept in a single bank account.

How does this create problems for the lawyer serving as trustee for client funds?

Generally, a trustee is not permitted to commingle trust funds--that is, to put one
set of trust funds with another.

Lawyers have traditionally been permitted to commingle client funds because the
assumption has been that the lawyer will not hold the funds for very long.

As a result, the money will not earn enough interest to be credited to any one
trust.
1

a
a

A
A

None of this diminishes the lawyers trustee obligations, which are set out nicely in Johnson
article, Trust and Business Accounting for Attorneys;.

The basic obligation is to keep track of the money.


That is key concept #1 in the Keep separate ledger for each client.

By keeping separate ledger sheets for each client, you keep track of the money for each
client in an otherwise commingled account.

You could always put each clients money into a separate account, but the time required to
upkeep each of those accounts would be enormous.
a
Key concept #2, same article.

Lawyer trust accounting is not an accounting method.

Key concept #3, It is a record keeping method always keep accurate records! Page 150
casebook

As a result, look at Key concept #6, same article--you don't allow negative balances; all
trust account balances must be positive numbers.

If the client owes you moneyor owes money for some sort of feeyou will keep track of
that elsewherein some sort of office account, not as a negative balance in the trust
account.
a
Key concept #4 may be the most importantit tells you exactly what pieces of paper you need to
keep to maintain an audit trail for each entry.

Pg 149-150
1
In re Wilson.
a
People simply assume their lawyer will handle the funds.
a
Usually, it is a matter of expedience or custom, but it is also usually essential.
a
Clients and third parties are willing to do this because of their faith in the legal system and the
bar; as the court writes: "No other explanation can account for clients' customary willingness to
entrust their funds to relative strangers because they are lawyers."
a
Sanctions for trust violations tend to be severe, as can be seen from the ABA recommendations,
Note 1 following Wilson.
a
Here, discuss any different or supplemental local obligation.
Now look at the Problem at the beginning of the section.
Question (a)--Why should you deposit the $1,500 into your Trust Account, not your office account.
1
It appears to be a "retainer," however it is not.
1
A retainer is an amount of money used to secure availability over a given period of time.
1
This is an "advance fee payment; what is the difference?

What happens if the client fires you tomorrow?

You would be obligated to give part of the money back.


1
If that is true, it is money in which the client has an interest and therefore must be deposited into
the trust account.
1
Here, look specifically at Rule 1.15(c)which requires that monies paid in advance and from
which the fee will be withdrawn as earned must be deposited in the trust account.
1
So when do you have a real retainer;

Only when the amount paid to you need never be repaid and is not contingent on doing
anything.
a
A true retainer will almost always say something likeyou will pay me $5,000 as a retainer for
which I will be available to you on an as needed basis. In the event you need my services, you
will pay me at the rate of $100 per hour for all work done.
a
A true retainer that need not be deposited into trust does no more than secure the availability of
the lawyer over the given time and assure that the lawyer will not accept any conflicting cases.

At which point, those of you with a contract fixation ought to be sayingthere is no


consideration for such an agreementbecause the lawyer is already under an obligation
under the Rules of Professional Conduct to avoid conflicting interests, the lawyer is not
giving up anything to get the $5,000;

And a few states and courts have agreed with you, banning the use of such a retainer.
a
Occasionally, you read or hear about the nonrefundable retainer;

As my spell checker indicated when I typed the word, there is no such word.

If a payment is a payment made in advance from which the lawyer will draw the fee as
service is provided, it is an advance fee payment and must be deposited in trust.

If a payment is designed to secure the lawyers availability over time, and individual
services are separately billed, the payment is a retainer; all true retainers are
nonrefundable.
a
In planning this edition of the book, we could not decide where to put Notes 2 and 3 following
ABA Formal Opinion 93-379 in the next section on Legal Fees; I assign those two notes here
because they both cover the material I have just covered, and make the same essential points; If
you wish, you can save these notes until you get to legal fees and just gloss over the concepts of
advance fee agreements and retainers for now.
Question (b); You pay to obtain reports on behalf of MacMillan.
1
Provided the clients check has cleared the banking system, this money can come out of the trust
account, because you have an advance fee payment of $1,500 to cover it.
1
The documents you keep are listed in the Johnson article, key concept #4: "The Audit Trail;"
1
Suppose you did not have an advance fee payment or it had all been usedcan you still pay for
additional documents?
a
Rule 1.8(e)lawyer is permitted to advance the costs of litigation.
a
Once the lawyer has done so, however, notation of that fact does not go into the trust account,
because that would create a negative balance; this is key concept #6; you will keep record of those
amounts someplace else.
Question (c); what are your options in dealing with a check for $150,000 made payable to you and your
client?
1
Sign your name and turn the entire check over to MacMillan--This may, of course, impair your
ability to get paid.
1
Deposit the check into the Trust Account.
a
To do that might require you to sign MacMillan's name.
a
Does your authority as "attorney" authorize you to sign the client's name to checks?

That would seem to be an objective within the meaning of Rule 1.2so the answer would
seem to be no;

Even if it could be a means, it would only be procedural if the money were deposited into
trust--because then it would keep its character as MacMillan's money.
1
What is absolutely clear is that it is money in which MacMillan has an interesttherefore it must
be deposited into the trust account under Rule 1.15(a) and (d).
Question (d)You paid out on a settlement check when it was deposited.
1
Can you pay out on any check when deposited?
1
No, it is just a checkan order to payit is not money.
1
Having done so, you are now out of trust.

When you paid out upon deposit of the check, you violated key concept #2,--you spent what you
did not have.
a
As a result of improperly paying out on the check that had not yet cleared, you are out of trust.
a
You may well be disciplinedalthough probably not disbarredlook at the materials on
"enforcement of the trust account obligation," Note 1 following Wilson.
a
Most states authorize audits of trust accounts when it is shown that the account is out of balance.

Here, yours is clearly out of balance and you have some explaining to do.

These type of provisions have been upheld against challenges involving an invasion of the
attorney-client relationship and the constitutional rights of both the lawyer and client.
a
What should you have done?

Should have waited until the check cleared?

Is MacMillan going to understand this delay?you got the money to settle his case and
now you wont pay outMacMillan will want to know why.

This issue really needs to be handled in the retainer agreement you have with MacMillan
or any clientas we will discuss in upcoming sections.
Question (e)--This is In re Grubb, section d(4).
1
Facts.
1
Attorney received ring from client.
1
Kept it in a drawer--took it outthen took it homethen lost it.
1
The lawyers trustee obligation is the same for property as it is for money--must keep the
property safe.
a
Safety deposit box
a
Moving and storage company
Question (f)
1
Why can't this money be deposited directly into the office account?
1
The only reason would be if there was a question about your fee.
a
Your agreement was $150 per hour and you have spent two hours working for the client
a
Plus $75 court cost and $125 in expenses.
a
You're already entitled to $500 when she brings the check.
a
But, if there is a question of the amount of your fee, the contested amount must remain in trust.

Under Rule 1.15(e), you must keep the amount of any dispute in trust.

As the rule indicates, amounts that are not in dispute must be promptly paid to the
rightful owner.
Question (g)
1
Interest on Lawyer's Trust Account (IOLTA).
1
It is normally best to consider IOLTA in context of local rule.
1
But, most rules either require or permit an attorney to deposit certain funds into an interest
bearing trust account.

Those funds which can be so deposited are generally:

Money that is too small in amount that the interest the money would earn would
not offset normal administrative expenses, OR.

Money (even a large amount) that will be held for too short a period of time to
earn enough interest to have the interest offset the administrative expense

When the lawyer is permitted to opt into an IOLTA, the lawyer is normally required to opt
in for all or none of the clients.

This to prevent an individual client from controlling the attorney's decision-making.

If the client had a right to direct the attorney to deposit money in an interest
bearing account, the client would have the right to control the money.

And this would be in contrast to the notion that the amount is too small or will
be held for too short a period to have interest be greater than administrative
expense.

a
a
a

a
1
a

That interest goes to a private foundation which then pays it out to particular projects.
Why aren't such programs considered a "taking" of the value of the interest?
Because of Trust rules mentioned earlier.

Lawyer is trustee.

As a result, if the money the lawyer holds will be held for a long enough period of time to
earn interest; or if the money is large enough; and if the money that will be earned is more
than would be administration expenses for setting up a totally separate account, the
lawyer must set up such an account.

Normally, that is not the case; as a result, no client or third party would have actually
earned anything if their money was singly deposited into the account.

The reason that interest is earned is because so many funds are deposited into a
commingled account.

The only entity that is "losing" is the bank which does not have the money for free.
Are such programs worthwhileLook at Note following BrownIOLTA programs generate
roughly $200 million a year.
Brown v. Legal Foundation of Washington
Facts:

Washington rules require lawyers to put their trust funds into interest bearing account.

Note 2 following the caseABA attempts to require banks that accept trust accounts to
accept IOLTA accounts.

A public interest law firm and four citizens challenged the rule and the Supreme Court
seeks to address two issues:

Does taking the interest from small accounts amount to a taking by the State
under the 5th Amendment?

Is the client being paid just compensation?

Taking issuethe Court applies a per se test in finding that a taking took place.

Compensation issue.

Here the Court attempts to determine the value of the lossthe net lossto the
plaintiffsand therein lies the problem.

If money is so large or is going to remain in the trust account for so long a period
of time that it will generate a measurable amount of money, then the lawyeras a
trusteehas an obligation to earn money for the client.

In that circumstance, the lawyer would be obligated to place the client money
that which is going to earn a measurable amountinto a separate account so that it
earns money for that client.

By definition, then, the only monies that can go into the commingled trust
account, are client monies that are either too small in amount, or are going to be
held for too short a period to generate any measurable amount of money for the
client who deposited those monies.

Here, the fact that the total amount in the trust account might generate
substantial amountsor that around the countrythese small amounts will generate
over $200 million a year does not matter.

Because the only compensation due the individual who deposited it into the trust
account is the amount of the net loss to that personand that net lossby definition
is zero.

Neither Justice Scalia nor Justice Kennedy see the logic of this and we are not likely done
with the topic.

Section E. Client Liability--Legal Fees.


A
The Fee Agreement and the Amount; Introduction.

In most instances, the issue of legal fees is part of the agency contract that exists between attorney
and client.
1
Under Rule 1.5(b), it is the obligation of the attorney to communicate the basis of the fee to clients
the attorney has not regularly represented at or near the beginning of the relationship.
1
While compensation agreement is often said to be unrestrained at law, Rule 1.5(a) requires that
the agreement be reasonable.
1
The basis of a reasonable fee is set out in Rule 1.5(a)(1)--(8).
1
Reasonable fee is set at the outset and is determined by fairness to the client in light of these
factors.
1
A court is always an expert on determining both the reasonable fee and the reasonable value of
legal services.
Defining "Reasonable Fees; Problem.
1
Question (a); Fourchon Docks v. Milchem, is an appeal of the amount of attorney's fees permitted
by a court under an agreement between the parties.
1
This should certainly suggest a strong amount of judicial control over legal fees.
1
Generally, if a fee is not fair, it can be questioned by the client and a court should exercise its
equitable powers to review the fee, as asked.
1
Facts.

In the case, a sublease forbids further subleasing without written permission and
contains provisions for accelerating the lease and providing for attorney fees in the event
of breach.

Defendant breached and was sued.

Trial court found that plaintiff's attorney would get $216,000 under the terms of
the lease agreement; but reduced that to $57,750, an amount it considered to be the
"reasonable value of the legal services;"

How can court reduce?

Part of court's supervisory power.

This is why court makes reference to Rules of Professional Conduct.

In determining a "reasonable fee," court is guided by provisions of Model Rule


1.5, but not limited to them.

Thus, does not have to consider each one.

There is no "maximum" or "minimum;"

Note 1 following ABA Opinion 93-379 talks about the impact of subsequent
events on a reasonable fee.

Notes 2 and 3 following ABA Opinion 93-379 discuss retainers and advance fee
payments; my notes on those two concepts were put in this outline under trust accounting,
supra.

Also Note 5 following ABA Opinion 93-379--if there has been no agreement on
the fee, the measure of recovery is the fair value of the legal services.

In that regard, then, it is a restitutionary damage standard--it is the


court's definition of "reasonableness" with the court sitting as something of an
equity tribunal.

The same measure can also apply at other times, however, because court
could always say agreed-upon fee is not reasonable--thereby leaving the parties in a
contract-like relationship with no price term.

At which point reasonableness governs.

Is a writing necessary? This is Note 4 following ABA Opinion 93-379

Here look at Rule 1.5(b) indicating that when the lawyer has not
regularly represented the client, the fee should be discussed at the outsetand does
not requirebut prefers a writing.

But what if there is no writing.

When lawyer then starts to work for client, lawyer undertakes


obligations imposed by the rules of professional conduct.

Thus, lawyer owes unbridled fidelity to the client's interests.

As a result, any self-dealing by the lawyer--as in setting a legal


fee--becomes a conflict of interestThis is Note 7 following the Opinion.

Under such circumstances, the lawyer must then show not just
that the lawyer rendered service to the client, but also

That those services and any fee charged is "fair and reasonable"
to the client.

This second prong is needed because of the conflict that has


arisen.
1
Question (b.),--all fees calculated on hourly basis but, in addition
a
All time is rounded to nearest 15 minutes.
a
All telephone calls are billed $20 regardless of time.
a
All files surcharged $150 for computer research and $50 for photocopying.
1
And Question (c)--while traveling, Attorney Walters dictated letters and wrote a memo on
another case. Can attorney bill both the file that put him on the plane and the case upon which
the dictation and memo are done for the same time--thus billing two clients for the same time?
a
These issues are discussed in ABA Formal Opinion 93-379.
a
This is first time we have seen an ABA Formal Opinion.
a
It is not binding law in any state.
a
It is, however, the interpretation of a formal committee appointed by the ABA whose job it is to
render ethics opinions.
a
Therefore, it is often persuasive evidence of the way rules should be interpreted.
a
Question (b)(1)--rounding to the nearest 15 minutes--it is not "reasonable" to charge a client for
more time than is actually expended.
a
Question (b)(2)--charging flat rate for phone calls--same--and also the opinion speaks of
disbursements and surcharges--it would be impermissible to assess a surcharge over and above
the amount actually spent by the lawyer.
a
It might be permissible to charge in this fashion, but, as indicated, only if the client agreed and
only if that agreement came at the outset of the relationship, as discussed in Notes 4 and 5
following this Opinion.
a
Question (b)(3)--adding surcharge for computer research and photocopying.

What is the problem with doing this?

It is really just a different form of general overhead.

Would the lawyer charge, when calculating an hourly fee, the actual cost the
lawyer was paying the lawyer's secretary?

Presumably, this cost was built into the hourly rate.

But again, could be permissible, if client agrees at the outset.

Note permission to agree to a set amount.

But in absence of such a provision, actual cost only; and then only if
there is an agreement as to payment of costs.
1
It is because of charges like those in this problem that the current version of Rule 1.5(a) makes it
clear that not only does the legal fee need to be reasonableBUT all costs charged to the client
need to be reasonable as well.
Question (c)--charging multiple clients for the same time.
1
Again, Formal Opinion 93-379 answers these issues.
1
When a lawyer has enough clients to schedule multiple motions on the same day, and each client
is paying on an hourly basis, why shouldn't the lawyer charge each client for the hours worked?
a
Why should the client get the benefit of the lawyer's good planning?

The opinion does not answer that question, but seems to assume there is something inherently
bad in billing more hours than there are in a day.
a
Could you make a distinction between flying time (which client agreed to pay) and actually
working on something for another client (which that client agreed to pay) and charging as an
original for reused work product?

How would the public perceive that distinction?

Isn't public perception of lawyers a large part of the issue?


a
Also note provisions of Note 9 following ABA Opinion.

Many states have begun "fee arbitration" programs, whereby client can ask for
review of the legal fees by a panel of lawyers and non-lawyers in lieu of going to court
over the fee.

Some states require the lawyer to go to arbitration and this is the latest
recommendation of the ABA.

Mandatory fee arbitration is accomplished through use of the state supreme


court's inherent power to regulate the legal profession.

It has been upheld against Constitutional challenges.

Most states, however, use voluntary fee arbitration--client can request that a
dispute go to arbitration but lawyer can refuse and case is then taken to court for decision.
a
The reason that fee arbitration programs are encouraged relates to the implied teaching of Note
8; there is nothing impermissible about suing a client for legal fees--however, as Notes indicate, a
suit by a lawyer over fees often results in a counterclaim against the lawyer for malpractice.
Question (d)Delores wins a federal civil rights case in which court is authorized to award a legal fee;
Notes 10-11 following Formal Opinion 93-379.
a
The American Rule generally requires that litigants pay their own legal bills.
a
The American Rule is the reason contingent fee financing developed in America and has been
such a problem here; Without contingent fee financing, many injured workers during the early
1900'sthe muckraker periodwould not have had enough money to hire a lawyer; Under the
American Rule, they would not have been able to go to court.
a
Today, there are federal and state legislative provisions that ameliorate the American Rule and
allow a successful litigant to collect their legal fee from the other sideas Note indicates, over 100
federal statutes and over 200 such statutes in California alone.
a
The federal court, in determining the amount of a fee, often uses the lodestar amountand as
the cases seem to indicate, lodestar is really a reasonable fee using the factors in Rule 1.5(a)
as a guide.
a
Once that is understood, Note 11 is easier understood.
a
In Delaware Valley, the Court considered whether to increase the lodestar amount because of
the contingent nature of the casethat is, the more difficult the case to prove, the more a
plaintiffs lawyer should be rewarded.
a
The Court considered a number of issues, as discussed in the Note, but wouldnt an easier
answer have been found in Rule 1.5(a)(1)?
a
If the lodestar amount already considers the time required for completion of the work, the
novelty of the issue, the difficulty of the issue and the amount of skill required by counsel to
complete the job--are the lawyers in Delaware Valley asking that these same factors be counted
twicea second time to increase the amount of the reasonable value?
1
At this point, it seems logical that the Court could find that the difficulty of some cases could
cause an increase in the amount, but that the difficulty of this case did not.
1
Indeed, if another case were more difficult, the amount of the reasonable fee under Rule 1.5(a)(1)
should be higher.

Section E(1)(b) Types of Legal Fees

There is no problem for this section, but it is good for students to understand the different types of ways
in which clients can be charged for legal services.
A
Billable HoursObviously, the primary billing method is billable hoursbut the ABA Commission on
Billable Hours report looks at the disadvantages of this practice, including the heavy burden placed on
associates who have to meet those billing hour standards.
1
Billable hours also discourage pro bono work because there is no time to do it.
1
For the client, there is no advance certainty about the feeit can simply escalate as the hours
increase.
1
Billable hours do not encourage efficiency because they encourage hours.
A
On the positive side, using hourly billing is easy and works regardless of the type of case.
A
The report then discusses alternative methodsI encourage students to look at those alternative
methods and ask questions about themI also ask if they know what is being used in any of their firms
I do not do much more than that.
A
The Note on Billable Hours is well worth reading. Do students expect that a lawyer who billed 16.5
hours a day for 365 days is using some of the double billing methods condemned in the ABA Opinion?
E(2) Special Problems with Contingent Fees
A
I always make the starting point for contingent fees Rule 1.8(i) that forbids a lawyer from acquiring an
interest in the client's cause of action.
A
If this is the general rule, it is clear that contingent fee, under which the lawyer shares in the award to
the client, is an exception to general rule.
A
Contingent fees are allowed to permit persons who could not otherwise afford access to the courts to
gain access.
A
But such fees always place the lawyer in a conflict between obtaining fee and obtaining best result for
client.
1
Suppose that there is a proper contingent fee agreement and, at a time when the lawyer thought
the case was about ready to hit for big dollars, the client said: "I'm tired of this--it is not worth it-I want to stop the litigation;" What can lawyer do?
1
The attorney is agent not principal
1
Contingent fee is exception to rule preventing lawyer from acquiring an interest in the client's
cause of action.
1
As an exception, it is to be read narrowly
1
As a result of the above, it is clear that the lawyer cannot control the action--when client wants to
discontinue, the lawyer must discontinue.
A
Problem
1
Problem (a).(1)--Who decides if case is to be funded on contingent fee basis?
1
Note 2 following Rohon; 1986 ABA informal opinion indicates that contingent fee can only be
offered.
1
After client is offered regular fee and
1
Either can't afford it or does not want regular fee.
1
Unclear about where this opinion considers Comment [3] to Rule 1.5 indicating that whether to
charge a contingent fee requires the lawyer to consider all factors under the circumstances.
1
I read Comment [3] as requiring the lawyer to consider the factors in the informal opinion
thereby meaning that comment and the opinion agree.
1
Question (a)(2)--What terms or conditions are required under Model Rules for permissible
contingent fee?
a
As an exception, the rule permitting such fees should be restrictively read.
a
Rule 1.5(c) requires a writing signed by the client for a contingent fee to existsuppose you
send a letter to client and advise client that unless you hear back to the contrary, you will
charge a contingent fee? This is not authorized under Rule 1.5(c).
a
In fact, the rule requires 2 writings.

The initial fee agreement must be in writing and state.

The method for calculating the fee, including any percentages.

Whether expenses are to be deducted from recovery and for which


expenses client is responsible.

Whether such deduction comes before or after the fee is calculated.

If it comes before, lawyer shares in the expenses.

If legal fee is deducted first, then expenses removed, client


alone pays the expenses.

Before disbursing money, there must be another writing showing how the
proceeds are to be distributed.
Problem (a)(3)--"Reasonableness" and contingent fees
a
Contingent fees can be reviewed under the reasonableness standard of Rule 1.5(a) just as any
other fee.
a
If it is unreasonable to get this fee for the amount of work done, a court can review it.
a
Review comes upon the client asking the court for review.
a
This is the point of Rohon v. Rosenblatt
a
Facts.

Plaintiffs wife applied for a life insurance policy.

After application, wife diagnosed with cancer.

Policy issuedthen wife died.

Lawyer told husband that suit would be necessary to collect on the life insurance
policyand that lawsuit would be horrendous; it is unclear why the lawyer thought the
lawsuit would be needed or why it would be difficult.

Insurance company paid policy plus interest.

Lawyer collected 1/3 of $100,000, even though no suit ever filed.

Plaintiff now sues to get legal fee back.


a
Is this case saying that there can NEVER be a situation in which a lawyer takes a case on a
contingent fee basis, the case turns out easier than both parties thought and the lawyer gets a
large fee? NO
a
Is this case saying that a contingent fee can NEVER result in a large amount for a lawyer for a
relatively small amount of work? NO
a
This case does help us understand the relationship between lawyer and client and contingent
fees.

Even though attorney is the agent, the attorney is a fiduciary to the client.

As a result, the attorney has an obligation of proving fair dealing with the client
why? Because of Wasserstroma distressed client may yield too easily to the demands of
the lawyer, who knows more about the system.

When is a contingent fee justified?

When there is an actual risk that the client will get nothing.

In this case, no reason for insurance company not to paythere was no


risk.

Does this mean that you cannot take a contingent fee in an accident case
in which it is absolutely clear that your client will win, but the only amount is
damages? NO

In that case, the amount of damages is at risk and that is the basis for
the fee.
a
Here the court finds the contingent fee unreasonable.

Does court then award the lawyer the reasonable value of the lawyers services?

No, the court awards the lawyer nothing.

It is clear that the court is penalizing the lawyer for the conduct.

A court is always an expert in the amount of legal fees and here determines the
lawyer should receive none based on combination of work and bad faith dealing with the
client.

Problem (a)(4)
a
Contingent fee in structured settlement; Note 3 following Rohon.
a
Here, the Restatement helps.
a
If there is an advance fee agreement about how to divide the fee in a structured settlement, the
Restatement appears to suggest that it should be followedBUT WHAT?

The structured settlement agreement can always be reviewed for reasonableness.

If lawyer is getting all the money and client is getting virtually none, the fee is
likely be struck down as unreasonable.
a
If there is no advance agreement, the lawyer gets paid the lawyers share of each payment
received by the client.
a
If lawyer gets all the fee at the time of the settlement, then fee is based on present value of the
settlement.
Look now at Note 5 following Rohonyou get a large check from the insurance company and you let the
client know about it; You put the check into your trust accountcan you immediately pay out on that
check?
a
No, of course not, you do not yet have the money.
a
Probably need to explain this to the client.
Now look at Note 6 following Rohon, the counterclaim issue; client is awarded $100,000 and you have a
30% contingent feethe problem is that the defendant is awarded $50,000 on a counterclaimdo you get
30% of $100,000 or 30% of $50,000?
a
This problem is compounded by fact that the client will have another lawyer on the
counterclaima lawyer from the clients insurance company.
a
As a result, whatever the adverse party gets on the counterclaim is not lawyers fault; the
lawyer representing client on the $100,000 claim will have little to say about the presentation of
the counterclaimbecause the lawyer hired by the clients insurance company will control
presentation of that claim.
a
And the lawyer on the counterclaim will be getting paid from the insurance company.
Thats why Note 7 following Rohon is present, it sets out some of the considerations you might want in
a retainer agreement with your client; you are asked what additional provisions you want?
a
The immediate one is that payment check/draft will be deposited into trust and will not be
immediately payable.
a
Then certainly a better provision on Structured SettlementI do not want to get my money with
the client on each payment.
a
What about appeal? Does the contingent fee cover trial only or trial and appeal?
a
What about the counterclaim issue?
Problem Question (b)--Note 9 following Rohon-a
Rule 1.5(d)(1) forbids contingent fee in matrimonial case, including one seeking distribution of
property or support.
a
All of these contingent fees are contingent upon the outcome of a matrimonial action and would
be prohibited under Rule 1.5.
Problem Question (c)--This is different from Question (b) in that the divorce action is over and the past
due amounts have been reduced to a judgment.
a
As Note 9 indicates, there is some authority for the idea that an action to collect only past due
amounts for support, which have been reduced to a judgment, are actions that will support a
contingent fee because they are really debt collection actions.
a
In such cases, the public policy in favor of preservation of marriage is gone.
Problem Question (d)This is Note 8 following Rohon.
a
Rule 1.5(d)(1) forbids a contingent fee in any case the outcome of which is tied to the result of
a criminal case.
a
This fee is contingent upon outcome of criminal case; even though structured in a civil action,
this fee is not permitted.
Two other Notes.

a
a

Note 10there is generally no prohibition against taking contingent fee cases in public policy
matters, other than the public policy preventing such a fee in matrimonial and criminal cases.
Second, Note 4 following Rohon is for those of you who want to know the income tax
consequences of contingent fee cases; The courts are split as to whether the fee of the lawyer is
part of the clients gross incomebut if the fee is to be a lien on the clients award, courts less
likely to find the fee part of the clients gross estate.

Section E(3) Division of Fees:


A
Problem Question (a)
A
Here, we have to review some history before we can go forward.
1
Please note distinction between prior D.R. 2-107 and current Model Rule 1.5(e)
1
Under D.R. 2-107, division of fees between lawyers not in same firm had to be made on the basis
of the percentage of the work done by each lawyer; Thus, in problem, question (a), under the
former Disciplinary Rules, lawyer would have to return the "referral fee" check.
a
Why do I mention this? Two reasons.
a
First, many lawyers for whom you work will not know about the provisions of Rule 1.5(e)
allowing fee splitting.
a
Second, the old rule did not workit was honored in the breach more than it was honored in
reality.
A

Under Model Rule 1.5(e), division of fees between lawyers not in same firm can be made if:
1
First, either.
1
The division is in proportion to the services rendered, OR
1
If all lawyers who participate in the fee assume joint responsibility for the representation;
AND
1
Second, the client is advised and agrees to the share of each lawyer (and here there is a change
from the 2001 Rules which only required that client did not object); AND
1
Third, the agreement is confirmed in writing, AND
1
Fourth, the total fee charged the client is reasonable.

What does Chambers v. Kay do for us?


1
Facts:
1
Chambers and Kay had separate law practices.
1
How do I know they are not partnerslook at Rule 7.5(d); these lawyers did not hold themselves
out as partnerstherefore they were not partners. *because remember, if they are partners, we dont
worry about the fee splitting...
1
Kay had a client (Weeks) in a sexual harassment case.
1
Kay sought Chambers help in the case and Chambers worked on the case and advanced costs in
the case.
1
There was an agreement between Kay and Chambers as to division of the feea copy of that
agreement was sent to the client, Weeks, who never consented to it.
1
There was a falling out between Kay and Chambers and Kay fired Chambers, allegedly with the
consent of the client.
1
Weeks won her liability case, Kay collected a fee and Chambers wants some.
1
What does the court do?
a
First, it looks to whether Kay and Chambers were partnersWhy?

If they were partners, they would not be governed by Rule 1.5(e).

If they were partners they would not need an agreement and Chambers could share.

The court finds they are not partnershow?

Looks at Rule 1.0(c) which defines firm and Rule 1.10 and determines that
whether a firm exists is a factual question.

The court reviews the facts and finds no firm;

Second, the court then looks at Californias version of Rule 1.5(e) and finds that like the ABA
rule, there can be no division without the clients consent.

Here, it is clear client was notified.

But no consent.

Court unwilling to consider the notice to the client to be consent.


a
Third, the court decides whether Chambers should be paid based on quantum meruitthat is, is
it unfair for Kay to keep the entire fee under these circumstances.

What does court say about that?

Cannot let Chambers skirt the rule by getting around it indirectly.


a
What, then, does Chambers get? Nothing.
1
Note 1 following the case makes all these pointsthe answers to all of the questions are yes;
Chambers does not get paid simply because Kay did not comply with Rule 1.5(e) in obtaining
client consent.
1
Daynard, the case in Note 2 following Chambers, takes the opposite position holding that the
lawyer in Kays position both breaches a contract and a fiduciary duty in breaching the
agreement to split the fee.
a
This, of course, sounds like the language of quantum meruit.
a
The court does not make that clear in its decision.
How does this help us with Question (b).
1
If the client is notified, and does consent, the Rules allow for the pure referral fee,
1
Provided the requirements of the Rule are met regarding joint responsibility.
1
Assuming that to be the case, then, there is nothing wrong with the agreement in this problem.
Look at a couple of the Notes:
1
Note 4 following Chamberswhere there is a valid fee splitting agreement, it can be overridden
by the fact that the representation is infected with a conflict of interest.
1
Note 5if a firm starts a case, and then the firm splits up, the division of fees among the members
of the former firm (not among non-members of the former firm) is not fee splitting within the
meaning of Rule 1.5(e)such a division is permissible.
1
Note 6 deals with sharing fees with non-lawyers and this is Question (c)--Rule 1.5(e) will not
allow a fee split with a non-lawyer.
a
As we saw in dealing with unauthorized practice, under Rule 5.4(a), a lawyer is not generally
permitted to share a fee with a non-lawyer; You can see the numerous examples of impermissible
fee splits with non-lawyers in the examples in the note; BUT
a
There are numerous exceptions.

First, it is permissible to share with the estate of a former member of the firm over a
reasonable period of time.

Second, it is permissible to pay the estate of a deceased or disappeared lawyer the


reasonable value of the practice of that lawyer when purchasing that practice under Rule
1.17.

Third, non-lawyer employees of the lawyer may be included in a firm benefit plan even though
that plan is funded entirely by law firm fees.

Fourth, as you can see at the end of the Note, Rule 5.4(a)(4), a lawyer may share legal fees
with a non-profit organization if that organization hired or recommended the lawyer.

Section F--Attorney Malpractice: Malpractice and Other Remedies


A
F(1) Basic Obligation of Care: Problem, Question (a) & (b)--Legal Theory for Malpractice and
requisite proof.
1
Togstad v. Vesely, Otto, Miller & Keefe, is the same case we dealt with on the issue of the
creation of the attorney-client relationship.
1
Facts:

a
a
a
a
a
a
a
a
a
1

1
a

1
1
a
a
a
a
a

Some 14 months following her husband suffering overly severe consequences of a medical
procedure, Mrs. Togstad was referred by a friend to attorney Miller for discussions about a
possible medical malpractice case.
Mrs. Togstad had no prior relationship--directly or through a family member--with Miller or
the firm.
Mrs. Togstad and Miller met for about 45 minutes in his office and discussed the possibility of
a medical malpractice action.
No fee was charged.
Miller kept no papers regarding Mrs. Togstads husbands case.
Mrs. Togstad said that Miller indicated that he did not think that Mrs. Togstad had a case, but
that he would discuss the matter with his partners and get back to her if he changed his mind.
When Mrs. Togstad did not hear, she assumed there was no case and did nothing further.
She was later advised there might have been a case.
By this time, statute of limitations had run on original action.
She filed a legal malpractice action against the law firm.
As the opinion suggests, because this is a Malpractice--rather than Disciplinary--action,
Privity is required--thus, there must be an attorney-client relationship between Miller and
Mrs. Togstad for the action to proceed.
In setting out the 4 elements for a malpractice cause of action, the court actually mentions two
distinct legal theories.
The first theory is Legal Malpractice under the Tort of Negligence and has the following
elements:

Duty--Supplied by A/C relationship.

Breach of one of those duties.

Damages.

Causation.
The second theory is Legal Malpractice under a Contract Theory and has the following
elements:

Request by client that lawyer provide services (Offer).

Consideration by client which can be based on reliance.

Acceptance by attorney.

Breach of one of the contracted for obligations.


As we decided in dealing with the attorney/client relationship, the court finds that when an
attorney is asked to render advice and does so in a situation in which it is reasonable for the
other person to rely on that advice, there is created either a duty, for tort law purposes, or a
request coupled with reliance and acceptance, for contract law purposes.

That finding then creates the requisite privity needed for a malpractice action.
The court found that Miller could have done several things to have avoided prejudice to Mrs.
Togstad.
Here, consider the lawyer's obligation under the Model Rules:
Rule 1.1 requiring competence--skill used by other similarly situated attorneys.
Rule 1.2(c) requiring that any limitations on representation be explained to and agreed upon
by the client.
Rule 1.3 requiring diligence.
Rule 1.4 requiring communication.
Rule 1.16 on withdrawal.

The rule allows an attorney to withdraw if good cause exists.

Good cause would exist if Miller and his firm did not think they could handle
the case.

The rule also requires that an attorney withdrawing must take steps
reasonably necessary to protect the client's interests.

a
1
1
a

1
a
a
a

a
1
a

a
a

As the opinion indicates, lawyer did not do the things necessary to protect those client
interests.
Use of these ethics rules to highlight the obligation of lawyer to client raises a couple of
interesting additional points.
First: The court mentions negligence in advice and the fact that "but for" this negligence, Mrs.
Togstad would have been successful.
As indicated in Note 6 following Togstad,--malpractice actions are usually viewed as "cases
within a case;" Thus, to be successful, plaintiff must prove that client would have been
successful in the underlying case; If the client would not have been successful in the
underlying case, the attorneys conduct does not cause the damage now.
That then brings up the notion of "but for" causation, mentioned in Note 3 following Togstad;
in both transactional and litigation cases, the client must show that but for the attorneys
conduct, the result would have been different or that the attorneys conduct was a concurrent
cause of the harm.
What about malpractice on appeal? Note 9 following Togstad.

But for causation on appeal is established by showing that but for the poor
performance on appeal, the appeal was lost; AND

Because of the loss on appeal, the underlying case was unsuccessful.


Second, Note 11 following Togstad.
Under the prior Rules, many courts would not permit use of the Ethics Rules to set out a duty
in a tort analysis, or to create a measure of contract performance in a contract case,
Those courts were looking at prior Model Rules Scope Notes, paragraph [6].
Today, court should look to paragraph [20] of the current Scope Notes indicating that
violation of a rule does not automatically give rise to a cause of action.

Why doesnt violation of a rule automatically give rise to a cause of action?

Because as we have just seen, there is not automatically causation from


violation of a rule; there also may be no damages.
As you can see from Restatement, Rules are the standard of care for attorneys and can be used
to show that standard.
Look at a couple of the other Notes:
Note 2 makes it clear that damages in a malpractice action are largely unlimitedthe benefit of
the bargain, the amount the client would have received in personal injury damages in the case
below, as well as additional legal fees are all part of the mix.
Note 5There is no requirement in most states that lawyers maintain malpractice insurance
Note 7 deals with Breach of Fiduciary Duty.

This often serves as an additional cause of action against an attorney.

As the Note indicates, breach of fiduciary duty claims often involve actual
breaches of duties such as conflict of interest, without regard to the damage from it; You
can see that the Restatement agrees that these kinds of violations serve as a basis of a
breach of fiduciary duty claim.

Consider a Missouri case: Lawyer represented a number of police officers in


defense of a civil rights claim. One of the officers claimed that he was not even present
at the scene and wanted to have the lawyer attempt to get the officer dismissed from the
actionthe lawyer delayed and the officer remained in the case for a significant period.

There were virtually no damages to the officer.

The underlying action was won by the officers.

The Missouri court found that breach of fiduciary duty is ONLY


available when neither negligence nor breach of contract can be used (and they
could not because of the absence of damage or breach of contract). Klemme v.
Best, 941 S.W.2d 493 (Mo., En Banc 1997)this is not what courts in other states
would domost would allow breach of fiduciary duty to be an additional claim.
Note 10need for Expert Testimonygenerally, must have expert testimony in malpractice case.

Expert needed to show both the duty (what rules were applicable to the
conduct) and

The breach of the duty (what the attorney did that deviated from what a
reasonable attorney should have done under the same or similar circumstances).
a
Note 15Judicial MalpracticeJudges are immune from damage suits even when acting in bad
faith, provided the judge had some subject matter jurisdiction and provided the act was
judicial in character.
Question (c)--If an attorney is held liable for malpractice, is there a credit against the judgment for
the amount of the fee the lawyer would have earned if the case had been successfully prosecuted?
a
This is answered in the last three paragraphs of Togstad.
a
No credit for fees that would have been earned in underlying action, primarily because that
reduction is likely wiped out by the fees the client will have to pay in the malpractice action.
a
What about contribution from the underlying tortfeasor?
a
This is Note 4 following Togstad.
a
The lawyer and the doctor are not engaged in any kind of joint enterprise so there is no
contribution.
a
Could be contribution from a joint counsel who helped work the underlying case.
1
Suppose the underlying judgment is uncollectible? This is Note 8 following Togstad.
a
If the underlying judgment could not be collected, the attorney is not the cause of the damage
to the client.
a
This, however, may be an affirmative defense that the lawyer would have to plead and prove.
Question (d) "Malpractice" in the criminal case/Ineffective assistance of counsel.
a
While malpractice in a criminal case is possible, see below, the more normal way in which
negligent or contractually inferior representation arises in a criminal case is by claim of
"ineffective assistance of counsel,"
a
Today, those claims are governed by the standards set out in Strickland Note 12 following
Togstad.
a
To require the setting aside of a conviction based on ineffective assistance of counsel, any
error by counsel must "prejudice" the defendant.
a
Prejudice is presumed where there is "actual or constructive" denial of counselprejudice per
se.
a
Prejudice is also presumed when there is an "actual conflict of interest," but to a lesser extent;
it is not prejudice per se; rather

Criminal defendant must show that the lawyer actively represented conflicting
interests AND

That this representation adversely affected the representation (but does not
need to show but for causation).
a
In all other cases:

Defendant must show with particularity the errors of counsel.

AND that BUT FOR the error, the result of the proceeding would probably be
different.
a
In judging prejudice.

Consider totality of the evidence.

Generally, decisions by lawyers on tactics will be entitled to heavy


presumption of correctness.

Must consider evidence as it appeared at time lawyer made decisions, not after
the fact.
1
While this standard is easy to articulate, does it actually prevent criminal defendants from
receiving "ineffective assistance? As Justice Blackmun's dissent in McFarland v. Scott (about
midway through the Note) indicates, the Strickland test has allowed some pretty poor
representation.
Also part of Question (d) is civil malpractice actions for criminal defense representation.

1
1

Note 13 following Togstad-- many states impose an additional element to the malpractice case.
That the criminal defendant/civil plaintiff be acquitted of the criminal charge before the
malpractice case is cognizable.
This will, in turn, be impacted by the statute of limitations.
If the statute of limitations runs throughout the period of incarceration, it could well expire
before a decision on appeal.
If the civil action cannot, however, be filed until there is a favorable decision on appeal, the
cause of action, if it accrues upon completion of the work, could expire before a cause of
action would be cognizable.
The issue is nicely set out in the Note and its cases; if a state follows the rule requiring
acquittal before the malpractice claim can be filed, the statute of limitations will have run on
the claim in the problem before suit could be filed.

1
a
a

F(2) Duties to Third Parties


A
Problem: If a lawyer drafts a document, like a Will or Trust, that is supposed to benefit certain named
beneficiaries and the lawyer performs negligently so that the named beneficiaries do not benefit, is the
lawyer liable to those beneficiaries for malpractice?
1
The problem here is, of course, privity.
1
The lawyer-client relationship is between the creator of the Will or Trust and the attorney.
1
This issue then often depends on whether the beneficiaries can be considered some kind of thirdparty beneficiary to the attorney-client relationship.
A

In response to Question (a), the court in Trask v. Butler, discusses the two tests that have been used to
impose liability.
1
First: Multifactor test requires court to balance a number of possible factors:
1
Extent to which transaction was intended to help beneficiary.
1
Foreseeability of harm to this plaintiff by attorney.
1
Degree of certainty that harm would result.
1
Proximity between attorney's conduct and injury.
1
Policy in favor of preventing further harm.
1
Extent of burden on profession by imposition of liability.
1
a
a
1
a
a
a

1
A

Second: Intended beneficiary.


If nonclient proves the primary purpose and intent of the attorney-client relationship was the
benefit of the third party,
Then court can impose liability.
This court does not adopt either test and, as a result, does not impose third party liability in
this case.
One of the reasons here is the relationship between the beneficiary and the personal
representative.
If the personal representative does not do what the testator wanted, the beneficiary has a cause of
action for breach of fiduciary duty.
Further, who does the attorney represent?

The estate, not the beneficiaries.

As a result, there is a conflict between the lawyer doing what is in the best interest of
the estate and the lawyer doing what is right for these beneficiaries.
Does court's decision forbid imposition of liability in any third-party case? NO

The Notes give you some variations.


1
Note 1all of the answers are yes; why then is there no liability.
1
Because of the availability of breach of fiduciary duty remedies against the personal
representative and

The lawyers conflict in representing the estate, but in working on behalf of the personal
representative.
1
Note 2 gives you the Kansas variation of Trask making it clear that the intent to benefit is the
first among the factors.
a
Thus if the person bringing suit is the adversary, there can be no intent.
a
If there is intent, then go to multi-factor test.
a
This version of the test probably better explains the result in Trask.
1
Notes 5 and 6 give you the idea of increased liability in business practice; Note the liability for
letters drafted by lawyer for client, but relied on by third parties.
1
Finally, Note 7 gives the Restatement view; there is liability.
a
If the client invites the third party to relythis may cover those cases in which a business lawyer
writes an opinion letter for third party reliance.
a
When the lawyer knows that a primary objective of the representation is to benefit a third party,
provided imposing a duty on the lawyer toward that third party will not impair the lawyers
obligations to the primary client.

This is point of the court in Trask.

It would also cover the adversary cases and likely the factual setting in Note 4.
a
Where client is a personal representative and the lawyer knows the lawyer needs to act because
of misconduct by the personal representative; this covers cases where the lawyer knows the client
is stealing trust assets.

This, in fact is Trask.

It would be interesting to see what court would do with this Restatement provision.
Section G--Termination of the Relationship--[this section deals with ending the attorney-client relationship
for reasons other than completion of the agreed task;]
A
G(1) Withdrawal
1
Under Model Rule 1.16, termination of the relationship is either mandatory or permissive
1
Question (a)client will not agree to settlement you think is reasonabledo you have the right,
under the Rules to withdraw?Certainly, this is Rule 1.16(b)(4) allowing (but not requiring)
the lawyer to withdraw when the client persists in a course of conduct with which the lawyer
has a fundamental disagreement; BUT
1
Any withdrawal is subject to Rule 1.16(d) requiring the lawyer to attempt to protect the
clients rights upon withdrawal. This, in part is Sobol v. District Court.
1
Facts:
a
Attorneys agreed to represent administratrix of estate.
a
Client wanted to sue another law firm and dispute developed.
a
Attorneys sought to withdraw.
a
Before they could, however, had to get permission of the court.
a
Trial court conditioned withdrawal upon attorneys using their "best efforts" to find the client
a new lawyer.
a
Lawyers found this an impossible standard to meet.
a
Appellate court reversed use of the standard.
a
Required trial court to allow lawyers to withdraw.
1
Teaching of the case, however, is twofold:
a
First, when attorney undertakes to represent a client, there is an "implied obligation" to
continue with the case until some logical conclusion; and
a
Second, where required to obtain permission of a tribunal before withdrawing, that
requirement overrides even a mandatory obligation to withdraw under Rule 1.16(a).

Any such motion is always addressed to the discretion of the judge.

This is Note 1 following Sobol.

You can see the balancing factors used by the court in exercising its
discretion.

This can put the lawyer in a difficult positionthe lawyer may have a
mandatory duty to withdraw, but the court refuses to allow that withdrawal.
a
In such a circumstance, Lawyer cannot meet obligation to client and to court.
a
Rules require lawyer to defer to obligation to court in this situation.
1
Problems
a
Now look again at Question (a)--Withdrawal would be permissive under Model Rule 1.16(b)
(4) or possibly (6), but if local rule requires, must get permission from court; (2002 Rules
reemphasize this requiring the lawyer to comply with local law on these issues;)
a
Question (b)--several problems with what you did.

First, however, because lawyer cannot offer evidence the lawyer believes is
false, following client instructions is likely to result in further violation of the rules,
thus making withdrawal mandatory under Model Rule 1.16(a)(1).

BUT, Withdrawal may require court approval.

In addition, any withdrawal, either permissive or mandatory, requires lawyer


to take steps reasonably necessary to protect the client's interest.

Simply leaving the client without an attorney is not likely to be seen as


protecting the client's interest.

Model Rule 3.3(c) also permits the lawyer to refuse to offer evidence the lawyer
reasonably believes is false.

In this problem, your knowledge, "beyond a reasonable doubt," as


indicated in the problem, seems to fit that standard.

Thus, even client insistence could be overruled by the lawyer.

That might create, however, a permissive withdrawal situation based


upon the same Model Rules used in question (a).
a
Question (c)--When client does not pay the bill, the attorney is permitted to withdraw under
Model Rule 1.16(b)(5).

Doesn't any such withdrawal look like the lawyer is abandoning the client?

If withdrawal requires court approval, won't the court have to balance this
appearance with the fact that the attorney should not be forced to represent a client and
the fact that the client should have ultimate choice--if the client does not pay the bill,
that amounts to a discharge of the lawyer--and that would require mandatory
withdrawal under Rule 1.16(a).

Look at Note 3 following Sobol.

When court does not allow the lawyer to get out, the lawyer is
essentially guaranteeing the client free legal services.

In the note case, the court would not allow that to happen.

The real problem in such cases is like that in Sobol--if the judge lets the
lawyer out, the case will not proceed very smoothly--and this is a judicial concern.
1
Finally, look at Note 2 following SobolA lawyer is permitted to withdraw for other good
causethis Note attempts to define that term.
a
As you look at the list in the Washington case, you should see that many of those reasons look
like they could fit into one of the defined provisions of Rule 1.16(b).
a
Probably key ones that might not be included are breakdown in communications and the
uncooperative client.
G(2) Discharge, Termination and Legal Fees
a
In our Problem you represent a corporation in defending a lawsuit--one that you think
subjects the corporation to a potentially high verdict. The Judge, at a pretrial conference,
suggests settlement for $200,000.
a
Question (a)--you recommend approval of the proposed settlement to the Board of Directors;
they disagree and say you're fired.
a
Starting point, Model Rule 1.16(a)(3)--when the lawyer is discharged, the lawyer is obligated
to withdraw.

This, in part, is Somuah v. Flachs.

Facts.

Petitioner hired Respondent to represent Petitioner and his daughter


in an auto accident case.

Respondent began investigating the filing of a lawsuit in Maryland


state court.

The problem was that Respondent is a lawyer admitted in Virginia,


but not in Marylandthis was not told to the Petitioner.

Eventually, the Respondent found a Maryland lawyer to help and,


when Respondent introduced this lawyer to the Petitioner, it was then that
Petitioner found out that Respondent was not admitted in Maryland.

The Petitioner then fired the Respondent.

The Respondent sued for the reasonable value of the services


rendered to the Petitioner.

Lets understand something at the outset.

Did the Respondent lawyer do anything WRONG in what he did for


the Petitioner?

While Respondent did not tell Petitioner about his inability to appear
in Maryland, there are no facts to suggest that anything that Respondent did was
wrong in a malpractice or unethical sense.

If we assume that the Respondent did nothing wrong, how does the client get
the right to fire the lawyer?

Part III of the opinion.

The client has the right to fire the lawyer at any time, with our
without cause.

Giving the client this right is necessary to preserve trust within the
relationship.

And again, understand something.

While this case may be about reasonable dissatisfaction, the rule


remains that the right of the client to discharge the lawyer is an implied part of
the agreement between attorney and client.

Here look at Note 1 following Somuah.

First that Note mentions AFLAC v. Williams, and its


teaching--that the client has an absolute right to discharge the lawyer at
any time, even without cause; Are there any qualifiers? YES.

When a lawyer is required to continue by tribunal, she must


do sothis is Note 7 following Somuah.

And note the discussion of standby counsel.

Court, in criminal case, even when counsel is fired by


counsel, can compel lawyer to stay on and assist the former client.

This may include answering questions or helping former


client understand the nature of the proceeding.

While it is not ineffective assistance for the court not to


appoint standby counsel, most courts who do it will do so to make sure
there is a lawyer up to speed in the event the client later wants to
discontinue their pro se appearance.

Also note that where permission of the court is required, the


lawyer is not relieved until that permission is obtained. As a result, the
lawyer is still liable on the caseand could be liable for malpractice.

Additionally, upon withdrawal, the lawyer must protect


client's interests.

The lawyer must advise of significant dates.

The lawyer must return parts of file.


The lawyer must cooperate with new lawyer.
The lawyer must refund any unearned fee.
The Clients right is subject to lawyers right to be paid for
the lawyers services, as noted in Comment [4] to Rule 1.16;In Question (a), lawyer is fired expressly, but does discharge have to be "express? Note 6
following Somuah.

When client files an ethics complaint, most courts considers this a discharge.

Similarly, when client shows up for court with a new lawyer, court views that
as a discharge.

Note provisions of Restatement listing other events that constitute discharge


death or incapacity of clientdeath or disbarment of lawyer

Again, however, discharge does not terminate obligation to court; if required


in such circumstances, attorney must still withdraw or file a substitution of attorney.
The second half of Question (a) deals with the issue of fees and this is Somuah.
If the lawyer is discharged without cause, the lawyer is entitled to the reasonable value of the
services up to the time of discharge.
As the dissent tells us, this case then breaks down discharge for cause into high grade and
low grade cause.

High Grade dischargesee Note 3 following Somuah.

Some misconduct is so serious that it eliminates any need to pay the


lawyer.

Serious breaches of conductusually including conflict of interest or


breach of fiduciary duty.

What kind of standard used to judgeobjective A court would look


at the conduct and determine if the lawyer had acted as a reasonable lawyer
would do and if the answer is nothe lawyer can be prevented from any fee.

1
a
a

Low Grade dischargeand Note 4 following Somuah.

In these cases, the lawyer has not done anything really wrong, but the client
subjectively believes the attorney should not continueand there is some cause.

Somuah is the perfect example.

Lawyer not admitted in Maryland.


Did not tell client.
But did not harm client either.
What could lawyer have done to turn this case into high grade discharge?
(Could have done nothing and let the statute of limitations runcould have
hired a Maryland attorney and never told clientcould have settled the case
without the clients permission).

Look at the cases in Note 4.

All of them involve some fault.


But fault that does not overly harm the client.
In those cases, what should the court do?
In each of these cases, the courts have to look at the case from the clients

perspective and determine, subjectively, what the lawyer should receive.


What and when does lawyer get paid?

Discharge without cause.

Under Somuah, if lawyer is discharged without cause, the lawyer is entitled to the
reasonable value of the legal services earned up to the point of discharge, and
can sue for that amount as soon as discharged.

This is the New York viewwhen client discharges the lawyer without
clause, the client is taking the position that the underlying contract has been
repudiated, thereby giving the lawyer the immediate right to sue.

The opposite is the California viewthat the lawyer is entitled to the


reasonable value of the services rendered up to the point of discharge, but
cannot sue for that amount until any contingency in the underlying contract
has been met; This is the Missouri viewexcept that Missouri adds the
additional requirement that the lawyer can never collect more than the
lawyer would have collected under the original contingent fee agreement.

Low Grade discharge.

Under Somuah, the attorney is entitled to the reasonable value of the legal services,
but not until any contingencies in the representation have been met; the reasonable
value can consider any fault by the lawyer in lowering the amount of the reasonable
value.

In states using the California approach to pay the lawyer discharged without cause,
there will be no difference in when the attorney can recover between discharge
without cause and discharge with low grade cause.

High Grade discharge.

Under Somuah, the Restatement, and most state interpretations, the presumption is
that the lawyer will get zero.

But if the lawyer has done any work, zero is likely to be a number of last resort for
the court which will attempt to provide the lawyer with some reasonable amount,
considering the lawyers fault.
1
Problem Question (c)--suppose lawyer is "house counsel?"
a
Then Note 2, should be considered
House counsel has the right to sue for damages for wrongful discharge, but must still withdraw.

A
I

One final topic, not covered by any of the problems; Attorney liens, Note 8 following Somuah.
A
Two kinds of liens.
1
"Charging or Special Lien," attaches to the funds in a case to pay the attorney fee in that case.
1
It simply allows the attorney to be paid out of the proceeds.
1
This is what allows the attorney to keep a claimed fee from the client and, if the amount is
disputed, deposit the funds in a trust account or in a separate interest bearing account.
1
The other type of lien is the "Retaining Lien; it attaches to anything belonging to the client that
comes into the lawyers hand as a result of the attorney client relationship.
a
Normally, this means client papers and the like.
a
But could also be money.

Where client gets big settlement from personal injury case.

Fee for the personal injury case would be covered by "charging lien;"

But if the client also owed for other non-personal injury work that
lawyer had done, a retaining lien would attach to the proceeds.
a
The purpose of this lien is to get payment.

Normally, what is held by the lawyer asserting a retaining lien has no intrinsic
value, and is thus considered "passive.

It is the coercive value of the lawyer holding the papers that gets the client to pay.

In the case of money, the proceeds from the personal injury case do have value.

By the same token, however, the lawyer cannot do anything with that money-other than to hold it--and can probably only hold the amount of the outstanding bill.

a
a

I
I

As a result of the oppression it can cause, retaining liens have been criticized because they run
afoul of it runs afoul of Rule 1.16(d), as you can see in Britton & Gray v. Shelton, the Oklahoma
case in the Note.

If the lawyer holds property belonging to the client.

And the only reason for so holding is the coercive effect, isn't the lawyer harming
the client's rights?

The last clause of Rule 1.16(d) allows for state variation on this subject by making
the lien subject to provisions of other substantive law

This court sets up a balancing test to be used when the client really needs a paper
to continue with a transaction.
Note that the Restatement does not authorize a retaining lien but does allow the lawyer and
client to contract for a charging lien;
Also note that the work product of the lawyer belongs to the lawyer and is not property of the
client that needs to be returned under Rule 1.16.

Chapter 4: Confidentiality
Confidentiality is one of the obligations of the lawyer to the client. Along with remaining free from conflict of
interest, which is Chapter 5, confidentiality preserves loyalty to the interests of the client.
Privilege and Confidentiality
A
As Comment [3] to Rule 1.6 indicates, preservation of confidentiality arises both because of the agency
relationship of attorney and client and because of the rules of evidence.
A
This dual history can be seen by review of former disciplinary rule DR 4-101(A).
A
Under this rule, "confidence" is information protected by the attorney-client privilege.
A
"Secret" is any other client information the lawyer obtains in the professional relationship that either
1
The client wants held in confidence, or
1
Which would harm the client if released.
A
Under this rule, all information considered "confidential," (protected by privilege) would also be
"secret," because by its very nature, release of privileged information could harm the client.
A
On the other hand, however, all "secrets" would not be privileged because, to be privileged, a
"communication" must be involved and that communication must meet all 6 elements contained in Note
1 of section A(1)(b).
A
This is the essence of the introductory problem and the two cases following.
1
Hughes v. Meade,
a
Facts.

A lawyer received a phone call from someone familiar with the lawyer's good
working relationship with the police department.

That person wanted the lawyer to return stolen property to the police without
getting the lawyer involved.

The lawyer called a police officer with whom the lawyer was friendly and made
arrangement to leave property.

And didthe lawyer refused to testify as to who called and was held in contempt.
a
Court held under these facts that the lawyer was not retained in a "professional capacity"
to act as a lawyer and delivery to police was not the rendition of a legal service.
a
As a result, the lawyer would have to reveal the name of the client in order to purge the
contempt.
1
Dean v. Dean,
a
Facts.

During a divorce case, the husband's office was burgled.

The Lawyer (who was not a lawyer in the divorce case) got a call from an
unidentified person.

The person knew the lawyer was a lawyer.

The person asked for advice about returning property.

The lawyer responded on basis of what he had learned as an assistant State's


attorney.

The Lawyer had been through similar problem with a hit and run case.

The lawyer asked the person if they knew that the lawyer was a lawyer.

The lawyer asked if the person was seeking legal advice

The lawyer asked if a condition of discussing the legal problem was protection of
the clients identity.
a
Court held the name protected.
1
Would either of these discussions have been necessary if all that was in issue is whether the
lawyer should have generally disclosed the nature of either of these conversations--to anybody?
a
No, clearly the information the lawyer received would be considered, in the context of the
former Disciplinary Rule, secrets;
a
As a result, the lawyer's duty as agent of the client principal would prohibit the lawyer
from discussing the information with the public at large.
a
Those general agent/principal rules, however, do not prohibit a lawyer from being
required to testify about the information; unless the information is not just a secret, but
also privileged.
1
What about problem?.
a
Maybe we don't know enough facts.
a
Even though Cleveland has hired other counsel for the civil and criminal cases does not
mean that he cannot hire Astor for a limited purpose.
a
If discussion between Cleveland and Astor meets the Dean requirements, the name can be
considered privileged.
1
Let's change the facts, however.
a
Suppose police don't demand--they simply request that Astor tell them.
a
And suppose Astor knows of both Meade and Dean and makes the realistic decision that
the court is not likely to protect the name of the client in this case.
a
Can Astor go ahead and give the police the name of the client?
Consider in light of current Rule 1.6
Rule 1.6 deals with "information relating to the representation;"
This is clearly a larger body of information than that which is covered by "privilege;"
1
For example, consider Fischer v. United States, Note 2, section A(1)(b) dealing with tax records.
a
Under Rule 1.6, the tax records given from the accountants to the lawyers would be
confidential, based on the obligation of the lawyer to the client under the law of agency.
a
These records would not, however, be privileged because they are records of the clients
transactions, not communications; if the client said: Ive committed tax fraud, and my tax
records reveal it, this statement is privileged as a communicationbut the tax record itself
is not.
1
In Morrell v. State, Note 3 in section A(1)(b), witnesses made statements about the client to the
lawyer.
a
These statements are confidential under Rule 1.6; this because of the obligation of attorney
to client under the law of agency.
a
They are not privileged
Information that is "Privileged" is thus a small class of information considered confidential
The lawyer's obligation of confidentiality arises from the Restatement (Second) of Agency requiring the
agent to be loyal to the principal's affairs.

A
A
A

A
A

A
A

This obligation arises without regard to the setting in which the lawyer might breach that
confidentiality--it is the agent/lawyer's duty to protect the confidences of the client/principal at all
times.
1
As a result, in our problem, Astor would be obligated to keep the information confidential--that
is not tell the police--even though he knows that it could later be compelled.
1
Because privilege only applies when there is testimonial compulsion.
Communications that are "privileged" are a special class of confidential information.
1
At common law, a party to a lawsuit was incompetent as a witness.
1
Others who heard information from a party were not, however, incompetent to tell what the
party had said.
1
Thus, if the party told a lawyer about a problem, the client/party would be incompetent to
testify, but the lawyer/hearer would not.
1
If lawyers were going to be any aid to the court, the privilege extended to parties had to be
extended to lawyers for those parties.
1
As it exists in most states, attorney-client privilege is found in statutes relating to witnesses-making the lawyer incompetent as a witness against the client.
1
Privilege is an exception to the normal rule of permitting persons with knowledge of events to
testify about them.
a
Anything said by client to lawyer is likely to be considered an "admission" and would thus
be valuable to the adversary.
a
As an exception, the rule is said to hinder the fact-finding process of most tribunals.
1
To be applicable, therefore, the elements listed must be present.
1
One of the critical elements that must be present is testimonial compulsion--the privilege rule
itself is a rule of evidenceit makes the lawyer incompetent as a witness; Thus, unless the lawyer
is under testimonial compulsion as a witness, there is no privilege applicable.
When those elements are present, however, the lawyer is forbidden from testifying about matters
covered by the privilege, unless the privilege is waived.
Consider in light of Problem,
1
Question (a)--the attorney is being deposed about his observations and about what the foreman
said on the tour.
a
This is a privilege situation because the lawyer is being deposed.
a
To consider whether the elements needed for privilege (Note 1 following the problem) are
present, we must understand the identity of the client.

In dealing with entity clients, our material in Chapter 2 told us that the entity
alone is the client.

But, entities only speak through people.

This has created a situation where communications between the lawyer


and a number of people within the entity may be privileged,

Upjohn v. United States helps determine with whom the lawyer can have
a relationship, the communications flowing from which will be considered
"privileged;"

Corporations made illegal campaign contributions to foreign


governments.

In-house counsel discovered and wanted to know extent of


involvement.

Sent out questionnaires to mid-level foreign general managers


and forepersons.

They returned the answered questionnaires.

Some information disclosed to SEC, but IRS wants more.

Corporation claims privilege, but lower courts reject under


prior "control group" test making privileged only those communications
between members of the control group of the corporation and counsel.

Supreme Court rejects this test as being too restrictive.

Some general rules flowing from the language used by the Court;
Privileged communications can exist.

When employee speaks with corporate counsel.

At the direction of management.

About matters within the scope of the employee's employment.

So long as corporate counsel is authorized to inquire about the


subject and seek information so as to perform one of the following.

Evaluating if the employee's conduct is binding on the entity.

Determining the legal consequences of the conduct.

Determining appropriate legal responses to the conduct.

As Notes indicate the Uniform Rules of Evidence were


amended following Upjohn to include the criteria mentioned by the Upjohn
Court.

In our problem Question (a), part (i) and (ii), there might be no difficulty in applying this
test either to what Harris observed or to the statements of the foreman to the lawyer,
however, privilege is not likely to be applicable because there was no expectation of
confidentiality in that the statements were made while others were presentin Note 1
following the problem, these communications were made in front of other interested
persons;

The argument could be made that these others are also within the scope of
privilege, but this is not likely to be successful; unless these people are within the
corporate family, the requirements for privilege are not met.

The observations of counsel will equally not be privileged for similar reasons.

This despite the general rule that observations made as a result of


privileged communications are equally privileged, as discussed in Note 3 following
the problem.

Here, the observations were not made in the context of any privileged
communication.

This is so because the presence of third parties destroyed the belief in


confidentiality.

Contrast the discussions of the incident and the observations with the statements
later made by the foreman to counsel.

These statements almost certainly qualify under Upjohn.

They were made by a middle management employee to counsel who


was investigating an event so that counsel could render advice to the corporation.
Question (b)subpoena for the statement of Jones, who observed the events, but who is not
employed by the Corporation. After the incident, Harris, the lawyer, took a statement from Jones,
who related what he saw and what the foreperson said; this is Work Product and the second
Upjohn opinion helps.

What does the opinion say constitutes Work Product.

Written statements, private memos, personal recollections, provided they are.

Formed by counsel; and

Formed in the course of counsels legal duties.

The reason for giving some protection to these items is that without some protection,
lawyers would have all their work requested by lazy lawyers on the other sideone side
would pay for all discovery; to avoid this, lawyers would not write down what needed to
be written.

When can material prepared by counsel be discoveredFed R. Civ. P. 26.


Material prepared in anticipation of litigation.
Prepared by a partys representative, including, but not limited to lawyer.
Only upon a showing of substantial need AND
An inability to obtain the information without undue hardship.
Even where the material can be discoveredthat is with the requisite showing.

There is no discovery of the lawyers mental impressions, conclusions,


opinions or legal theories.

Or trial strategy.

So look to Note 1 following work product; middle of the Notedetailing the facts of
Hickman v. Taylor.

The discussions between corporate representatives and counsel are privileged


because these are corporate employees speaking with counsel in the course of the
employees duties so that counsel can properly advise the corporation; they cannot be
discovered.

Statements by witnesses who are not part of the corporation can be in two
different classes.

First, the observations of these non-party witnesses are not protected in


any way--they are fact witnesses like any other fact witness.

As a result, they can be contacted by any of the parties; and they can be
subpoenaed to give testimony.

It would thus seem that statements written by these witnesses in their


own hand should be discoverable at will, even though those statements are in the
possession of counsel.

Second, suppose the lawyer who took the statements only took
statements from certain witnessesor asked the witness to write down what they
recall seeing only up to a certain momentor suppose the witness was deceased
and the statement to the lawyer was the only recorded testimony of that witness.

In any of these events, these statements, even though written by an


ordinary fact witness, look a lot more like work product; And if they are work
product, they can be reviewed under circumstances like those allowing review of
statements written by counsel.

Statements written by counsel of his interview with witnesses can be discovered,


but only upon the requisite showing of necessity and hardship discussed in Upjohn; this is
the answer to Question (b) in the problem.

But understand, this does not mean that counsel cannot talk to Jones.

If Jones is around he can be deposed and the factual information he


knows is not protected.

If Jones gave a statement to the police, or to anyone else, that statement


is not protected.

The only statement that is protected, then, is the statement made by


Jones to the lawyer.

The statements by counsel about how the witness will do cannot be discovered
because they are the lawyers mental impressions.
Question (c) deals with a non-testimonial revelation.

This is not a privilege situation because there is no testimonial compulsion.

In this situation, the general obligation of confidentiality applies.

Under Model Rule 1.6, the lawyer is obligated to keep information relating to the
representation confidential unless there is an exception that allows revelation.

In this problem, there is no such exception, so the attorney's revelation has violated the
rules of confidentiality and should subject the lawyer to discipline.

Exceptions to PrivilegeNotes following Upjohn (the first time).

Future crimeNote 2the intent by the client to commit a crime in the future, and the
attorneys advice on how to do itare not privileged (although depending on the crime,
they may be confidential).

Identity of clientNote 3normally, the name of the client is NOT privileged, but it can be
if learning the identity of the client will constitute a last link in the chain needed to convict
a criminal client.

Privilege after deathNote 4you can see that the court applies a balancing test on
whether to allow testimony about privileged information following the death of the
clientbut understand the presumption is that the information is still privilegedthat is
privilege survives the death of the client.
Exceptions to Confidentiality.

Note 5 following Upjohn makes the point that confidentiality applies to a broader ranger of
activity (everything) than does privilege (which applies only to attorney client confidential
communications).

Do we understand that everything that is privileged in the law of evidence is also


confidential under Rule 1.6, but that everything that is confidential within the meaning of
Rule 1.6 is not necessarily privileged in the law of evidence?

In the absence of an exception, the lawyer cannot reveal any information relating to the
representation regardless of how the lawyer acquired the information.

Rule 1.6 lists multiple exceptions, all of which authorize, but do not require,
disclosure of confidential information.

How about a statute like 26 USC 6050I that requires reporting of the name,
address, taxpayer identification number, the date and nature of all transactions in which a
person engaged in any business receives more than $10,000 in cash in one or more related
transactions?

Revelation of the "nature of this transaction" is clearly "information


related to the representation;"

Here, revelation is specifically authorized by Rule 1.6 which allows


disclosure when required to comply with law or court order.

Next, consider an attorney-client privilege situation.

When an attorney is called to testify, and is asked questions about client information, the
attorney is obligated to assert the privilege.

The court will then rule on whether privilege is applicable to the answer to the
pending question.

If the court rules that privilege is not applicable, then what?

If we assume that what is sought is still "information relating to the


representation;"

The information retains its confidentiality (though unprivileged).

Does the attorney then have discretion to answer the question?

Under Rule 1.6(b)(4), apparently so

The decision to comply with other law or court order is discretionary.

This result is solidified by Comment [11] to Rule 1.6.

Obviously, failure to answer could subject the attorney to contempt.


Implied Exceptions:

Comment [10] also makes it clear that other rules may permit or require the lawyer to
disclose.

These implied exceptions, then, could require what Rule 1.6 makes discretionary.

The comment does not specifically mention other implied exceptions within the Rules of
Professional Conduct, but consider these Rules.

Rule 2.3.

Rule 3.3.
Rule 4.1.
Rule 8.3.
When these rules are reviewed, it can be seen that these rules determine whether
revelation is required or discretionary.
Rule 3.3, for example, mandates disclosure, even if the disclosure violates the
general confidentiality notions of Rule 1.6.
Rules 4.1 and 8.3, on the other hand, mandate disclosure, but only if disclosure is
permitted under Rule 1.6.
The determination of whether an implied exception makes disclosure of
confidential information discretionary or mandatory rests, then, with the rule
embodying the implied exception, not with the discretionary standard of Rule 1.6.

Section A(2)--Direct Evidence in the Lawyer's Possession


A
Problem
1
Question (a).
a
The first issue in any such case is whether an attorney-client relationship existed.
a
Without such a relationship, the obligation of confidentiality under the Rules will not
arise.
a
While there was no formal agreement, no payment of money, it would seem clear that the
client, based upon the advice given, would assume such a relationship.
a
Thus, revelation of anything to the family of the watchman will involve revelation of
information relating to the representation.
a
Is there an exception so authorizing?.

None that are express in the rules.

No implied exceptions, other than as pertains to the physical evidence,


considered below.

What about simply making an anonymous telephone call?

This enables the lawyer to have a heart.

It also involves conduct that falls below the level of the rules.

Even though such revelation might not be caught, the conduct is still
impermissible.

Suppose, upon going out to the location, Barton discovers that the watchman is still alive
is there an exception?

Look at Rule 1.6(b)(1).

You can see from the portion that was excised that under the 2001 version of this
rule, there was no exception.

The lawyer under the prior (b)(1) could reveal only to prevent a client
from committing a criminal act likely to cause death or serious bodily harmhere
the criminal act has already been committed.

Now, the lawyer is permitted, but not required, to reveal when doing so
will prevent reasonably certain death or substantial bodily harmthe current rule
separates the criminal act from its consequences.

Whereas the prior rule allowed the lawyer to reveal only the act; the
current rule allows the lawyer to reveal the consequences, even if the act has
already been committed.
2
Question (b):

The only difference between a and b is the possibility that b could involve testimonial
compulsion, as where the lawyer is summoned to a grand jury investigating the
disappearance.

If the lawyer is called to grand jury, the lawyer must assert privilege.

If judge is then contacted and lawyer is ordered to testify, rules say lawyer may then
reveal.

If appeal is procedurally possible, lawyer should probably try to appeal.

If appeal substitute is available, such as action for prohibition (to prohibit the judge from
ordering certain questions to be answered), this should also be tried.

Generally, however, "final" order does not require the lawyer to be held in contempt before
answering.

Related issue: Length of Privilege--Suppose client kills him/herself--lawyer is not relieved


from obligations of confidentiality nor is lawyer permitted to waive the client's privilege
upon client's death. This is true even though the agency relationship of attorney-client is
terminated upon the death of the client/principal.
Question (c):--physical evidence (canvas bag, keys, photos taken by lawyer).

Are these confidential within the meaning of Rule 1.6?

Yes; they relate to and were obtained in the course of the representation.

Are they privileged?

No; they are all physical and will not necessarily qualify as "communications" for privilege
purposes.

They are thus subject to compelled demand by any lawful authority.

They gain no greater protection in hands of lawyer then they would have in hands of
client

The only possible exceptions are the photos

These are lawyer's work product and may require some higher showing before
being compelled from lawyer.

The second part of the question asks if Barton should do anything with these items
independent of some compulsion?

This raises lessons from Armani, Belge, and Stenhach.

Armani and Belge are part of same case, the case of Robert Garrow's lawyers.

The lawyers are told by their client of past murders he committed and
location of at least one of the bodies.

The lawyers investigated and found a body where the client said it
would be.

The lawyers rearranged the body and took photos of it; but left the body
where it was.

The lawyers refused comment on the murders, other than to tell the
parents of one of the victims they knew their daughter was dead.

All of this came out at trial on the client's attempted insanity defense.

The lawyers were then charged with failing to give a body a proper
burial; thus implying that what they knew should have been turned over.

Was what the lawyers were told confidential or privileged?

It was both.

It was privileged because it came from a client communication.

What the lawyers observed then, based on this communication,


is probably also privileged

Thus, the lawyers could not reveal and were protected from a
criminal charge.

The lawyers did do one thing wrong--they told parents they knew
daughter was dead.

There was no exception that allowed this disclosure.

There was no waiver of privilege by client, as Note 1 following Belge


indicates.

What about the courts statement that it would have had more difficulty
with a charge of obstruction of justice? Note 2, following Belge.

The New York Penal Law, section 5 makes it a crime to


suppress, by any act of concealment ... physical evidence;

By discovering this evidence and then telling no one about it,


didnt the lawyer conceal that evidence?

Aside from the criminal violation, would not telling be an


ethics violation for the lawyer?

Rule 8.4(b), (c), and (d) make it unethical for the lawyer to
commit a criminal act that adversely reflects on lawyers trustworthiness, or
engage in conduct that involves deceit, or engage in conduct that is
prejudicial to the administration of justice.

Thus it would be unethical for the lawyer to violate the New


York criminal law.

What problems exist with that?

The lawyers in Armani and Belge were simply trying to confirm


what the client told them by way of privileged communication.

Weve already said that what the lawyers observed as a result


of the privileged communication was probably also privileged.

How can the lawyer be unethical for maintaining privileged


communications? That is part of the point in the next case.
Commonwealth v. Stenhach.
Lawyers represented defendant accused of murder.
Met with client and investigator.
Client told the lawyers about the murder.

What issue immediately arises?

Whether these conversations are "privileged" in light of fact


that investigator was there.

Privilege will extend to employees of the attorney.


Client and investigator drew map of where part of the rifle was located,
investigator went out, got the stock for the rifle and returned it to lawyers.

This case then differs from Belge in that lawyer or person


acting for lawyer has here removed the evidence.
Balance of rifle was not found by investigator--was found by State and
introduced at trial.
At trial, prosecutor asked investigator about rifle.
Objection on privilege grounds was overruled.
Investigator testified and stock of rifle was returned.
After trial, prosecutor charged attorneys with hindering prosecution
and other criminal violations.
Court first looks at statute dealing with hindering prosecution and
indicates that it is reviewing the statute for possible over breadth.
Court refers to cases from other jurisdictions.
State v. Olwell.

Attorney held in contempt for failing to answer questions and


produce weapon.

Court held the attorney has a duty, on attorney's own motion,


to turn over the physical evidence.

Can withhold it for reasonable period of time, but ultimately


must turn over.

It is not privileged.

But State must take extreme precaution when introducing the


evidence to prevent source of evidence from becoming known to finder of
fact.
People v. Meredith,

Investigator for lawyer found and removed wallet from the


crime scene.

The wallet was a vital link in the chain of evidence needed to


lead to the defendant.

Ultimately, lawyer turned wallet over to the police.

The wallet was confidential because related to representation.

The wallet not privileged because not a communication.

When removed or changed, lawyer deprives state of


opportunity to observe evidence in original condition.

9th Circuit case of Clutchette v. Rushen, is similar in that the


investigator removed physical evidence, but turned it over to police. There
was not a constitutional prohibition against introducing the evidence, but
the prosecutor had to take care not to reveal the source of it.
People v. Lee,

The defendant's wife gave evidence to the lawyer who gave it


to a third party.

Court allowed search warrant to find and allowed testimony as


to source.

There is no privilege here, because, even though


communication, it was not from client.
Morell v. State is similar.

A third party brought physical evidence to the lawyer.

Because it did not come from the client, and because it is not a
communication, it is not privileged.

Ethics committee, upon attorney's request for advisory


opinion, told lawyer to turn it over.

What is there that compels this duty to turn over the evidence?

Is the evidence "information relating to the representation?

Certainly, and all the courts recognize that fact.

There is no express exception in Rule 1.6.

There is no implied exception?

No other rule permits or requires it?


Stenhach does not mention it, but all of these cases discuss the kind of
criminal statute like that under which these lawyers are charged.
Like the New York penal law, Note 2 following Belge.
In Belge, the court said it would have difficulty if the lawyers were
charged under a hindering statute, but in Stenhach, the court has no such
difficulty-
The Stenhach court holds that the criminal statute under which
the lawyers were charged is unconstitutionally overbroad when applied to
attorneys.

And does so for the same reasons we discussed earlierif the


client gave the attorney a handwritten note detailing the clients
participation in a crime, the lawyer would be guilty of hindering if he
concealed the note. At the same time, however, the attorney is prevented by
privilege from revealing that note.

In Note 2 following Stenhach, you can see this aspect of the


courts decision has been criticized for its own overbreadthwouldnt it have
been easier for the court to simply say the lawyers cannot be guilty of
hindering if what they held was privileged?

The court is thus finding that this statute is unconstitutional


when applied to attorneysand is doing so largely on the basis of the courts
inherent power.

At the same time, however, the court does not let the lawyers
off the hookparticularly in light of the cases referred to by the court. Instead
of relying on the criminal statute, the court exercises its inherent power and
imposes a duty on lawyers along the lines of the holdings in the cases it
discusses.

Duty imposed by the court is based upon a synthesis of all of the cases
cited and discussed by the court.

A lawyer may keep physical evidence for reasonable time for


examination.

The lawyer may then return the evidence to its source if that
can be done without altering, destroying or concealing the evidence; or
impairing its verity or availability.

Otherwise, the lawyer must turn the evidence over to state.

The state may use the evidence, information relating to


condition, location and discovery.

But, the state cannot use source of evidence.

Lets consider the returning evidence to its original location optionand here look
at Note 3 following Stenhach.

If you have taken the evidence, then had it tested, then tried to return it,
can it ever be in the same condition?

You can see that the drafters of the Restatement agreed with those who
thought the evidence could not be in the same condition.

How does lawyer turn evidence over?

The name of the client is not generally privileged--except where name


provides last link to crime, as indicated in the earlier Notes on privilege.

Thus, prosecution would generally be entitled to obtain name of person


who gave material to lawyer.

Whenever lawyer has permission or obligation to reveal confidential


information, the lawyer must still preserve confidentiality to the extent possible.

One way might be for lawyer to hire another lawyer for limited purpose;
following example of Dean v. Dean: the second lawyer would know the name of the
first lawyer, but not the name of the client.
To summarize, in question (c) in our problem.

Bag and contents are evidence and must be turned over either because state statute or
because of implied obligation arising in Stenhach.

Keys are evidence linking client to crime--like the wallet in Meredith, and must also be
turned over.
That leaves only the photographscertainly they are not communications and are not therefore
privilegedbut they could be considered work product of lawyer and therefore subject to higher
showing of need;

Fletcher v. Union Pacific RR,

Facts:

RR employee injured and filed suit against the railroad.

A medical exam by railroads doctor revealed no significant damage to


the plaintiff.

The Plaintiff wants to obtain copies of surveillance film that the railroad
took of the plaintiff following the accidentplaintiff claims these films will
corroborate his injuries.

The Defendant railroad refuses to turn these films over, claiming they
are work producteven though they will not be offered in evidence at trial (which
almost certainly means the films favor the plaintiffs side of the case).
First, the court finds the films to be work product.
As a result, the party seeking them must prove.

A substantial need, which includes.

Whether the information sought is essential to the partys case.

Whether the information can be obtained from an alternative


source.

Undue hardship if the party does not have this evidence, which
includes.

The burden of obtaining similar evidence from an alternate


source.

That could include the fact that witnesses might not remember
or that it is just too costlybut not merely just expensive or inconvenient.
The key to this case is the fact that the evidence sought only corroborates other evidence.

The plaintiff can call his doctor.

The plaintiff can call his friends.

The plaintiff can call his family.

The plaintiff can put in medical records.

All of which will detail plaintiffs injuries.


Because the evidence is available elsewhere, the plaintiff cannot show either need or
hardship.
The case is therefore easily decided against the plaintiff obtaining a copy of the film.
But thats not the point for us.
Lets try to look at both evidence of crime and work product; Look at Note 1
following Fletcherin Stenhach, the court imposes a duty on lawyers to turn over physical
evidence but in Fletcher, the court protects lawyers who conceal evidence. What is the
difference?
One difference is that Stenhach is a criminal case.

Here, Note 2 following FletcherA California case where the client


thought he may have been involved in a crimethe lawyer took pictures of the
clients van and showed them to police, after editing out the license plate number;
when prosecutor demanded the photos (and films), the lawyer resisted.

Certainly the photos are not privileged, but they are work product.

Here court rather easily eliminates work product by saying that it is


waived by showing the work to the police.

While a claim of work product is defeated by voluntarily


showing the work product to third parties.

You certainly get the impression that because this is a criminal


case, somehow the rules are different.

Why doesnt the lawyer have a Stenhach duty to turn this evidence
over?

One explanation is that the license plate does not match the car
being sought by the policetherefore the lawyer concludes that what the

lawyer possesses is NOT evidence; But that seems illogical because the
lawyer fights the case.

The other explanation is that California has not recognized the


Stenhach duty to the same extent as recognized by the court in Pennsylvania.

This case is difficult on a number of levels and thats probably why the
California Supreme Court does not want it cited.
The other reason there is a difference between Stenhach and Fletcher is that
courts simply want to protect work product.

The original notion of Hickman v. Taylorthat one side should not be


required to make the other sides casecontinues.`

Our adversary system must be protected and work product does just
that.

Section B--Protecting Third Persons from Harm by Client


A Problem, Question (a)
1
Can the lawyer telephone the police or Adams and warn them of client's assertion and apparent
ability to carry through with it?
a
Because the information about client comes to the lawyer in the context of the attorneyclient relationship, it is confidential within the meaning of rule 1.6; this is NOT a privilege
situationat least not yetwhat is missing? Testimonial compulsion.
a
Therefore, to be able to reveal would require an exception; like Rule 1.6(b)(1).

In question a, if Madison "reasonably" believes it necessary, the lawyer may reveal the
information necessary to prevent death or bodily harm.

Look at Comment [12]the lawyer still has obligations to the clienttherefore any
permitted disclosure should only reveal so much as necessary.

If disclosure is made in connection with a judicial proceeding, lawyer should attempt to


protect the information as much as possible.
1
Question (b)--if the lawyer does not reveal, and the client burns down the house, is the lawyer
subject to discipline or civil liability?

Again, note the language of the rule--"A lawyer may reveal ..."

Look at Comment [15], middle of Commenta lawyers decision not to disclose does not
violate the rules of professional conduct.

This statement is considerably weaker than in the 2001 version of the rules; Under the
"Scope" provisions of the 2001 Rules, interpretive paragraph [8], the decision by the lawyer
to (or not to) reveal should not be subject to later reexamination.

The change thus protects the lawyer from an ethics proceeding for failing to reveal, as the
pre-2002 Rules did, but apparently subjects the lawyer to possible tort liabilityif a duty
can be established between the lawyer and the injured party.

This change is NOT the position of the Restatement which, in section 67(4), indicates that
the lawyer who elects to not reveal is not ... liable for damages to the lawyers client or
any third person exclusively on the basis of the failure to disclose.

The civil law is, in part, the subject of the next two cases.

Hawkins v. Kings County.

A lawyer was appointed to represent Michael Hawkins, jailed on drug charges.

Michael wanted to be released on bail.

The lawyer spoke with Hawkins' mother and psychiatrist, both of whom indicated that
Hawkins was mentally ill and dangerous.

Both the mother and the psychiatrist did not think Michael should be released.

The lawyer followed the clients instruction and argued for bail.

As a result, Michael was released on bail and 8 days later, assaulted his mother and then
jumped off a building causing himself injuries necessitating amputation of both legs.

A negligence action was filed against the lawyer claiming the lawyer's failure to disclose
mental problems at bail hearing and failure to warn others of dangerousness was the
cause of harm.
The case forces us to consider for whose interests the Lawyer argued.

If the lawyer was representing Michael's mother, or the psychiatrist, or possibly


even society generally, the lawyer may not have obtained best result.

If the lawyer fulfilled the role as officer of court, the lawyer could be said to have
duty to advise of any relevant factor in making bail decisionwe would then have to
determine if the mental illness of a jailed defendant is "relevant" to the bail
determination.

If the lawyer represents the client, and the client wants to be released on bail, the
lawyer did what the lawyer was supposed to do.

That, of course, makes the lawyer look like a hired gun--particularly where the
client is or could allegedly be mentally ill and thus unable to make reasoned
decisions.

Rule 1.14 considers this problemas indicated in Note 5 following Gonella.

Rule 1.14(a) requires the lawyer to maintain as normal an attorney-client


relationship as possible with any potentially disabled client.

This rule allows others (courts, family) to determine the mental or


physical status of client.

Meanwhile, the lawyer advocates for the client rights.

Rule 1.14(b) permits the lawyer to consult with others when the client is
unable to make reasonable decisions.

This rule will certainly involve the lawyer revealing information relating
to representation (the lawyer's impression of client's condition)

As a result, the rule indicates that revelation of information protected by


Rule 1.6 is impliedly authorized; Look at Comment [8] to Rule 1.14
indicating that disclosure is impliedly authorized even when the client is
opposedpresumably, disclosure is impliedly authorized because the lawyer
has determined that the clients legal interests require such disclosure
although this rationalization by the drafters of the new rules is difficult to
understand.

As Comment [8] indicates, at the very least, the decision to disclose


under Rule 1.14(b) should be made with caution and with considerable
regard to the confidentiality provisions of Rule 1.6.
In Hawkins, the lawyer did not really have a Rule 1.14 problem. Others thought the client
was mentally ill, but lawyer did not so find.
As a result, the lawyer treated Michael like a normal clientat that point, the lawyer was
obligated to the provisions of Rule 1.6.
Under Rule 1.6, the lawyer was not authorized to disclose the clients condition.
If there is any obligation to disclose, then the compulsion to reveal must come from some
outside force such as tort law.

Hawkins stands for idea that disclosure to third parties is not generally a tort
obligation.

Certainly not an obligation to those who already know of the danger (in
Hawkins, for example, Michael's mother and psychiatrist).

There is language indicating that if the Lawyer believes beyond a reasonable


doubt, that danger is real, there may arise a duty to unknowing third parties.

As Note 1 following Gonella indicates, the tort duty to disclose is fairly


commonplace in dealing with physicians.

As to lawyers, however, no court has yet so held, and that is likely due to the
discretionary posture of Rule 1.6.

What does In re Gonella add to our discussion?

The client made allegedly threatening statements against a third party to the lawyer.

The lawyer was then subpoenaed to testify.

The lawyer resisted, claiming privilege.

The court overrules privilege, because of common law crime-fraud exception-refer to Notes on privilege and their exceptions following first Upjohn decision.

The court does not address confidentiality under law of agency; as an agent, the
lawyer has an obligation of confidentiality.

Rule 1.6(b)(1) partially overrules the agent's obligation of confidentiality by


permitting, but not requiring, the lawyer to tell in situations not involving
testimonial compulsion.

Gonella was still troubling for the court, and that is what this case adds.

The court limits the disclosure only to the alleged threatsand prohibits a fishing
expedition by the prosecutor into the attorney client relationship.

Look at Rule 3.8(e).

A prosecutor is not allowed to obtain information from a lawyer unless:

The prosecutor reasonably believes information is not protected by


privilege; AND

The information is essential to the success of an ongoing investigation;


AND

The information is not reasonably available elsewhere.

Even where information is not privileged; and even where the court has ordered the
lawyer to reveal; and even where the lawyer believes that the lawyer should reveal under
Rule 1.6, it is still difficult for the court to justify interference with the attorney client
relationship.

If that gives you the idea of some sort of presumption against revealing, that is probably a
correct impression.
Problem, Question (c) requires us to review any distinctions between crimes of violence and
fraud-type crimes, that is where there is no threat of physical harm to anyone, only the possibility
of fraud upon a personsome of these distinctions are pointed out in Notes 2 and 4 following
Gonella.

Look at New Jerseys Rule 1.6(b)(1) in the middle of Gonella.

It REQUIRES disclosure.

It requires disclosure not only of actions likely to cause death or bodily harm but also of
actions that will cause property or financial harm.

As you will recall, the common law crime/fraud exception exempts from privilege
information about a clients intent to commit a crime in the future.

This is without regard to the nature of the crime.

Thus, the intent by the client to commit a crime involving only property would be
as exempt from privilege as would be the intent by the client to commit a crime
involving substantial bodily harm.

The pre-2003 version of Rule 1.6(b)(1), however, allowed the lawyer to disclose
only those criminal acts by the client likely to result in imminent death or
substantial bodily harm.

Thus, under this rule, lawyer could be required to testify about clients intent to
commit other crimes, but could not voluntarily give that information up.

As you can see in Note 2 following Gonella, however, only a handful of states
adopted the ABAs version of the pre-2003 Rule 1.6(b)(1).

Most states kept the former Disciplinary Rule version that allowed a lawyer to
reveal the intent by the client to commit any crime.

Understand what is going on here.

As we discussed early in the course, the most difficult issues of


professional responsibilities involve conflicts among the lawyers duties to
clients, to the courts, and to the public generally.

When the rules protect more communication, they opt for protecting the
attorney-client relationship.

When the rules permit or require more disclosure, they protect the
courts and the public.

Current events are likely to have an impact on the issue.

You can see the impact of current events in the two versions of the new Rule
1.6(b).

In the draft adopted by the ABA, the lawyer was permitted to disclose in
order to prevent death or bodily harmregardless of when the act took place.

That was a significant changebut not one that allowed disclosure of


damage to financial or property interestsas was true under the New Jersey
in Gonella.

However, in light of corporate scandals of the late 1990's and later, the
Rules were amended in August, 2003.

Rule 1.6(b) now permits, but does not require, the lawyer to reveal
additional information.

First, the lawyer is permitted to reveal the intent by the client to commit
a crime that is reasonably certain to result in property or financial damage to
another PROVIDED the lawyer has been used or is being used by the client
to further that crime.

Second, the lawyer is also permitted to reveal past actions by the client
that are reasonably certain to result in property or financial damage to
another PROVIDED the lawyer has been used by the client to further the
crime or fraud.

As Note 2 following Gonella indicates, this was already the position


taken by the Restatement and while the ABAs position is new, it really only
conforms to what is, and has been, the law in most places.

Thus, in Problem, Question (c).

The information from client is not privileged.

It cannot be revealed under the pre-2003 version of the Rules, although the
lawyer cannot aid in the perpetration of fraud, or counsel the client on how to
commit the act.

The information can, however, be revealed under the current (post August, 2003)
version of Rule 1.6(b).
1
A different issue is permissive versus mandatory disclosureNote 4 following Gonella.

In Gonella, under the New Jersey rule, the lawyer was required to reveal those actions of
the client covered by the rulelook at it again.

Under the current (post August, 2003 version of) ABA Rule 1.6(b), the lawyer is permitted
to reveal

The Restatement, Section 67, agrees with the current ABA position, that is, permitting, but
not requiring disclosure.

The balance of Note 4 details what other states do.


The Next Problem is slightly different, but raises some of the same issues about disclosure of fraud.
1
In this problem, client commits fraud upon others while the attorneys are employed by the client.

This, then, is not a situation where warning a third party would do any good, because the fraud
has already been perpetrated; it is, however, a situation where, if the lawyer does anything to
protect the third party, the lawyer will be revealing information otherwise protected by Rule 1.6.
Note history of "corrective disclosure" provisions.
a
Originally, D.R. 7-102(B)(1) required disclosure to affected person of the clients fraud.
a
ABA interpreted the provision to not require disclosure if information about the client's
fraud

Section C--Protecting the Courts: Client Perjury


A
Problem--In representing a criminal defendant, you find that state has evidence that will place your
client at the scene of a robbery, but little else upon which to secure a conviction. You advise the client to
remain silent, but the client wants to take witness stand and testify to an "alibi" story that you now
clearly believe is false.
A
Some Legal Background:
1
When the client wants to--or does--lie on the witness stand, the lawyer is caught between the
duty to advocate the best interests of the client and the duty as officer of the judicial system.
1
Usually, it is in the client's interest to lie--at least that is what the client thinks.
1
If the lawyer refuses to allow that to happen, the lawyer, it has been argued, is not representing
the client effectively.
1
Our earlier discussion of the various models of the adversary system is also at issue here.
1
Complicating matters is that on many occasions, the decision to lie takes place in a criminal case.
a
From Rule 1.2(a), we know that the decision by a criminal client to take the witness stand
is an objective of the representation controlled by the client.
a
As a result, the client's decision to testify is said to be controlling.
a
Further, in criminal trials, the 6th Amendment to the U.S. Constitution requires the
criminal defendant have "effective" assistance of counsel.
1
Responding to this, the American Bar Association, in their ABA Standards Relating to the
Administration of Criminal Justice, The Defense Function, originally proposed that if withdrawal
was not practical or feasible when the criminal defendant wanted to lie.
a
Counsel could ask questions that would produce truthful answers.
a
When the defendant wanted to lie, counsel could ask a "free narrative" question.

This would allow the defendant, on his/her own, to tell the fact finder whatever
the defendant wanted to tell.

Counsel could not use any of this information in further questioning or in


summation.
1
While this standard was proposed, it was never adopted--primarily because it involved the
attorney participating, at least passively, in fraud by the client.
1
The drafters of the Rules of Professional Conduct attempt to resolve this difficult problem in Rule
3.3.

Rule 3.3 is applicable in either a civil or criminal proceeding.

Under Rule 3.3(a)(3), the attorney MUST refuse to offer evidence the lawyer knows is
false.

Under the same Rule, the attorney MAY refuse to offer evidence the lawyer reasonably
believes to be false, EXCEPT the testimony of a criminal defendant.

Comment [9]because of the protections given to a criminal defendant, that


defendant must be allowed to testify if the D wants to do so.

Thus, if the lawyer reasonably believes the testimony will be false, the lawyer
must still allow the defendant to testify if the defendant wants to do so, even to what the
lawyer reasonably believes will be false.

But, if the lawyer KNOWS the testimony is false, the lawyer cannot offer this
testimony.

Also under Rule 3.3(a)(3), if the lawyer, the lawyers client, or a witness called by the
lawyer has offered material evidence and the lawyer later learns that the evidence was
false (NOT reasonably believed), the lawyer is under an affirmative obligation to take
reasonable remedial measures.

Under Rule 3.3(b), in an adjudicative proceedingand this is certainly more than just being
in court, if the lawyer represents a client in that adjudicative proceeding and KNOWS that
a person INTENDS to or IS, or HAS engaged in criminal or fraudulent conduct in that
proceeding, the lawyer MUST take reasonable remedial measures, which could, but does
not necessarily have to, include revelation of information otherwise protected by Rule 1.6.

Comment [12] to Rule 3.3 makes clear that the rules apply regardless of the civil or
criminal nature of the case.

If the Constitutional obligation of the lawyer to provide effective assistance were


interpreted to obligate the lawyer to a different standard, Comment [12] also makes clear
the rule would then be subservient to the constitutional requirement.

Thus, looking only at the rules, while Rule 1.2 would make the decision to testify in a
criminal case an object of the representation, that object would be modified by the
provisions of Rule 3.3.

In this circumstance, the Rules have opted to protect the courts more than protecting the
rights of the client.
This does not mean that the lawyer is always permitted/required to reveal client perjury; rather, under
the rules, and the cases, the lawyer's obligation is partially affected by when the lawyer learns of the
client's decision to commit perjury.
Lets look at the problem.
1
Question (a) Client wants to lie in a pending case--When the client advises the lawyer of the
intent to take the stand and lie in an upcoming case, the situation is akin to Nix v. Whiteside.

The lawyer represented Whiteside, accusing of murder in a drug transaction.

Whiteside went to the victim's residence and there was a dispute.

The victim told his girlfriend to "get me my piece;"

There was further argument and the victim reached under a pillow whereupon Whiteside
stabbed the victim.

Just before trial, Whiteside, for the first time, recalled seeing something "shiny" just before
stabbing victim.

When asked about it, Whiteside told the lawyer that Whiteside thought there
needed to be a gun for Whiteside to get off.

Despite the lawyer's statement that a gun was not necessary, the client continued
to make these assertions.

The lawyer told the client that the lawyer would withdraw if Whiteside testified falsely.

Whiteside did testify, truthfully, and challenged his conviction on the basis of ineffective
assistance, claiming that the lawyer should have allowed him to testify to something
"shiny;"

The Supreme Court first assumed that any testimony about "shiny" object would, in fact,
be false.

The Court then refused to require counsel to offer such testimony.

This, then, follows Rule 3.3(a)(3)

The lawyer may not offer testimony the lawyer knows is false.

And the lawyer's failure to offer this evidence will not be ineffective assistance.

Difficult issues of proof arise for the lawyer; that is, when does the lawyer "know"
the client's story is false.

Best standard from the cases is "firm factual basis" for the belief.

But note People v. Bartee, rejects that standard in favor of more lenient
"good faith" rule.

What do Rules say? Rule 1.0(f) defines know for Rule purposes as
actual knowledge, although the rule indicates that actual knowledge can come
from the surrounding circumstances.

Rules are thus more stringentunless the lawyer KNOWS, and here look
at Comment [8] to Rule 3.3, the lawyer cannot ignore an obvious falsehood, the
lawyer must let the client testify.
What about other options? These are in Notes following People v. Bartee.
Nix indicates that when the client wants to lie, the lawyer should always try to
dissuade the client from doing so, Note 1 following Bartee.
The lawyer can seek withdrawal, as indicated in Nix v. Whiteside, and as
discussed in Note 2 following Bartee.

Indeed, if the lawyer refuses to put the testimony of the client on, lawyer
may be in situation where withdrawal is either permitted or required under Rule
1.16(b) or 1.16(a).

If the lawyer seeks to withdraw, and the rules of the tribunal so require,
the lawyer must still seek permission of the court to withdraw.

That motion should not disclose the reason for seeking withdrawal;
Disclosure would disclose the client's intent to lie.

Here, Compare Note 4 following Bartee quoting an article


indicating that disclosure of the clients intent to lie destroys the attorneyclient relationship.

With Rule 3.3(b) which appears to permit the lawyer to


disclose to the tribunal the clients intent to lie.

What is clear from the comments is that the purpose of Rule


3.3 is to prevent fraud upon the tribunal.

If the client only intends to lie, there has been no fraud yetbut
the lawyer must take steps to avoid that fraudsurely something other than
disclosure might sufficeand the comments to Rule 3.3 do not mention
disclosure in this scenario.

If the judge forces the lawyer to disclose, the lawyer may need to file a
further motion to recuse the judge, particularly if the case is to be judge-tried.

When the lawyer withdraws, and new counsel is appointed, understand


that the client will tell the new lawyer the false story from the outset.

Thus, allowing the original lawyer to withdraw only encourages later


perjuryand thats why Comment [15] does not require the lawyer to seek
withdraw.
Another possible option is to allow client to testify in a free-narrative format, as
indicated in People v. Bartee, and Note 5 following Bartee.

In the middle of trial, defense counsel advised court that a situation had
developed between counsel and client that required counsel to withdraw--counsel
cited Nix in seeking to withdraw.

The court denied counsel's request to withdraw, but ruled that the
defendant could testify in narrative form.

The court indicated that counsel could not ask questions other
than the free narrative question.

The court further indicated that counsel could not use


testimony produced from the free narrative testimony in closing.

The Illinois appeals court held that it was not ineffective assistance of
counsel to compel testimony in this fashion.

Use of the free narrative does aid the fraud of the client.

It may also avoid retrial of the case.

Look at Comment [7] to Rule 3.3.


Free narrative is not an approved option.
Rather, it can be used only when required by a tribunal or a
jurisdiction.

As indicated in Note 5 following Bartee, the Restatement, in


comments to section 120, agrees with Comment [7].

Before using the free narrative, then, you should be sure the
court or the jurisdiction requires its use.

Note that nothing yet has required the lawyer to reveal anything, other than what the
lawyer would be required to reveal if the lawyer seeks to withdraw.

Go back, then, to Question (a).

You must first seek to dissuade Howard.

If you cannot, and if he insists on taking the stand, you must decide if you
KNOW he is going to lie.

And by know we mean actual knowledge.

Although that knowledge could come from circumstances.

If you KNOW he is going to lie, you cannot put him on the witness stand.

If this happens, you may wish or be required to withdraw.

In making motion to withdraw, you are allowed to reveal, but must


protect the client, therefore you cannot reveal too much.

A good judge will prevent the lawyer from withdrawing because the
judge now knows that the lawyer has an ethics problem and wants it addressed by
the lawyer who knows all of the problem.

If you only reasonably believe that client will lie, you must put the client on the
witness stand and allow the client to testify.

This is NOT a situation where you need to ask a free narrative question,
because Rules allow the client to testify.

Here, then, you can ask client regular questions, UNTIL you KNOW the
client is lying.

When does free narrative come up here?

Only if you KNOW the client is going to lie.

And only if the client somehow gets on the witness standit could be
that the court compels you to call him.

And only if the court or states rule requires you use this type of
questioning.
Question (b)you put the client on the standthis same scenario could happen in a number of
ways: For example, Either judge refuses motion to withdraw and lawyer puts client on stand to
testify to truthful material; or client does not suggest perjury and does get on stand; and, after
getting on stand, client lies--unanticipated client perjury--Unanticipated client perjury arises
whenever the client is on the stand and, in response to a question, begins to testify falsely.

This situation requires application of another portion of Rule 3.3(a)(3).

It is clear that the lawyer has an obligation to take remedial measures, but what?

There is universal agreement that the first measure is to attempt to have the client recant
the perjury and testify truthfully.

This is usually done by asking the court for a brief recess.

The lawyer then explains to the client what will happen if the client
continues to lie.

If the judge denies the recess, or if the client refuses to rectify, there is some authority that
the lawyer should seek to withdrawal.

This raises the same issues as earlier; first, how does the attorney tell the
judge what is going on, so that the judge has enough information to rule on the
motion.

Second, will withdrawal prevent the perjury?

What will the client tell the new lawyer?

If the lawyer can tell the new lawyer nothing, won't the new
lawyer be duped into allowing the client to testify falsely?

And isn't that reason enough for denying the motion to


withdraw?

Assuming the motion is denied, Rule 3.3(a)(3) indicates that lawyer must then advise the
court of the false evidence.

In a civil case, or in dealing with a non-client witness, as Notes 6, 7, 11


following Bartee indicate, the lawyers obligation to reveal does not raise any
serious problems.

In criminal case, however, problems do arise.

The lawyers revelation is very likely to result in a mistrial of the case


and is not a result favored by courts.

Nonetheless, as Comment [7] to Rule 3.3 indicates, the lawyer must


disclose, even in a criminal trial.

Comment [12] then indicates that the Court must decide on what to do,
including the possibility of granting a mistrial.

This is where the free narrative option of People v. Bartee becomes


most attractive.

The client is already on stand.

The lawyer cant ask questions that allow the client to lie, other
than the free narrative question.

The judge does not want a mistrial.

The public interest is served by continuing the trialor at least


by not having a mistrial that forces all of the witnesses to be recalled.

The Judge can order the lawyer to continue and ask the free
narrative question, directing counsel not to use the testimony following that
question in close.

What rules and cases do not help with is the scenario when Judge is also the fact-finder
where there is no jury.

While Rule 3.3(a) appears to be applicable.

Do we want to require the lawyer to reveal to the fact finder?

And if we do, isnt a mistrial virtually automatic?

And suppose the lawyer says: Your honor, something has just
happened in the testimony, and despite consultation with my client, I need to reveal
something; and the judge says: Stop, dont tell methis trial is going to come to a
conclusion; What does the lawyer do at that point?

Under comments to the Rules, the lawyer cannot jump into


free narrative without court order or local rule.

But, the lawyer cannot continue questioning the client.

Would it be ineffective if the lawyer simply said at that point,


no further questions, and sat down?
Now look at Question (c); Judge allows withdrawal, mistries the case and appoints a new lawyer.
Original attorney happens to be in the courtroom when the retrial takes place and hears the false
testimony by the (former) client--If we assume that the judge does allow counsel to withdraw, it
seems reasonable that the defendant will not tell a new lawyer the whole truth. As a result, a

question arises as to the original lawyer's obligation when the lawyer, after being relieved, is at
the court and hears the former client testify falsely.

Here, the lawyer is not offering false evidence.

The information is still protected by Rule 1.6

There is no exception and the lawyer must keep quiet.

For this reason, an efficient judge should refuse the motion to withdraw--particularly if the
judge thinks the reason for it is proposed perjury--it is better to keep the lawyer who
knows about it in the case.
1
Question (d); Assume the case is over and the lawyer thought the client had told the truth on the
witness stand. The client comes back to the lawyer's office and confesses to lying while on the
stand about several material matters. There is an ongoing Civil Case over the same issues; what is
lawyer's obligation?

This is Rule 3.3(c) and Note 12 following Bartee.

The lawyer's obligation to reveal fraud upon the court, to which the lawyer was a party,
continues until "the conclusion of the proceeding;"

Look at Comment [13] to Rule 3.3

The proceeding has concluded when there is a final judgment on appeal or the
time for appeal has expired.

Thus, in this problem,

In Question (d)(i), there was an acquittal.

There is no possibility of appeal; as a result.

The proceeding has concluded.

The lawyer cannot reveal.

In Question (d)(ii), the parallel civil case; the client tells the same lie in the trial
here we do not have enough facts.

If the lawyer represented the client in the civil proceeding, and the
client lied, the proceeding has not concluded and the lawyer is obligated to take
remedial measures, which could include revealing.

If, in fact, another lawyer represented client in the civil proceeding.

First, the client did not lie in this proceeding while being
represented by this lawyer.

The criminal proceeding in which the lawyer represented the


client has concluded.

Therefore, the lawyer must keep silent.


A couple of other issues in this area.
1
Look at Note 13 following Bartee

These same rules apply to prosecutors.

If a witness lies, and the prosecutor knows it, the prosecutor must take remedial measures,
including alerting the fact finder to the lie.

As the Note indicates, this cannot be done so late in the proceeding that the defense has
no time to deal with it.
1
Lets assume the criminal case is over, the client is convicted and that a pre-sentence report has
been created; As a result, the Court has information that is clearly in error and the lawyer's
knowledge of that error is based upon client confidential information. The court asks the lawyer
a direct question of the lawyer about the accuracy of the court's information.

Here, the information is confidential.

The lawyer is not yet under testimonial compulsion, but there is a strong professional
obligation to tell the court the truth.

But, the information could even be privileged.

Generally, the lawyer should ask to be excused from answering the question.

If the court asks why, the lawyer can indicate that information necessary to
answer comes to the lawyer only by way of confidential (or privileged) communications.

If the court then rules that the information is not privileged and compels the
lawyer to answer, the lawyer is under the type of testimonial compulsion envisioned by
Rule 1.6 and MAY reveal.

If the lawyer believes the court is in error in ordering release of information, the
lawyer's only other option may be to refuse to reveal, thereby subjecting the lawyer to
possible sanction of contempt.

Although, if trial court agrees to give the lawyer time, it would be possible to
challenge the court's question by way of prohibition or mandamus, alleging that the trial
court does not have jurisdiction to ask the question.
3
Now, lets assume that when the client first comes to the lawyer, the client advises of past perjury
while represented by another--If the client tells the lawyer that the client committed perjury
while represented by another lawyer not affiliated with the lawyer being told, the lawyer being
told is under no obligation to tell anyone.

This information is information relating to the representation under Rule 1.6.

It is likely a communication within the meaning of that term for "privilege" purposes.

None of the exceptions in either rule 1.6 or 3.3 apply to permit or obligate the lawyer to
reveal or withdraw.
Section D--Protecting the Attorney: Self-Defense
A
Problem; Can Cicero disclose the fraud of the client either to a third party victim who has threatened
suit or to the local prosecutor investigating the possible fraud.
A
Consider in light of In re Boelter.
1
Facts
a
Boelter was hired by Withey to represent Withey in a proceeding with the IRS.
a
Withey told Boelter that he had previously concealed assets from the IRSinformation that
is clearly privileged.
a
Withey did not pay Boelters billed legal fees.
a
As a result, Boelter wrote a letter threatening suit and indicating that if there was a suit,
Boelter would be forced to reveal that client concealed assets from IRS.
a
Boelter also indicated that he had tape recorded conversations about these assets.
a
The client now believed that Boelter had a tape of their conversations.
a
In a subsequent letter, Boelter told the client to liquidate one of his undisclosed art works
to pay the legal fees.
a
The fee dispute settled.
a
Part of the settlement was that Boelter return tapes of conversations with the client;
Boelter first said the tapes did not exist and later claimed they were erased.
a
The client filed an ethics proceeding alleging that Boelter misused confidential
information.
1
Is there an exception that allows lawyer to reveal to recover fees.
a
Yes, and court cites to it.
a
Under Rule 1.6(b)(5), lawyer is permitted to reveal confidential information in a
controversy between lawyer and client; a fee dispute is just such a controversy.
1
What happens to Boelter in this case? Suspended 6 months.
1
If there is an exception allowing Boelter to reveal, what did he do wrong?
a
He told the client he was forced to reveal; and he was not required to revealhe was
permitted to do so.
a
He assumed that the exception authorizing revelation allowed him to threaten the client
and the court finds the exception cannot be so read.
a
He lied/misrepresented/defrauded the client into paying the bill.
A
Consider the case in light of the Notes following Boelter.

Meyerhofer v. Empire Fire & Marine, Note 1, was an early case in which the exception was used;
There, the lawyer relied on the exception when sued because of actions the lawyer claimed were
taken by his firm, not by him; The lawyer provided confidential information to both the SEC and
to the plaintiffs counsel; The Court of Appeals sustained use of the exception, therefore refusing
to disqualify the firm to whom the lawyer disclosed.
1
Note 1 points to Comment [10] to Rule 1.6 (and this is the August, 2003 version of Comment [10]
it is Comment [8] to the 2002 Rules and Comment [17] to the 2001 Rules) indicating that the
lawyer may use the exception when there is a mere assertion of the lawyers complicitythis
obviously follows from Meyerhofer.
1
In fact, Rule 1.6(b)(5) is quite specific in authorizing disclosure when:
a
Disclosure is needed in a controversy between lawyer and client.
a
Disclosure is needed to defend either a criminal or civil charge or claim based on conduct
in which the client was involved.
a
Disclosure is needed to respond to allegations in any action involving the lawyer's
representation of the client.
1
As a result, look at Note 2 following Boelter, allowing attorney to reveal in anticipation of
possible indictment so as to avoid stigma therefrom.
If there is any limit to the extent of disclosure, and Boelter suggests there is, it is the limit of necessity,
discussed beginning Note 3 following Boelter.
1
The permission authorized by Rule 1.6(b) limits disclosure to what the lawyer reasonably
believes necessary; what does this phrase really mean?
1
What would a reasonable lawyer believe necessary to disclose based on these facts?
1
In Note 3 following Boelter, you see the way the courts have reviewed that standard.
a
The courts have limited the exception to those disclosures that will provide significant
assistance to the lawyers defense.
a
Therefore, disclosures from prior or even current representations that are designed only to
show the impeachability of the former clients character are not likely to be necessary;
because such disclosures are likely to be little more than intimidation against the client.
a
Which, of course, brings us right back to Boelter and his threatening letters and again
shows us why his conduct was wrongful.
The response, then, to the problem is that the lawyer can disclose to either the bank or the prosecutor,
provided there is an actual allegation against the lawyer, and provided that the lawyer "reasonably
believes" disclosure is necessary in self-defense and provided that the lawyer does not disclose more
than is reasonably necessary to accomplish the purpose of protecting the attorney.

Chapter 5: Conflict of Interest


Introduction: Maritrans GP, Inc. v. Pepper, Hamilton & Scheetz.
A
Like confidentiality, conflict of interest has its roots both in the common law and in the ethics rules; this
case is designed to give an overview of both:
1
The common law obligations of attorney to the client, and
1
The common law remedies available for breach of that obligation.
A
Facts:
1
Maritrans ships oil products along the east and gulf coasts in competition with other similar
shipping companies.
1
Pepper Firms, partner Messina represented Maritrans in public offering and received over $1
million in fees.
a
Messina became familiar with Maritrans overall operations.
a
Including long-range planning and strategy goals.
a
Messina received a great deal of confidential information.
a
Including analysis of each of Maritrans major competitors.

The Pepper firm and Messina then agreed to represent several of Maritrans N.Y.
competitors in labor negotiations which were designed to reduce cost to and improve the
competitiveness of those competitors.
a
Maritrans objected, but the lawyers said this was a "business conflict," not a "legal
conflict;" although this term was not explained.
a
Ultimately, Pepper terminated representation of Maritrans and continued to represent its
N.Y. competitors--ultimately adding several more by hiring an attorney who took on a
couple of competitors after Pepper said they would not take any more cases.
a
Maritrans sued, seeking to enjoin the firm from representing the competitors.
1
The trial court issued the injunction, relying on then code of professional responsibility.
1
See Scope, paragraph [20] of Rules indicating that rules should not "give rise to a cause of action;"
and that violation of a Rule does not necessarily warrant other remedies such as
disqualification; This is apparently what the superior court said, indicating rules did not support
injunctive relief.
1
The Supreme Court, however, indicated that both courts were correct.
a
Rules could not be used to support injunctive relief.
a
But common law duties of attorney support the cause of action.
1
Common Law Duties all flow from legal position of attorney as "fiduciary" (or as we know, as
agent)
a
Involves more than mere business transaction--higher trust and confidence.
a
One of the duties is to respect confidences of the client.
a
Predates any of the professional codes.
a
Cases have long permitted damages by way of disgorgement or denial of fees for breaches
of fiduciary duties, even where those breaches also caused violation of one of the
professional codes.
a
That includes conflicts of interest.
1
A court can issue injunctive relief where there is no adequate remedy at law.
a
Must be decided case-by-case.
a
Here, more than enough to conclude that Maritrans position could be irreparably
compromised by its former lawyers, who knew what they knew, working for competition.
a
Damages would be difficult to prove; Why?

If the lawyer continues to represent multiple clients, and case concluded, how
could it be shown that the lawyer allowed representation of one of the entities to
impermissibly affect (taint is sometimes uses in cases) the other representation?

Obviously, the affected client could lose, but there can be lots of reasons for
losing, unrelated to conflict.

Who has burden of proving impermissible taint?

Obviously the client whose representation was affected.

How does that client go about securing proof of the taint?


1
Partially as a result of the difficulties of proving impermissible conflict, the conflict of interest
rules do not always require actual adversity before declaring something an impermissible
conflict.
1
Instead, the rules regulate those situations where an impermissible conflict could arise.
Defining conflict of interest; one of the best can be found in the definitions section at the end of the prior
disciplinary rules.
1
In fact, those rules did not refer to conflicting interests, because that phrase suggests the interests
must be adverse to cause a problem; rather, the disciplinary rules referred to "differing interests"
and made clear that included within the definition were any interests that could adversely affect
the neutral, independent judgment of the lawyer, be those interests

Conflicting.

Inconsistent.

Diverse.

Or other
1
In addition, the different interest could either be personal to the lawyer or between the clients.
1
Also, the differing interests could exist at the same time or in different times, as where the lawyer
once represented one client and then sought to represent a different client with a differing
interest.
1
Finally, the differing interests could be those of another member of the lawyer's firm, because the
rules presumed that information shared with one member of a law firm was shared by all
members of the firm.

The concept was that the client hires the firm, not a particular lawyer in it.

It did not matter that there are multiple offices of the same firm.
Rules of Professional Conduct differentiate the types of differing interests that lawyers often face.
1
Rule 1.7 sets forth the rule for current clients;
1
Rule 1.8 sets forth particular transactions, prohibiting some and allowing others only under
certain circumstances; the Rules refer to these as current clientsspecific roles; things like

The lawyer and the client in a business transaction.

The lawyer's use of the clients confidential information.

The lawyer's preparation of gift instrument where the lawyer or member of the lawyer's
family benefits.

The lawyer telling the clients story as a result of media rights agreement.

Prohibited financial assistance to the client.

Compensation from one other than the client.

Aggregate settlements between clients.

Prospective limit of liability for the lawyer's negligence.

Acquiring an interest in outcome of the client's cause.

The lawyer having a sexual relationship with the client.


1
Rule 1.9 limits appearance against former clients.
1
Rule 1.10 imputes many of the disqualifications of one lawyer to other lawyers in the same firm.
1
Rule 1.11 limits the impact of Rules 1.9 and 1.10 when it comes to former government lawyers.
1
Rule 1.12 limits the practice of former adjudicatory officers including not only judges but also
mediators and other third-party neutrals, such as arbitrators.
In each conflict or differing interest situation, some or all of the following issues will arise:
1
Disclosure or informed consent under Rule 1.0(e); There must be full disclosure to all affected
persons.

Of the conflict.

Of the consequences of the conflict.

Of the consequences of the lawyer's continuation in the matter.


1
There must be consent to the conflict by the affected persons.
1
A reasonable attorney must be able to conclude that the lawyer can represent all the differing
interests adequately.
1
Withdrawal.

If a reasonable lawyer could not represent all sides adequately, the lawyer must advise all
affected persons.

Of the conflict.

Of the consequences of the conflict.

Of the consequences of the lawyer's remaining in the matter.

The lawyer must then withdraw.

Sometimes from some of the clients.

More often from all of them.


Particularly in dealing with discipline and motions to have a particular lawyer or firm disqualified from
further participation in a case, these issues have often been tested under the standard of "the appearance

of impropriety," a standard rejected by the current rules as being too vague, but one which continues to
show up in cases.
A
And this is perhaps because the conflict of interest rules do come to us from the common law and long
predated any applicable Rulesas seen in Note 2 following Maritrans, discussing the 1883 New York
case.
Part A Conflicts of Interest of Multiple Clients
A
Multiple Clients in the Same Matter; Problem.
1
Several transactions in Questions (a) and (b).
a
Can you represent both buyer and seller in the same real estate transaction.
a
Can you represent two plaintiffs, one the driver, the other the passenger in the same car,
against a common driver defendant.
a
Can you represent both insurer and insured in defense of a claim by another driver.
a
For the non-litigation representation, consider in context of Iowa Supreme Court Board of
Professional Ethics and Conduct v. Wagner.

Facts.

Wagner agreed to represent one Oehl, who was trying to sell a


restaurant; If Wagner found a buyer and represented Oehl in the sale, Wagner
would get 10% of the gross selling price.

Is there anything wrong with this fee agreement?

No, provided 10% of gross selling price is reasonable within


the meaning of Rule 1.5.

The first proposed buyer was Childers, who had not owned a business
and did not have funds; Wagner did work for Childers in seeking to buy the
business and charged Childers for the work.

Wagner did indicate his representation of Oehl.

Wagner suggested the need for independent counsel.

But did not tell him why, did not mention the need for another
appraisal and did not mention that Wagner was to get 10% of the sales price.

Wagner prepared documents for Childers for the bankso Childers


could get a loan, and also prepared documents for the Childers/Oehl transaction.

Wagner sent a letter to both indicating there was a conflict and


indicating his belief he could represent both and also indicating that Wagner did
not help develop the purchase price.

Why is failure to help negotiate price important?

Because Wagner knows he is being paid on the basis of that


priceif he negotiated it, he would have even more interest in the matter.

Wagner did not tell either Oehl or Childers why he was


making this point.

Wagner and Oehl changed their fee agreement slightly, and while the
case is unclear, it may be because Wagner is also being paid by Childers.

Childers bought the restaurant and it failed.

Wagner then told both to obtain separate counsel.

Wagner found another buyeranother former clientand that failed.

Eventually, Wagner and his wife bought the restaurant and leased it to
another who ran it successfully.

Childers new lawyer found out about Wagners financial interest, sued
Wagner and the case settledfrom that, however, the ethics proceeding arose.

The court found that Wagner represented both Oehl as seller and Childers as
buyeris this correct.

Almost certainly both were relying on Wagners advice.

And it was reasonable, in a Togstad sense for them to do so.

The court finds multiple conflicts.


The conflict in representing both buyer and seller.

Here, the court quotes Wolfram.

That quote describes the areas in which differing interests can,


and often do arise.

The conflict in having an interest in the outcome meant that Wagner was
not truly disinterestedrather, he was looking out for his own interests.

Look at Rule 1.7(a).

Is this a case where the interests of one client were directly


adverse to those of another (1.7(a)(1))at least at the start?

Look at Comment [6]that comment discusses direct adversity


in the litigation situation and mentions economic adversity in unrelated
cases only at the end of the comment.

Comment [7] mentions transactions and suggests it would be


direct adversity where the lawyer represents a seller in one transaction
selling to another client of the lawyer (but who is not a client of the lawyer in
this transaction).

That would certainly suggest that representing both the buyer


and the seller in the same transaction is directly adversebut does not say so
directly.

Hold that thought and look at Rule 1.7(a)(2); in representing


both, was there a significant risk that the interests of either Oehl or Childers
would be materially limited by Wagners responsibilities to the other?

Or by Wagners own interests?

Therein lies the problem in this, and many other non-litigationtype transactions.

While the parties interests often start out similarly, so that you
dont have direct adversity under Rule 1.7(a)(1), either those interests
become adverse as the transaction progresses, OR, it is clear that the
interests of one client are going to materially limit what the lawyer can do
under Rule 1.7(a)(2).

And therein should be at least one lessoneven when it


appears that, in representing multiple clients, the interests of those clients
are not differing at the start of the representation, the lawyer has a duty to
assess those interests as the transaction progressesif the interests differ later
in the transaction, Rule 1.7(a) still applies and the lawyer must do
something.
What did Wagner do to try to eliminate the conflict and what does the court say
about it?

Wagner tried to acquire consent though his disclosure letters, and this
is generally permissible under Rule 1.7(b).

The court found that disclosure was not enoughWagner did not point
out the need for independent counsel or the consequences of why Wagner did not
help work out the purchase pricenor the fact that Wagner was helped by a high
purchase price or a large down payment.

Here, look at Rule 1.0(e) and now decide what Wagner should have
said.
Lets assume that Wagner did provide an adequate disclosure under Rule 1.0(e);
would that be enough to allow Wagner to represent both under Rule 1.7? Look at Rule
1.7(b).

No, the consent would need to be provided in writingwhich Wagner


could have done by putting such a provision in the closing documents.

In addition, however, Wagner must have reasonably believed that he


could competently and adequately represent both interests.

What does that phrase really mean?

Would a reasonable lawyer under the same or similar


circumstances agree that she could do so?

Notes 1 and 2 following Wagner show similar cases, including one case
involving a lawyer who had limited the representation under Rule 1.2 to being a
mere scrivener, in which lawyers have been held accountable for representing too
many parties in non-litigation transactions.

Note that New Jersey has made such representation


presumptively prohibitive.

Most states have not gone this far, although it is easy, as


Wagner demonstrates, to see why a state would do so.

Note that this problem applies to the common situation where


lawyer represents both the buyer of real estate and the bank providing the
funding, at least in terms of providing opinions on things like title.
The litigation side of this is shown in Shillens Case.
Facts

Husband and wife retained Shillen to represent them in an automobile


case in which the husband was driving and wife and husband were both injured in
a collision with another car; husband and wife each signed a separate contingent
fee agreement.

Both husband and wife claimed that accident was the fault of the other
driver, although Shillen received a copy of the police report that indicated that the
husband said at the scene that he was traveling 40 mph in a 30 mph zone.

The wifes case settled, but during negotiations with the other drivers
insurance company, that company indicated they had a witness who indicated that
husband was speedingmaybe going in excess of 65 mph.

The husband and wife then indicated that the wife wanted to pursue a
claim against the husband and his insurance company and eventually husband put
his acknowledgment of speeding into writing for use in that case; there was a letter
from husband indicating that Shillen represented both.

Eventually, Shillen was forced out of the case and both husband and
wife were separately represented, the case was tried and there were problems with
the husbands testimony that ultimately resulted in an attorney general
investigation for possible perjury; although that was not pursued, the AGs office
filed an ethics complaint against Shillen for his dual representation of husband and
wife during the period that they were on opposite sides.
Could Shillen represent both husband and wife at the outset?

The court seems to think he couldbecause he could have reasonably


concluded that the interests of husband and wife were not differing.

Both wanted to recover from other driver.

To Shillen, this looks like a 1.7(a)(2) conflict.


But, when he found out that the husband may have been at fault, the conflict
becomes one under Rule 1.7(a)(1) because the parties are directly adversewife should be
suing husband.

Here, the court discusses whether Shillen continued to represent


husband after the conflict developed and the facts clearly indicate that he did.

What about consent?:

Court discusses it,


But finds it is not allowed in this case; why?
Because, no reasonable lawyer could conclude that they could
adequately represent parties on opposite sides of the same case.

And that is the thrust of Note 1 following Shillen, discussing


Jedwabney.

And look at Comment [17] to Rule 1.7, along with Rule 1.7(b)
(3).

When the clients are on opposite sides of the same case, the
conflict is now NON-CONSENTABLE.

And it is non-consentable because no reasonable lawyer would


do it.

So, in answer to the problem, both representations are possible and look, at the
outset, like Rule 1.7(a)(2) situations in which the interests of the clients are NOT materially
adverse.

As we have seen, however, they can become adverse very quickly.

If the litigation situation becomes adverse, then the lawyer absolutely


cannot represent both.

If the non-litigation situation becomes adverse, the lawyer could


sometimes represent both, but both must consent after FULL disclosure.

Look at Note 3 following Shillen, can the client consent to future conflicts.

Comment [22] helps, by telling you it is possible.

But depends on the explanation.

In either Shillen or Wagner, do you want to draft the consent provision


that would have allowed the client to fully comprehend, in the Rule 1.0(e) sense the
conflict that would develop later?

Look at Note 4, following Shillen, in which the same client represents multiple
interests.

If those interests are different...

As where the client has personal interest as a parent and a fiduciary


interests as guardian, particularly where there are other beneficiaries.

The lawyer must treat those interests as though they were separate
clients.

This goes back to the material we talked back earlier under Identifying
the clientthe interests being represented.
And that is a good lead in to Question (b) of the problem, where the lawyer is appointed by the
insurance company to represent Jones, the insured.

Here, we must assume that the lawyer does not represent Jones for any other purpose; as
is normally the case in such circumstances.

This is, in part, Rogers v. Robson, Masters, Ryan et als,

Facts.

The lawyer represented a medical malpractice defendant as insurance


counsel, on appointment from the insurance carrier.

Who is the client?

Whose interests are being represented?

Those of insured; with the insurer providing the defense.

Compare Model Rule 5.4(c) which prohibits the lawyer from


allowing one who pays the lawyer to render legal services to another to
allow the payor to regulate the lawyer's professional judgment.

Isn't this what the court finds holding that attorney represents
the insured, not the carrier?

The insurance policy at issue allowed the insurance company to settle


the lawsuit without permission from the insured.

The insured did not want the case settled.

Does the court in this case find liability against the insurance company? NO, the
insurance company had the right to settle under the policy.

The lawyers are separately sued, presumably for malpractice or, more likely,
breach of fiduciary duty, for failing to advise the insured of the proposed settlement.

Is there an attorney- client relationshipYES, the court finds that the


attorney appointed by the insurance company represents the insured, not the
insurerand this is consistent with our findings back in dealing with the identity of
the clientit is the insured whose interests the lawyer is representing.

When the lawyer learned of insurer's impending desire to settle, this


should have been communicated to the lawyer's client--the insured.

Failing to do so breaches the duty.

Notes following discuss the triparte relationship.

Some courts hold that the lawyer represents both the insured and insurer.

But Note the 1950 ABA opinion that when a conflict arose, the lawyers duty is to
the insured.

That could never be true if the lawyer represented bothso this opinion, however
ill-stated must mean that the lawyer represents the insured, and is being paid by the
insurer;and as you see later in the Note, the Hawaii court calls this the modern view.

Notes also discuss the other kinds of conflicts that arise in this scenarioI dont
go over them but others can spend as much or as little time as they like looking at them.
Problem question (c) is very similar; the lawyer is asked to represent juvenile and is being paid by
the Father; The Father wants to settle; when the lawyer gets juvenile alone, the juvenile does not
want to settle.

Notes in section A(1)(d) help.

Look at the combination of Rules 5.4(c) and 1.8(f).

Under Rule 5.4(c), the lawyer may not allow the person or entity who pays the
lawyer to interfere with the lawyers independent professional judgment on behalf of the
client.

Rule 1.8(f) makes the same point more specifically requiring for the
representation to be proper.

Consent by the client after consultation.

An absence of interference.

Protection of confidential information.

The Note cases all show how these issues are reconciled by the courts and serve as good
reading material.

In answer to the problem, however,

The lawyer cannot allow the Father to interfere with the lawyers professional
judgment.

Thus, the lawyer must decide with the client, the juvenile, how the case will
proceedkeeping in mind what is right for the juvenile.

And the lawyer must preserve confidentialitythus, unless the lawyer considers
the father a client as well, the lawyer should not be discussing the client/juveniles case in
front of third partiesthe father.

How does the lawyer deal with the practical issues?

Presumably, very carefully.

The lawyer must discuss the fee arrangement with the father and make sure the
father understands the nature of the representation and the limitations on discussion
between father and the lawyer; the extent of consultation with the father will depend, in

part, on whether the father is a clientif so, there is likely the requirement of informed
consent under Rule 1.0.

Under Rule 1.8(f), the lawyer must also discuss these issues with the juvenile.
Problem Question (d) requires some moving ahead in the reading.

Problem Question (d) is covered by the material on Unrelated Representation of Current


Clients Adversary.

Look at Worldspan, L.P. v. Sabre Group Holdings, Inc..

Facts.

This is a tort case in which the defendants are represented by a law firm.

The same law firm represents the plaintiffs in unrelated tax matters.

There is only a distant relationship between the tort litigation and the
tax matters, although both seem to revolve around Worldspans computer airline
reservation system.

For purpose of the problem, however, please assume that any


confidential information learned by the law firm in representing plaintiffs in the tax
matter CANNOT now be used unfavorably by the lawyers in representing the
defendants.

What does the court do?

First, the court recites a paragraph from the ABA/BNA Lawyers


Manual on Professional Conducta book we have not seen before; it is like U.S. Law
Week and concerns issues of professional conduct and is an excellent resource on
these issues.

The paragraph cited by the court indicates that it is a conflict for one
lawyer to represent a client whose interests are adverse to those of another client,
even where there is no relationship between the two representations, unless the
clients consent and the lawyer reasonably believes that all the clients can be
adequately represented.

The basis for the rule is not the potential misuse of confidential
information, the traditional rationale for conflict of interest, but rather simple
loyalty.

Now look at Rule 1.7(a) defining concurrent conflict of interest.

First, those interests that are adverse interests or those that create a
reasonable likelihood of material limitation upon the lawyers ability to represent
the client.

Now, look at Comment [6] to Rule 1.7this sets out the reasons why it is
generally impermissible for a lawyer to represent a client suing another client of the
lawyer; the Comment mentions things like breach of loyaltythe client will feel
betrayedcould be either a material limitation or direct adversityas where the
lawyer is required to cross-examine the lawyers own client.

What about the note at end of Commentif two clients interests are only
economically different, then representation of competing economic interests in
unrelated litigation is not a problem.

Does Worldspan court mention this? No.

Did the court in Maritrans, at the beginning of this chapter,


mention this? No.

Take from that the notion that the comment is not talking
about either of these cases, both of which involve economic concepts, but
both of which have the lawyers representing differing legal interests in an
economic context.

Are these representations totally banned? No, as the court indicates, and as does
Rule 1.7(b), the representation is permitted with consent and a finding that a reasonable
lawyer could believe that the clients interests will not be adversely affected.
Was there consent in Worldspan?

Yes, there was a lengthy consent letter sent to the client before the
representation.

While the client objected to the representation when it arose, the client
did not object when the letter was sent and firm seeks to rely on this non-objection.
What does the court say?

If the firm wanted to be allowed to represent a different client in future


directly adverse litigation, the letter would need to expressly or at least strongly
impliedly say so.

What do rules say about that?

Rule 1.7(b)(4) allows consent, but MUST be in writing and


must be informed;

To be informed, requires, as we have already seen,


compliance with Rule 1.0(e).

The lawyer must provide adequate information about the


material risks of and reasonably available alternatives to the proposed
course of conduct.

In Worldspan, the advance letter does not do that.


The Notes give you this same material in some different factual contextsmost of
which are pretty self-explanatory.
Look at Note 5.

In the Worldspan situation, the law firm cannot generally avoid the
problem when it arises by dropping one of the clients.

The courts have not generally allowed the easier substantial


relationship test, that we have not gotten to, to govern this.
So what is the answer to the problem, Question (d)?.

Because you already represent Bartman, you cannot now appear for a
client when Bartman will be a material witness.

You will need to cross- examine Bartman about her ability to seean
issue critical to the outcome of the case.

Even if information about her eye problem is a matter of public record,


you learned it in a confidential relationship.

You will need to withdraw from representing Altman.

Now, go back to Positional Conflicts and look at the problem; You represent Smith Gun Company
defending a products liability case in which a plaintiff modified a hand gun to give it a hair trigger and
then shot himself; can you also represent an unrelated plaintiff who was injured by a different gun
manufactured by a different manufacturer when the plaintiff filed down the trigger mechanism.
1
First, are there confidences that you learned in representing Smith Gun that can be used against
Smith Gun? No.
1
Like the unrelated representation of the adversary, this case is more about loyalty than it is about
abusing a confidential relationship.
1
Before looking at Williams, look at the Note following the case.
a
Under prior rules, positional conflicts really only arose on appealprior to an appeal, only
facts were being argued.
a
Following a 1993 Advisory Opinion by the ABA, the issue became a trial one as well.
1
Williams v. State,
a
Facts.

The lawyer represents the client in death penalty case.


The client needs to argue that trial court was wrong when it gave great weight to jurys
10-2 vote in favor of death.
In another case, the lawyer argued that trial court was required to give great weight to
jurys 2-10 verdict against death.
The lawyer seeks to withdraw because of positional conflict.
Court looks at what was its equivalent to current Comment [24] to Rule 1.7.
Normally, it is permissible to argue inconsistent legal theories in different courts on behalf
of different clients.
BUT, if arguing one legal position will materially limit the lawyers representation of
another client, the lawyer cannot take these positions without informed consent; the body
of Comment [24] indicates factors to be considered.
The key appears to be: if the lawyer's legal position on behalf of one client will set
precedent that undermines the legal position of the other client, there is a conflict.
The court considers the loyalty issue.

All the clients have a right to the lawyers fidelity.

The lawyer must strike a balance between arguing any reasonable interpretation
of the law on behalf of one client with the other clients right to counsels loyalty.

In these cases, one of the lawyers clients will have their loyalty compromised,
simply because of the arguments.
Comment [24] indicates that consent will permit the two arguments.
Note that both the clients would be required to consent.
What would you include in a presentation to each client that would comply with Rule
1.0(e) on informed consent?

Section B--Personal Interests of Attorney.


A
Problem, question (a).
1
The conflict is not being in business with a client.
1
The conflict is both being in business with a client and being the attorney for the business
transaction.
a
The client thinks the lawyer will protect both the client and business.
a
The lawyer, however, will also protect the lawyer.
a
Even where the interests are differing, not just conflicting.
1
Consider in light of Monco v. Janus.
1
Facts.
a
Attorney and client each became 50% shareholders in a business.
a
Here we must assume that the client thinks the lawyer will be protecting the clients
interests.

The lawyer told business partner/client of need for independent lawyer.

The client cannot be allowed to make up own mind in situation where the client
is so reliant on the lawyer.

Look at what follows.

What was NOT disclosed and what it would mean to business


partner/client.

Eventually there is disclosure in writing, but even then, the disclosure does not
appear to meet the rigid requirements necessary to rebut the charge of undue influence by
the attorney, stated by the court:

Full and frank disclosure of all relevant information.

Adequate consideration.

The client had independent advice.

And note, court requires these to be proved by attorney by "clear and convincing
evidence;"
Consider how cases like this led to current provision of Rule 1.8(a) requiring.
Full disclosure, in writing.
Fair terms.
Advice, in writing to the client to seek independent lawyer.
The clients written consent that agrees to all of the above and indicating an
understanding that the lawyer is not representing the client.
There is not, however, a heightened evidentiary requirement.
Can the lawyer participate, if the client does not get independent advice?
Yes.
But, the lawyer must be able to prove the client was advised to seek independent
counsel.
Look at Notes:
Note 2 following Monco presents two issues.

The ongoing transactionthe lawyer becomes an officer or director of the


clients corporation and the client and the lawyer have ongoing interactions.

The Note asks if Rule 1.8(a) deals with that transaction after it has
started?

Assuming the answer is no, is there a rule that does deal with the
ongoing transaction?

Sure, Rule 1.7, which is the general rule dealing with current
client conflicts.

Under Rule 1.7(a)(2), the lawyer cannot continue to represent a


client if the interests of that client will be materially limited by the lawyers
personal interestsas would certainly be the case if the lawyer is in an
ongoing relationship with the client.

The second issue with which Note 2 deals is when the lawyer acquires
an ownership interest in something belonging to a client as a feeas where the
lawyer gets a piece of a corporation for representing the corporation.

As the Note indicates, Rule 1.8(a) deals with the lawyers


acquisition of the interest.

While Rule 1.7 deals with the lawyers personal interest after
the lawyer acquires the interest.
Note 3 following Monco deals with full disclosure;

What is required?

Enough information so it can reasonably be understood by the client.

How do you restate this test: Would a reasonable person, in this clients
position, understand? That necessarily includes the level of sophistication of the
client.
Note 4 following Monco contrasts the current ABA Rule with the prior Rule.

Look at pre-2002 Rule 1.8it only required that the client be given a
reasonable time to obtain independent advice.

If the client elected not to obtain that independent legal advice, it was
not the lawyers problem.

As a result, the Note case did not require the lawyer to tell the client of
the need to obtain advice.

The current Rule, however, explicitly requires the lawyer to advise the
client, in writing, of the need to obtain independent counsel; this should therefore
change the note decision.
The Notes on Other Self-Interest Conflicts are similar.

Note 1 simply shows you that the attorney can have conflicts in
litigation, as where the attorney is also a litigant.
Note 2 raises a provision of the Rules not previously included; Under
Rule 1.8(j), the lawyer is prohibited from having a sexual relationship with a client,
unless the sexual relationship preceded the attorney-client relationshipthis rule,
however, is different from the other attorney-client conflict provisions of Rule 1.8
here look at Rule 1.8(k).

Under Rule 1.8(k), any provision of Rule 1.8 that is applicable


to any member of the firm is also applicable to other members of the firm.

Thus, if I have a business transaction with my client, and am


therefore prohibited from representing that client because I did not advise
them in writing of the need to obtain independent counsel, then no lawyer in
my firm can represent that client.

However, if my partner is having a sexual relationship with a


client of the firm, and that sexual relationship did not precede the attorneyclient relationship, my partner cannot represent that client, but I can.

Problem Question (b) raises the issues in In re Crarywhen can a lawyer be a beneficiary of a clients
will/trust/or other gift?
1
Facts:
a
Crarys wife met 85 year old Mary Harris.
a
Harris asked to meet Crary to prepare a new Will.
a
A relationship developed and Harris became dependent on Crary and his wifethey took
her to appointments, made meals for her, etc.
a
During the relationship, Crary drafted a codicil to Harris Will whereby Crarys wife
would receive any funds Harris estate received from insurance and a grandfather clock.
a
Harris also wanted investment advice and Crary helped Harris make several investments
with Independent Order of Foresters (Crary did not tell Harris that Crary was an agent for
Foresters and received a commission from each sale).
a
Harris apparently loaned $3,500 to Crary.
a
The relationship soured and a disciplinary proceeding was eventually filed.
1
First, what about the purchase of annuities from Forestersand the loan from Harris to Crary?
a
This is a business transaction within the meaning of Rule 1.8(a).
a
Crary says Harris knew he was an agent because Crary filled out the forms for Forester.

Clearly not sufficient information within the meaning of Rule 1.0(e) and 1.8(a)(3).

No advice about independent counsel.

Nothing in writing.

These facts thus serve as a good review of what we previously covered.


1
What about Rule 1.8(b)It is impermissible for the lawyer to use client confidential information
for the lawyers personal interest without client consent; how does that happen here?

Crary knew Harris wanted to make investments.

Crary knew it because of client confidential communications.

Crary took that information and turned it to his own profit as an agent of Foresters.
1
What about gift to Crarys wifeRule 1.8(c).

The lawyer may not solicit a gift.

May not prepare an instrument that provides gift.

To either the lawyer or person related to the lawyer unless the donor is also a relative.

Rule defines the relatives that are affectedand it certainly includes Crarys wife.

What about consent?

Consent is NOT permitted for the Rule 1.8(c) violation, although informed consent
would alleviate the Rule 1.8(b) violation.

Here, look at Note 1 following Craryin the case there, the client insisted that the lawyer
draft the instrument giving the lawyer a gift and the lawyer continually declined;
Eventually, the lawyer went ahead and drafted the document giving the lawyer the gift.

Comments [6], [7], and [8] deal with this issue.

It is permissible to receive gifts from clients.

But, where the gift involves the drafting of legal instruments, someone other than
the lawyer MUST do it.

Can another the lawyer in the firm do it?

No.

This is Rule 1.8(k).

Who are the drafters thinking about when this rule is drafted? Wasserstromit is too easy
for the lawyer to overreach.
Now look at Problem, Question (c)when the client does not have any money, can the lawyer and the
client agree that the lawyer will take a fee based on sale of media rights to the clients story?
1
The answer is in Note 5 following Monco v. Janus.
1
When the lawyer agrees to a fee arrangement whereby the lawyer will be allowed to gain from
sale of media rights to the client's story, this is business transaction with the client.
1
When the agreement is entered into before the conclusion of the representation, the lawyer will
be torn.
a
Between representing the client zealously.
a
And making more money by making the case as sensational as possible.
1
Rules, however, do not treat this business transaction the same as Rule 1.8(a) business
transactions.
a
Because this business transaction could impact the way the lawyer represents the client,
particularly in court, and because the client will not likely know that,
a
Rule 1.8(d) prohibits the media rights transactionconsent cannot be used to permit this
transaction.
a
Note 5 does mention the one California case that opted to treat this transaction much like
other business transactions in Maxwell v. Superior Court, allowing the agreement to stand
if the other provisions of Rule 1.8(a) are metNOTEthis decision is unique to California.
One additional personal interest worth discussing at this point.
1
See Note 3 following Crarycan the lawyer be named as Executor or other Fiduciary in some
client document?
a
Comment [8] to Rule 1.8.
a
The lawyer can recommend the lawyer or other lawyers in the firm be fiduciary.
a
But that relationship creates a personal interest in the attorney that needs to be reconciled
with Rule 1.7 That is, will the lawyers role materially limit the lawyers ability to
represent the client.
And that provides a nice summary to Rule 1.8.
1
When the lawyer has a personal interest that differs from that of the client, Rule 1.7 generally
governs the issue; Thus, if the lawyers personal interest will materially limit the lawyers ability
to represent the client, the lawyer must take the precautionary action contained in that rule.
1
In some circumstances, the personal interest of the lawyer is deemed, by the Rules, to be a
material interest requiring additional comment.
a
Those are the situations covered by Rule 1.8.
a
When one of those situations exists, the material interest issue is manifest and the lawyer
must follow the provisions of the appropriate section of Rule 1.8.

Part C-- "Switching Sides": Successive Representation of Conflicting Interests


1
Problem--this problem will continue in Part D, Imputed Disqualification
a
The lawyer worked for firm that represented steel company in bridge collapse.

a
The case ends; the lawyer leaves firm and goes to another firm.
a
Another bridge collapse involving same steel company in another state.
a
The plaintiffs in second bridge collapse want to use the lawyer's new firm.
1
In part, this problem is addressed in Kanaga v. Gannett.
a
Facts:

A lawyer in a firm represented Dr. Kanaga in defense of a medical malpractice action


involving a hysterectomy.

That case is over.

In current case, Dr. K is suing newspaper for libel as a result of article paper published
about Dr. recommending that patient have hysterectomy later determined to be
unnecessary

The present case, then, has nothing to do with the earlier malpractice case--other than
involving Dr. Kanaga and a hysterectomy.
a
The lawyer now represents the newspaper defending the claim; Are there any problems?
1
What is the Test to be used:
a
The test is "Substantial Relationship:"
a
If the formerly represented client shows a prior attorney-client relationship AND there is a
substantial relationship between the subject matter of the present representation and the
subject matter of the prior representation, AND the lawyer involved had access to relevant
client confidences,
a
Then there is a conflict such that the current representation is improper.
a
Court in Kanaga spends time on the access of the lawyer to confidential information-why?

Because that's the loyalty obligation most likely to be breached.

But, in most cases, the client will say "I gave the lawyer confidential info" and the lawyer
will deny it.

How, then, can proof be developed?


1
Court resolves this dispute.

It is enough if the client "ought" to have discussed it with the lawyer.

If that exists, then the lawyer is presumed to have had access.

What is actually happening here?

Once the substantial relationship test is met, party alleging conflict of interest is relieved of
showing how the substantial relationship affected the representation of the party now
alleging conflict.

This because once the substantial relationship test is met, then, if the affected attorney had
access to confidential client information, an irrebuttable presumption arises that any
representation adverse to the former client would be tainted by the substantial
relationship.
5
In answering the court's first two questions,:

What is the nature and scope of the prior representation and

What is the scope of the present matter?

Courts generally look at:

Identity of parties or one of them.

Identity of issues.
6
In Kanaga, the issue is pretty straightforward; it was cases like this that prompted Model Rule 1.9.
Under Rule 1.9(a); if the lawyer formerly represented the client, the lawyer cannot now represent a new
client in a different matter IF:
1
There is a substantial relationship between the subjects of the two matters; AND
1
The new client's interests are materially adverse to those of former client; BUT
1
The former client can consent to the representation.

Consent is allowed because it is former client's confidential information that could be


used.
a
The client/former client is always the master of waiver of confidential or privileged
information.
a
The consent must be in writingthis is different from earlier rules.
This rule does not necessarily resolve our problem, however, because there is some question about the
lawyer's actual involvement in the Bramco matter at the other firmthe lawyer could have been an
Associate who had no access to confidential client information.
Rule 1.9(b) then comes into play.
1
Under Rule 1.9(b); If the lawyer was formerly in a firm that represented a client, the lawyer
cannot now represent a different client IF:
a
The new representation takes place in the same matter; OR
a
The new matter is substantially related to one in which the former firm's client was
represented by the former firm; AND
a
The interests of the new client are materially adverse to those of former firm's client; AND
a
The lawyer acquired material information protected by any of the Rules (but usually Rule
1.6).

And again, here a court would use a test like that in Kanaga, was the nature of the
lawyer's access to information such that the lawyer should have received the kind of
information protected by Rule 1.6.

That is probably the case in dealing with Bramco.

Note the importance of Wilson P. Abraham Constr. Co. v. Armco Steel.

The material information protected by Rule 1.6 might come from someone other than the
client.

Recall that rule speaks in terms of "information relating to representation" of a client-regardless of source.

Facts:

A lawyer with a Texas firm represented Whitlow Steel in a federal grand jury
investigation of part of the steel industry.

Eventually, antitrust violations were alleged against a number of steel makers.

Each of the steel makers was separately represented in the proceeding, but all of
the lawyers for all the steel makers worked cooperatively with each other
attempting to share information and plan a common strategy; the lawyer who
represented Whitlow was one of these cooperating lawyers.

Later a civil suit about the same facts was filed against all of the criminal
defendants, except Whitlow.

The lawyer who formerly represented Whitlow then wanted to move to one of
the firm that represented the plaintiff in the civil case.

Can the lawyer become associated with plaintiff's the lawyer in that civil suit?

The court does not resolve the case, but does instruct us:

There is no "presumption" that confidential information was shared--as


there would be if the former client was on the other side.

If, however, the lawyer was actually privy to confidential information


that might be used in this case, then the lawyer cannot participate.

Section D. Imputed Disqualification [Dont forget, 2002 Rules moved imputed disqualification as a result of
conduct within Rule 1.8 to Rule 1.8(k) and imputed the actions of one attorney in a firm to all others for all
transactions within that rule except Rule 1.8(j) having sex with the clientand this is mentioned in Note 4
following Sexson.]
A
The problem is a continuation of the Bramco problem started in the Substantial Relationship section.
A
Question (a)

If, during the pendency of the first Bramco case, and while the firm represented Bramco, a
potential client, injured in the bridge collapse, went to the Boston office of the law firm, could a
lawyer in that different office represent that other client.
1
Rule 1.10(a)--No the lawyer in a firm can represent a client if any one of the lawyers, practicing
alone, would be prohibited from doing so by virtue of Rules 1.7 or 1.9 (except for those personal
conflicts of attorney under Rule 1.7 that do not pose a significant conflict for other members of
the firm).
1
The rule embodies two concepts:
a
First, when a client goes to a firm, she hires the firm, not an individual lawyer.
a
Second, there is a presumption of shared confidences within the firm.
1
Consider in light of Matter of Sexson,
a
Facts.

Sexson had an office that he shared with other lawyers; Thompson and four
others.

The attorneys shared a secretary, common letterhead and had three common
phone lines; doors of the lawyers offices were routinely left unlocked.

Lawyer Thompson represented a Mr. Zimmerman on a personal injury claim that


was settled.

While that case was pending, Zimmerman hired another, wholly unrelated
attorney, to sue Mrs. Zimmerman for divorce.

Mrs. Zimmerman asked Sexson to represent her.

One of the financial issues in the divorce was distribution of the settlement of the
personal injury action--as a result, a judge issued a restraining order against Mr.
Zimmerman's negotiation of that check; it was served on Mr. Zimmerman at the office
when Zimmerman came to collect the money.
a
The issue is whether Sexson can represent Mrs. Zimmerman in the divorce case.

It should be clear that Mr. and Mrs. Zimmerman have differing interests in the
proceeds of the personal injury claim.

Thus, Lawyer Thompson, who represents Mr. Zimmerman on the PI claim, could
not also represent Mrs. Zimmerman in the divorce case.

Lawyer Sexson would be under the same disability if Sexson were in the same
firm as Thompson.

Under Comment [1] to Model Rule 1.10, the issue of whether there is a "firm" is a
factual onesame definition of firm is used in Rule 1.0(c).

Here, the lawyers shared stationary, phones, secretary; all of this is in


Note 3 following Sexson.

As court finds, Sexson and Thompson are in the same firm and, as a
result, Sexson cannot represent Mrs. Zimmerman.
1
The Problem, question (a) issue, then, is similar;:

Could the Chicago member of the firm, who represents Bramco, represent this plaintiff.

And the answer is a resounding no.


a
As a result, the Boston member of the firm cannot.
1
Look at Note 5 following Sexson; where there is imputed disqualification, informed consent can
eliminate the conflict.
Question (b).
1
What is attempted by this question is what is often referred to in ethics literature as screening;
a
Theory is that if we can prevent the lawyers from actually sharing confidences, that which
the rule seeks to protect remains protected.
a
As the cases demonstrate, whether the court will approve often depends on the kind of
firm that is in issue.
a
In re TChalla D. looks at these cases in public interest organizations.

Facts.

In this adoption proceeding, the childs mother seeks to disqualify the guardian
for the child.

The law guardian wants to take the child from the mother because of her alleged
on-going relationship with the father.

Both mother and father were arrested, apparently together, when mother was not
supposed to be with the father.

Law guardian is an attorney with the Juvenile Rights Division of the Legal Aid
Society (New Yorks version of Legal Services).

In the criminal proceeding, the mother was represented for a period of time by
the Criminal Defense Division of the Legal Aid Society.

When it learned of these facts, the Criminal Defense Division withdrew from
representing the mother.

Despite this, mother claims a conflict and seeks disqualification of law guardian.
The two divisions of the Legal Aid Society are like different departments of a law firm
and the court so finds.
Simultaneous Representation; Even if you were to assume that there was no relationship
between the criminal proceeding and the adoption proceeding (and there is a
relationship), the same lawyer who represents the mother in the criminal proceeding
could not appear against her in the adoption proceeding, as we have already studied.

The court here says if there is simultaneous representation, the court MAY
require disqualification.

This does not necessarily comport with Comment [6] to Rule 1.7 which indicates
that loyalty prevents the lawyer from representing another client adverse to the first
client.

Here, however, the court discusses the equities and finds it unfair for the law
guardian to be disqualified, even in the absence of consent by the mother.
Subsequent Representation; If we assume the Criminal Division got out of the case, as the
facts show they did, then the conflict is one under Rule 1.9 and there is clearly a
substantial relationship between the criminal proceeding, in which the mother was
arrested with the father, and the adoption proceeding in which the law guardian seeks to
fault the mother for being with the father.

The court goes through the presumption of disqualification similar to that in


Kanaga.

First, there is a prior attorney-client relationship.

Second, the former and current representation are adverse.

Third, the former and current representation are substantially related.

Instead of stopping there, however, the court goes further and looks at the
likelihood of a violation of the clients confidences.

The court also looks at the evidence of screening alleged by the Legal Aid Society
and uses that to show the unlikelihood that confidential information was abused.

You can look through all the comments to Rule 1.9 and you will NOT
find any that deal with screening in a private firm.

Because screening is not normally permitted for lawyers who leave


private firmswhich is what the court found the Legal Aid Society is.

Finally, the court looks at the mother and finds that she has waived objection to
any potential conflict both by failing to disclose the nature of the grand jury inquiry
and by failing to objectthis would not be consent under Rule 1.9, Comment [9]
making it clear that consent under this rule needs to be in writing.

Finally, the court finds no appearance of impropriety in allowing the attorney to


continue.

Appearance of impropriety is now a discredited standard that many


courts formerly used in these kinds of cases.

The standard is discredited because it simply cannot be defined and is,


therefore, vague.
Before we look at the notes, look at screening in a private firm caseLennartson v. AnokaHennepin Independent School District, DONT READ
Facts.

Lennartson retained a law firm to represent her in a sexual harassment action


against the school district; Fischer worked for this law firm and took one deposition
in the Lennartson case.

Law firm withdrew from representing Lennartson and then Fischer went to work
with the law firm representing the school district.

Fischer provided a conflicts report to the new firm and mentioned the
Lennartson case, but firm determined that her involvement did not involve learning
significant confidential information.

Law firm undertook screening, and it was pretty extensive and detailed in the
case.
Minnesota had a case in which the court adopted a three-part ruleand the rule is
dependent on the extent of confidential information learned by the departing lawyer.

Where the leaving lawyer did not acquire confidential informationthere is no


disqualification of the lawyer and no need to worry about imputed
disqualification;.

Where the lawyer learns some information, but the knowledge is not likely to be
significant in the later representationand the lawyer can be screened and notice is
provided to the other side, the lawyer is disqualified, but the other members of the
firm are not.

Where the lawyers knowledge is significant, because of prior representation,


both the lawyer and firm are disqualified because screening cannot cure the conflict
(this is a loyalty disqualification).
Following this case, however, the Minnesota Supreme Court adopted a new Rule 1.10;
under new rule, disqualification of one lawyer is imputed to others in a firm unless there
is no apparent risk of misuse of confidential information because any such information is
insignificant and because the lawyer is subjected to screening and because the prior client
has been notified.
Minnesota court then tries to reconcile the two rules, recognizes that the rule established
in case precedent is more lenient than its rule or professional conduct; the court then
enforces the conduct rule, rather than the precedent, recognizing that other states have
different versions of Rule 1.10
The court then finds that the fact that in this case, the lawyer had to review the entire file
before being able to take the deposition in the case and therefore had significant
information; therefore under either Minnesota test, the disqualification of Fischer would
be imputed to the entire firm.
Now you need to look at the Notes following Lennartson.

Note 1 following the case tells you that neither of the Minnesota positions is that
of the ABA.

When the Ethics, 2000 Report was finalized, the drafters included a screening
provision that would allow a disqualified lawyer to be screened from participation
(regardless of the extent of the disqualified lawyers knowledge), including
screened from participation in any fee from the matter, and provided notice is
provided to the affected client(s).

This proposal was rejected by the ABA House of Delegates in favor of the current
rule, which says nothing about screeningand based on what was proposed, the
absence of any language permitting screening is clearly a rejection of it in most
cases.

Note 1 also talks of the Restatement and how the Restatement essentially adopts
the Minnesota position.

Disallowing screening when the information learned by the lawyer is


significant.

But allowing screening in some other cases even without client consent.

With all these views, lets make sure we understand the ABA position: Under
Rule 1.10.

When one lawyer in a firm is disqualified from representing a client


because of Rule 1.7 or 1.9, except for a personal conflict of that one lawyer.

Then all lawyers in that firm are equally disqualified.

Also, under Rule 1.10(b), when a lawyer leaves a firm, the FIRM is not
prevented from representing new clients directly adverse to a client formerly
represented by the lawyer who left and not currently represented by the
firm, unless.

The new matter for the new client is substantially related to the matter
in which the former lawyer represented a different client, AND

Any lawyer still in the firm has Rule 1.6 or Rule 1.9(c) confidential
information that is material to the matter (THUS, if the new matter is
substantially related BUT no lawyer in the firm has any confidential
information because the former lawyer did all the work for the former client,
the old firm is not disqualified).
Given how strict Rule 1.10 is, how do you then get to the result in In re TChalla D.

Would the lawyer who represented the Mother in the grand jury proceeding be
permitted to be the law guardian? NO

If that lawyer is disqualified, and the court treats the Legal Aid Society as a
firm within the meaning of rule 1.10, how does the law guardian get to be in the
case?
Is it the public nature of the representation? The notion that because there is no money at
issue, the conflict rules should not be as strenuously applied?

Note 1 following TChalla rejects that very view and refuses to allow legal
services attorneys to represent both husband and wife in the same divorce case;
Although you can make the argument that Borden, the Note case, is different
because it involves a non-consentable Rule 1.7(b)(3) situation of direct adversity in
the same case.

Note 2 following TChalla, the court rejected imputed disqualification for a public
defenders office representing both a defendant and a witness in the same caseon
theory that the nature of the legal services agency was such that misuse of
confidential information was unlikelybut another case indicated that a lawyer in a
public defenders office could not represent a criminal defendant challenging the
effectiveness of a different public defender counsel.
In short, it is very difficult to reconcile TChalla under the current version of Rule 1.10; It
may be that the court is treating the lawyers more like government lawyers and applying
the rules applicable in Rule 1.11, our next sectionor it could be that the court is
reconsidering its position in Borden and making the public policy statement that when
dealing with those who might not be able to afford legal representation in the absence of
the Legal Aid Society, we will overlook some conflictsparticularly where there does not
appear to be any sharing of confidential information of the father.

Section E. The Former Government Attorney.


A
Introduction.
1
When the rules talk of former government lawyer, the rules refer to any former government
employment--from local prosecutor to assistant to the President.
1
The conflict of interest rules have traditionally treated such lawyers differently from lawyers in
private practice; as you can see in Rule 1.10(d), Rule 1.11 governs former government lawyers and
those lawyers are not subject to the provisions of Rule 1.10.
1
Further, as you can see from ABA Formal Opinion 97-409, Rules 1.9(a) and (b) do not apply to
former government lawyers.
a
Theory is that the government needs to be able to attract the best and brightest new
lawyers.
a
To do that, the government cannot unduly hamper those lawyers when they decide to
move into private practice.
a
With the result that the conflict of interest rules have traditionally permitted screening
when a lawyer leaves government service and moves into private practice.
a
Also with the result that the conflict of interest rules have not always used the Substantial
Relationship test when dealing with former government attorneys.
a
Rule 1.9(c) does apply to former government attorneys.

As a result, former government attorney cannot use information learned during


that representation to the disadvantage of the government, OR

Reveal information protected by Rule 1.6.


1
This is reflected in Rule 1.11

Rule 1.11(a) covers the government lawyer who leaves government service and moves into
private practice.

This lawyer can still represent a client, even if the lawyer did participate personally and
substantially.

If the government agency gives informed consent;

Rule 1.11(b) allows other members of the former government lawyers new firm to
represent the clients, even when the former government attorney is disqualifiedthis rule
differentiates the former government attorney rules from those affecting private
practitioners.

New law firm can represent this new client, however, with screening.

Provided the government is notified in a way that will enable the government to
determine if there is compliance with the Rule.

Rule 1.11(c) covers use of government information acquired while a government attorney.

It is broader than Rule 1.6 because it covers information regardless of whether it


was obtained within the scope of an attorney-client relationship.

"Confidential government information" is defined in the Rule.

Rule 1.11(d) deals with the lawyer who is now in government services.

This lawyer is bound by Rules 1.7 and 1.9.

The lawyer may not participate in government service in matter in which the
lawyer participated personally and substantially while in private practice unless
government agency gives informed consent in writing.

Note the rule does not say anything about consent by the former private
client.

But that is because the lawyer is covered by Rule 1.7 and Rule 1.9, both
of which could require consent from the former client, depending on the
circumstances.

The lawyer may not negotiate for post-government private employment with
person or entity with whom the lawyer is personally and substantially involved in a
matter.

Exempted from this last requirement are those who are judicial clerks negotiating
for employment under Rule 1.12.

This is discussed in Note 2 under Notes on the Former Government


Attorneys Current Firm.

Under Rule 1.12, law clerk must notify the judge of the positions for
which there is negotiation.

Rule 1.11(e) defines the word "matter" for the other provisions of Rule 1.11.

"Matter," as used in Rule 1.11 only means:

Any judicial or other proceeding; OR

Any application or request for a ruling; OR

Any contract, OR

Any claim, OR

Any controversy, OR

Any investigation, OR

Any charge or accusation, OR

Any arrest, OR

Any other Particular Matter, AND

As to each of the above matters Only if it involves the same specific


party or parties.

"Matter" may also be defined by the government agency involved and under Rule
1.11(c)(2), the FGA is obligated to heed that definition.

"Matter" does not include things like drafting rules or legislation.


In re Sofaer is an interesting example of Rule 1.11.

Facts.

Discipline is sought against Respondent, a former attorney for the State Department.

While he was at State, Pam Am Flight 103 exploded over Lockerbie, Scotland.

This sparked a massive hunt for perpetratorsRespondent received the daily State
Department briefing on this issue, which contained sensitive informationit is unclear if
this information included any information about Libya.

Respondent left State and went to work at a law firm; both the firm and the Respondent
then agreed to represent Libya in negotiating both with the U.S. and the victims families
to both turn over two Libyan citizens to stand trial and to compensate the victims
families.

In this disciplinary proceeding, Respondent says that what he did at State in connection
with the Lockerbie bombing is not the same matter as what he did for the Libyan
government in negotiations over reparations for that bombing.

First, if this case were considered under the substantial relationship test of Rule
1.9, would we even be discussing it? NO, it is obvious that there is a substantial
relationship between the two cases.

Then why does Sofaer even argue that it is not the same matter?

He argues that the matter he was involved in was the U.S.


governments efforts flowing from the bombing, not the efforts of families to be
compensated.

That investigation, he says, was largely criminal.

The respondent claims he only worked on the civil litigation while with
the private firm.

And you can see the court disagreeing defining matter in a way that
includes substantially related to a matter for which the government lawyer had
responsibility; But that does not help us with matter

Here the court says to use a practical approach which requires an


examination of the overlap between the facts of the two representations.

Here, the court finds that overlap.


Consider these teachings in light of our Problem,
1
Question (a); this is Rule 1.11(a).

Was Croft's involvement "substantial?" Probably, BUT

Is this the same "matter?"

Not under the definition in Rule 1.11(e)

There is not identity of parties.

Here, case differs from Sofaerin this case, the criminal proceeding is over and
this is a civil proceeding, even though it does involve similar facts.

In Sofaer, the lawyer could only participate.

If government gave informed consent.

And that is obviously missing from the facts.

If this is not the same matter, Croft is not precluded under Rule 1.11(a) from taking the
case.
1
Question (b) instead of talking to Croft, Williams talks to Crofts partner Day (the question could
also presumes that Croft is somehow disqualified in question (a))

If Croft is not disqualified, Day can accept the case without reservation.

Unlike Rule 1.9, however, Day can also accept the case, even if Croft is disqualified.

If Croft is screened from participation in the case, including participation in any


fee generated; AND

If the government agency is given notice.


3
Question (c) is also a Rule 1.11(a) situation.

Now there is identity of parties--Armstrong and the Division.

But there is no identity of claims.

Thus, same answer as Question (a), unless the Division has previously adopted a
definition of "matter" that would include both; if so, FGA would be obligated to heed that
definition.

In this regard, please look at Rule 1.11(a) which allows other law to provide
otherwise.

As a result, even if Rule 1.11(a) would allow a representation, state or federal


ethics in government statutes may prohibit the lawyer from some representations.

One such statute is in Note 4 following Sofaer.


4
Question (d) involves same parties and matter, but adds a new party.

As to this party, Croft has no personal and substantial involvement.

Given the other "identities," however, this may be too close to permit Croft to become
involved; this is most like Sofaer and the courts practical view of the facts could well
prohibit it.

At which point, Croft could only appear if he had informed consent from the
government.

But, if Croft did not participate, his private firm could participate if Croft were screened
and if the agency was informed.
1
Finally, look at Note 3 following Sofaer.

Some states still use the now repudiated Appearance of Impropriety standard.

These cases take a more rigid look at matter to determine the impact of the
representation on general public feelings about the lawyers and the judiciary.

Section F. Judicial Disqualification and Recusal.


A
Note that matters of Judicial Ethics are often a unique mixture of ethics "rules," like those for lawyers,
and statutes.
1
Particularly when related to federal judges, statutes have far more import in the regulatory
process.
1
This perhaps for at least two reasons.
a
First, because the Constitution itself, in Article 1, 8 authorizes Congress to create
tribunals inferior to the Supreme Court.

The federal courts have traditionally held that the power to create such courts is
also the power to prescribe the jurisdiction of such courts.

That would include the kinds of cases upon which a judge cannot sit.

Particularly when dealing with statutes on recusal of judges in certain kinds of


cases, these statutes seem very similar to "jurisdictional" statutes.
a
Second, because the judiciary itself recommended and played a role in preparing the
recusal provisions; as a result, the statutes themselves are akin to rules adopted by the
judiciary.
1
Additionally, federal judges can be removed only by impeachment.
1
As a result, the disciplinary aspects of the ethics rules for judges envision reference to federal
statutes relating to the judicial office.
A
Question (a), should Judge Whynne be disqualified from hearing Bolte's appeal because of:
1
Her public stance against "discriminatory prosecution?
1
Her relationship with "Big John" Thompson and his various political campaigns, because Angie
Doyle, an aid to Thompson, was a principle witness in Bolte's trial?
1
Because of prior ruling against Bolte in a state libel case?
A
The history of judicial disqualification provisions is well-covered in Liteky v. United States.
1
Facts.
a
Criminal defendants on trial for protest at a U.S. military base.
a
Defendants apparently wanted to use the trial as a way of expressing political opposition
to whatever was happening.
a
Defendants seek to overturn their conviction, from a trial held before the judge only,
because of alleged judicial bias and because the judge did not self-disqualify under 28
U.S.C. 455.
1
Recusal or disqualification non-existent at common law.
1
In United States, recusal provisions for interest have existed since the judiciary act of 1792.
1
This was expanded in 1821 to include any relationship to a party or proceeding that made it, in
the judge's opinion, improper for the judge to hear a case.
1
As codified, these provisions became 28 U.S.C. 455.
1
Beginning in 1911, additional statutes were adopted to prevent a judge from hearing a case in
which the judge did not have a "relationship," within the meaning of 455, but did have a bias for
or against one of the parties.
1
This later section ultimately became 28 U.S.C. 144.
a
As interpreted, this statute required that the cause for bias had to arise from something
outside the actual judicial proceedings.
a
As indicated, the fact that the judge finds the facts of the underlying case repulsive is not
enough to create bias.
1
At issue in Liteky is whether this "extrajudicial source" doctrine is as applicable to
disqualification or recusal under 455 as it is under 144.
1
Court holds that "extrajudicial source" doctrine is applicable to both 455 and 144-type claims,
a
But then negates the effect of the doctrine.
a
Court holds that doctrine only seeks to prevent bias or prejudice that would prevent the
judge from being impartial in the normal sense.
a
The actual source of that bias could come from either inside or outside the lawsuit.

1
1

15

In either event, recusal is necessary only where it shows something that would make a fair
judgment impossible.
Dissenters disagreetheir position is that 455 adopts an objective standard of whether
reasonable people might question the courts ability to decide the case; dissenters want to keep
this objective standard.
Note 1 following Liteky gives two examples of bias coming from within the proceedingin both,
the actions of the defendant have clearly gotten to the judge, who needs to be excused, because,
even in Justice Scalias terminology, fair judgment has become impossible.
Development of the Canons of judicial ethics relating to disqualification and recusal has been
along a similar path to the federal statutes.
a
The original Canons of Judicial Ethics of 1923 called for disqualification

When a near relative was a party, AND

When the judge's "personal interests" were involved. (Canons 13, 29).
a
Since then, the former Canons have been replaced by the Code of Judicial Conduct 3(E)
which is nearly identical to 28 U.S.C. 455, the federal statute.
What else can cause the kind of bias or prejudice that would activate Code 3(E) or 144/455?
What about a prior public stance? Consider Note 2 following Litekythat is Justice Rehnquist's
memorandum in Laird v. Tatum.
a
Justice Rehnquist had been in charge of a Justice Department division that defended a case
that was ultimately heard by the Supreme Court after Justice Rehnquist was appointed
and on which he voted to defeat the plaintiffs claims (the same position as the DOJ
advanced while he was in charge).
a
Justice Rehnquist referred to a portion of the case in Congressional testimony, but said that
testimony was based on a briefing memo from the DOJ
a
With regard to public comments, Justice Rehnquist distinguishes between public
comments

In general terms about a point of law on the one hand, and

About a specific case which involves that point of law on the other.

Rehnquist would find that the latter is a potentially disqualifying act while the
former is not.

In Laird, Justice Rehnquist was referring to the former statute. You can see from the Note
that some have suggested the result would be different under the current statute and Code
provision; Certainly, 455(b)(3) seems to speak directly to the issues; In addition, Code
Canon 3(E)(1)(b) seems to be applicable, although if Justice Rehnquist only supervised
lawyers, would it be so? If that is true, then he might not have actual knowledge, as that is
defined in 3(E)(1)(a).

Should it matter whether the opinion expressed in the public statement has already been
expressed by this judge, or by some other judge, in a decided case?

This is where the issue of "extrajudicial" bias may become relevant.

If the opinion were expressed in prior judicial opinions, the bias would not be
"extrajudicial"

Even under the more liberal standard in Liteky, whether the statement is a wellfounded conclusion from established authority, or is a statement of position in anticipation
of a specific ruling, will bear substantially on whether it reflects the requisite bias.
Note 3 following Liteky deals with public comment during a casethis is Code Canon 3(B)(9) and
you can see that Justice Scalia recused himself from the pledge of allegiance case; you can also
see recusal of the judge in the Microsoft case for repeated public comments (NOTE: the Code of
Conduct for United States Judges is different from the Code of Judicial Conductthats why the
numbers are different. The Microsoft case is another 3(B)(9) case.)

In dealing with United States Supreme Court Justices, please consider Note 4 following
Liteky.

In the Supreme Court, as opposed to other courts, if a judge is disqualified, there is no one
to replace the judgethe Court does not bring up somebody from the Court of Appealsas
a result, a 9 member court becomes an 8 member court and that can substantially change
the outcomea 4-4 tie leaves the lower court decision while a 5-4 decision can reverse the
lower court.
Notes 6 and 7 related to prior service as counsel and with prior non-judicial involvement with
parties and that, in our problem, goes to Doyles relationship to "Big John" Thompson.

"Big John" Thompson was the subject of Bolte's expose, out of which the present
prosecution arose.

Thompson and Whynne were former partners.

Whynne is a former Thompson campaign manager and may have had run-ins with Bolte,
"Gray Power," or both.

Are these potential conflicts covered directly by Canon 3(E)?

Canon 3(E)(1)(a) would disqualify if Judge Whynne had personal knowledge of


disputed facts; but that seems unlikely in the context of the presented facts.

Canon 3(E)(1)(b) would disqualify if "Big John" was a material witness, because
he was a former law partner: it is unlikely that he even knew any of these facts.

Isn't the real problem likely to be the appearance of partiality, under the general
standard of Canon 3(E)(1), created by Judge Whynne hearing a case involving her former
partner for whom she was campaign manager, where the case itself involves a political
campaign?

Particularly where Angie Doyle as a witness.

Doyle is employed in the Thompson organization, raising the same


political issues raised by Thompson generally.

If Whynne hired Doyle while Whynne was campaign manager, or if the


two were close as employees of the campaign, Whynne might give Doyle's
testimony more weight than would an objective observer; the reverse might be true
if Whynne and Doyle were enemies.

You can see the former counsel issue in Note 6 following Liteky; if a judge is simply a
former prosecutor or defense lawyer, disqualification is rarely justified; if, however, the
judge is the former prosecutor or defense counsel for this defendant, disqualification is
appropriate.

Note 7 following Liteky provides good examples of the kinds of extrajudicial involvement
that will disqualify a judge.

Here consider the actions of Justice Scalia in being a social friend of Vice
President Cheney when the Court was to consider a request to release information about
the energy task force chaired by Cheney as Vice President.

Justice Scalia wrote that Justices have engaged in social interaction with
Presidents, executive officers and members of Congress for over a century, including
poker games, dinner parties, and ski trips; These activities, according to Justice Scalia,
have not caused recusal of a Supreme Court justice; See Cheney v. United States Dist. Ct.
(Memorandum of Scalia, J) (3/18/2004), available on line at FindLaw,
http://news.findlaw.com/hdocs/docs/scotus/chny31804jsmem.pdf

Here, it may be worth asking if the result would be different if a judge of


a court below the United States Supreme Court had the same conflict as Justice
Scalia?

It strikes Professor Devine that the result would be different.


What about Judge Whynnes prior judicial involvement with Bolte?

Note 5 following Liteky indicates that generally, prior judicial involvement with a party-Judge Whynne ruled against Bolte while serving as a state judge--is rarely going to give
rise to disqualification.

17

Although specific conduct directed toward a party that shows specific bias or prejudice
might, as indicated in Sommers, Note 5.

And note the other family and prior involvement cases, Sanders v. Cockrell (Judge should
not sit in review of report drafted by judges wife, a United States Magistrate judge) and
State v. DAntonio (judge should not sit on case where judge had engaged in unsuccessful
plea negotiations with a party).

Presumably, these are Canon 3(E)(1) cases where reasonable people might
question the judges ability to be neutral.

Although they could also be Canon 3(E)(1)(a) cases in which the judge has a
personal bias (his wife drafted the underlying report) or prejudice (former prosecutor who
unsuccessfully negotiated with the defendant) cases.
Problem Question (a) also asks what process would be used to seek disqualification?
1
Consider United States v. Sykes.

Facts:

After pleading guilty to using a false social security number to obtain credit, defendant
appealed claiming judge should have been disqualified based on comments made by the
judge.

District court denied defendant's motion for recusal, something the court of appeal will
review de novo.

Court discusses law relative to motions to disqualify under 28 U.S.C. 144.

Disqualification is mandatory if the party seeking disqualification submits a


"timely" and "sufficient" affidavit.

"Timely"--at the earliest moment after the facts requiring disqualification are
known.

Sufficient"--must be facts pleaded with enough definiteness and particularity to


reasonably show that bias exists.

Court must accept truth of those factseven if it knows that the facts
are untrue.

Facts must come from source other than the judicial proceeding.

Here, however, look at the date of the case, 1993, a year before
the decision in Liteky.

Presumably, now, the facts could come from within the


proceeding.

Here, the court also discusses the fact that the bias alleged is more against
defendants lawyer than against the defendant.

What is normal remedy when the client believes the court is biased
against the clients counsel?.

Normally, the appropriate remedy is discharge by the client of their


attorney.

Does that mean that bias against counsel will NEVER justify recusal?

No, and the court says so; but in the affidavit it must be shown that the
bias against the attorney might extend to the client.

So why does court deny the motion.

Not timely; based on statements from June, but not filed until August.

Bias alleged against counsel without showing how those comments


might affect courts dealing with the client.

No affidavit of counsel showing the motion is made in good faith, a


requirement under 144.

Here look at Note 1 following Cooley.

Although motion filed under 144 mandates disqualification.

It does so only if the affidavit is sufficient in alleging a


disqualifying basis.

As a result, a judge can say: Even if the allegations in your


affidavit are true, and I am assuming they are, they do not state a sufficient
basis for disqualification;
2
United States v. Cooley,

Facts.

Abortion protesters were enjoined from blocking access to abortion facility.

After several protesters stormed the facility and were charged with crimes relating thereto,
judge who was to try the cases made public statements showing anger at the open
defiance of the court order.

Protesters sought to disqualify under 28 U.S.C. 455 for bias.

Standard for disqualification; would a reasonable person, knowing all the facts, harbor
doubts as to the impartiality of the judge?

Court does not make it clear, but process akin to that under 144 was used;
motion reviewed for.

Timeliness.

Sufficiency--including the pleading of facts that would show bias.

This because 455 does not have a remedy attachedit merely requires the judge
to self-disqualify; if the judge does not, the only motion is the one under 144.

Standard for disqualification is broader than bias based on facts of a particular


case.

Here, the judges appearance on Nightline showed frustration with


entire abortion protest movement.

The court finds that to be enough because it would convince "reasonable


person" of potential bias.
3
Problem does not ask, but some disqualification can be remittedthis is Notes 5 and 6 following
Cooley.

First, Note 5.

Under the Code of Judicial Conduct,

All grounds for disqualification can be waived by consent of the


parties EXCEPT personal bias or prejudice of the judge.

Grounds for disqualification must be placed on the record of the


proceeding.

Parties must consider whether to waive outside the presence of the


judge.

Under 28 U.S.C. 455(e),

ONLY grounds that can be remitted is the general ground contained in


455(a), impartiality might reasonably be questionedother grounds in 455 cannot
be waived.

Judge must explain basis for disqualification on the record.

Now, Note 6 following Cooley; if a party has grounds to disqualify a judge, they must be
asserted in a timely fashionin some states within a statutory period of timein other
states and in the federal court, timeliness is a judicially created doctrine, as discussed in
Liteky.
So, go back and look at our Problem, Question (a).
1
In a federal proceeding, the objection to the judge continuing must be made timely.
1
Must contain the grounds with sufficiency so that a reasonable person would be able to
determine that disqualifying grounds exist.
1
There must be an affidavit of counsel showing that the motion is made in good faith.

Here, the affidavit should probably concentrate on bias against the client, rather than against/or
in favor of counsel.
1
If disqualification is based on 455(a), the parties could waive it; if based on 455(b), the
grounds for disqualification cannot be waived.
1
As Note 3 following Cooley indicates, most of these motions are ruled upon by the judge who is
being challenged.
1
Whether any of that would change in state court normally depends on the state; As Note 2
indicates.

Some states have rules that allow automatic change of judge if the request is made in a
timely manner.

These rules prevent the political problems associated with making the motion to
disqualify directly to the affected judge.
A
Problem Question (b) asks whether your firm has an conflicts.
1
The only possible one is that your firm regularly hires Judge Whynnes former clerks.
1
Here, Rule 1.12 controls.

If a matter is pending while the clerk is working for the judge and the clerk works
personally and substantially on the matter, the clerk is prevented from representing a
client in that matter.

Other lawyers in the former clerks firm are not prevented, however, for the same reasons
as dealing with Former Government Attorneys.

Former clerk must be screened from participation, including sharing in any fee.

Written notice must be given to the tribunal.

NOTE here the symmetry between the requirements under this rule and those
required for the former government attorney under Rule 1.11; Isnt this because the judge
is a former government attorney?
3
As to Judge Whynnes clerks who are negotiating with the firm; Rule 1.12 indicates that the law
clerk can negotiate for post-clerk employment even with firms appearing in the court, but must
notify the judge.
Chapter 6: Particular Lawyer Roles and Responsibilities
Part A--Advocate: The Lawyer in Civil Litigation
A
Procedural and Disciplinary Sanctions for Frivolous Claims and Defenses--Problem.
1
Problem Questions (a) and (b)is there a duty to investigate the factual and legal soundness of
the claim before filing, beyond the statements of the client?
a
Rule 3.1 is a general statement forbidding lawyers from all claims that are frivolous.

Frivolous is not generally defined, BUT it is not frivolous to either.

Make a good faith claim for an extension, modification, or reversal of current law.

Defend a criminal case by forcing the prosecution to prove every element of an


offense.
a
Further definition is left, by Comment [1], to the "procedural and substantive" law; By
recognizing that the law is never "static," the general standard of Rule 3.1 ties the rule into
the provisions of ongoing developments in the law of procedure.
a
Rule 3.1 applies not only to the filing of papers, as do some of the procedural provisions,
but also to trial conduct, conduct on appeal, and the like.

It is broader than any of the individual procedural rules.

It could, therefore, be interpreted as a requirement for investigation.

Thus, look at Note 11 following Giangrasso;.

Discipline for violation is relatively rare.

Rather than a formal sanction, these violations are often handled in context, as
here; the court indicates that a violation of one of the rules has occurred.
2
In the modern world, sanction for frivolous claims is done under Federal Civil Rule 11 (and state
counterparts):

Rule 11 is set out by the court in Giangrasso; it applies to all papers (except discovery for
which there is a virtually identical provision of Federal Civil Rule 26) filed in the U.S.
District Courts; and as to each such paper.
There must be a signature--if none, the paper could be dismissed.
The signature of either a party (as in one acting pro se or a party signing with an attorney)
or an attorney constitutes a certification that the signor:

Has read the paper being filed.

Has made a reasonable inquiry into the facts and law and, as a result.

Believes the paper is well grounded in fact.

Believes the paper is warranted under existing law--or a good faith


argument for extension, modification or reversal of existing law.

Is not interposing the paper for an improper purpose.

Like unnecessary delay.

Harassment.

Needless cost of litigation.


If any of these provisions are violated, there is a violation of the rule.

Thus, consider the defense lawyer in our problem.

That lawyer denied that Dr. Stevens was at Midtown, denied that
Stevens treated Lincoln and denied that another doctor read the X-rays.

None of these allegations have any basis in fact and should not be
tolerated under Fed. R. Civ. P. 11.
Consider in light of Giangrasso v. Kittatinny Regional High School.

Facts.

The plaintiff filed an action against the School Board and


others alleging a deprivation of civil rights for suspending the student in the
absence of proper process and for conspiracy in attempting to get the
students mother to enroll the student in a school for the emotionally
disturbed.

The defendants moved for summary judgment and for Rule 11


Sanctions alleging in documents unopposed by the plaintiff that the student
committed an act, received two hearings and was not transferred to another
school.

The court grants motion for Summary Judgment and considers


Rule 11 Sanctions.

First, NOTE that sanctions were ordered prior to the 1993 amendments
to Rule 11 that allow a party to withdraw their paper often without penalty.

But, as Note 2 following the case indicates, the Safe Harbor rule
does not apply when the judge acts on their own to sanction a person.

Thus, even with Safe Harbor, withdrawal of the paper by


counsel will not prevent the judge from still initiating Rule 11 proceedings.

The bulk of the case discusses the violations and it should be clear that
the lawyer should have known the law because he previously had handled similar
cases; that the lawyer had an opportunity to comply with court orders and did not;
that the lawyer really refused to defend the Rule 11 proceeding; and likely had
some sort of vendetta against the School Board.

So what about sanctions.

Under Rule 11, sanctions are NOT normally used to


compensate the other party.

Sanctions are used for deterrence only.

Here, the attorney had previously been fined $5,000 for a


similar violation.

This time, there is a recommendation for a sanction of

$100,000.

There is a further recommendation that this money be paid to


the defendants, rather than into court as is customary. Under Fed. R. Civ. P.
11(c)(2), sanctions can only be awarded to a party when needed for effective
deterrence.

Rule 11 allows such additional sanctions as the court may find


necessary; here, the recommendation is for a permanent injunction against
the lawyer filing a lawsuit involving the High School defendant.

The Notes following Giangrasso take you through the provisions of Rule 11.

Note 3would Rule 11 prevent the lawyers in cases like MacPherson v.


Buick from bringing a claim that had previously been rejected? Probably not, but
note the difficulty of defining frivolous in these contexts.

Note 4 helps here as well.

First, the standard should be an objective onewould a


reasonable lawyer be willing to bring the claim?

Second, while there is no requirement to do so, claims like


those in MacPherson can be identified in the pleadings as being arguments
for change, extension, modification of existing lawand any claim so
identified should be viewed more tolerantly.

Note 5how much can the lawyer rely on the client?

The lawyer can rely on the statement of the client to the extent
reliance is reasonable under the circumstances.

Generally, the lawyer should seek independent verification of


the clients facts.

But, if no other sources of information are available OR it is an


emergency, can rely.

In those situations, however, there is an ongoing duty to assess


the factsthus if the lawyer later learns that the clients facts were not true,
must do something.

Similar rule holds true for trusting what a forwarding lawyer


says;reasonablenesswould a reasonable lawyer believe these facts?
The answer to problem Question (c) is also contained in our previous discussion; but what other
remedies besides discipline and procedural sanctions might be available?

What about Civil Liability, Prewitt v. Sexton.

Facts.

The client comes to the lawyer claiming that child has been improperly
removed by local authorities.

The lawyer and the client both attempt to find some judicial document
authorizing removal of the children, but are thwarted by apparent bureaucratic red
tape at every turn.

The lawyer then files suit.

At that time, the county attorney turns over the proper document.

The lawyer dismisses suit.

County workers named in suit file action against the lawyer

Nature of the action is unclear.

Abuse of process??

Malicious prosecution??

Intentional inflection of mental distress??

The court holds that any of these actions would appear to fall under
Restatement of Torts (2d) "wrongful use of civil proceedings" a tort that has at least
two elements:

Lack of probable cause.

Improper purpose.

In these circumstances, even though the lawyer filed suit to get


somebody to turn over the correct piece of paper to him, that does not create a tort.

Enough probable cause, given fact that nobody in authority


would answer questions.

No "improper purpose" which the court equates with "malice;"

Consider Notes 1 and 2 following Prewitt; they demonstrate the difficulty in using the civil
suit for damages.

The cases in that Note show that the improper purpose standard is difficult to
meet and

Certainly in the context of Spencer v. Burglass, upon which the problem is based,
the improper purpose standard could not be met and that would prevent a civil damage
action; in the action, however, Rule 11 would almost certainly justify procedural sanctions.

Note 2 tells you that even if further inquiry is required, merely relying on the
clients story, even though insufficient for procedural purposes under Rule 11, is likely
enough to avoid the action for damages.

Note 3 following Prewitt is largely historical.

Prior Rules of Professional Conduct explicitly prohibited a lawyer from


threatening criminal prosecution merely to gain an advantage in a civil action.

In many cases, however, a client might need to threaten criminal action


particularly where the activities supporting the civil action also constitute a crime.

What was really wrong with threatening criminal conduct was when it was being
used to extort money out of the defendant.

And extortionate kind of conduct is already covered in the Rules mentioned in


the Note.

Note 4 follows up on this concept and asks if it is improper to threaten the filing of a
disciplinary action to gain an advantage in a civil suit.

On one hand, an ABA opinion indicated that the same principles applythere is
no specific rule, but if the conduct is designed to extort, it is covered by other rules.

On the other hand, however, lawyers are under the affirmative obligation to turn
in other lawyers so there should rarely be such a threat!
Pretrial Delay and Discovery and Disclosure Abuse, ProblemThis is one of the few problems where the
answers come almost directly from the supporting Notes.
1
For this problem, students will likely need a copy of Fed. R. Civ. P. 26 and 37.
2
Problem, Question (a)Is it necessary for defendant to disclose documents that hurt the
defendant.

Fed. R. Civ. P. 26(a) provides for mandatory disclosure of certain items by all parties.

The names and addresses of all persons known by a party who are likely to have
discoverable information that might be used to help the disclosing partys claim or
defense (unless the person would be used solely for impeachment).

A copy or description of all documents that might be used to help the disclosing partys
claim or defense (unless the document would be used solely for impeachment).

A computation of any damages sought by the disclosing party.

A copy of any insurance agreements that might be applicable.

There is thus nothing in the rule that suggests that the defendant in our problem has a
duty to disclose documents that hurt the defendants case or that help the plaintiffs case
these documents will not aid the defendant.

What about under the Rules of Professional Conduct.


Here, look at Rule 3.1, which is applicable not only to pleadings, but to all
papersbut there is nothing in that rule that requires disclosure of these documents.

Then, look at Rule 3.4 which details fairness to opposing partiesand again,
there is nothing in this rule that requires voluntary disclosure of these documents at this
time.
Problem Question (b)interrogatories ask for any information about theophylline; defendant
objects for overbreadth and then indicates that all documents will be produced at the office of the
defense counselbut only those documents relating to Somophyllin Oral Liquid, the drug that
actually killed the plaintiffs decedent.

Here, look at Note 1 following the Problem and Questions; Fisons stands for the notion
that the normal adversary system does not allow a truly adversary discovery process.

Even in most adversarial of cases, rules still require discoverylawyers often have
difficulty separating the adverse nature of the proceeding from our discovery
requirements.

When, however, should the court become involved?

While, under Fed. R. Civ. P. 26(f), the parties must engage in a discovery
conference, few judges want to get involved in unnecessary discovery disputes.

As a result, the overwhelming number of cases that deal with the issue suggest
that it is only a systematic pattern of abuse that will justify sanctions.

Lets look at the rules on the issues in this problem.

Fed. R. Civ. P. 26(b) deals with discovery relevance and makes relevant, for
discovery purposes, any matter, not privileged that is reasonably calculated to lead to
discovery of evidence.

Are any documents discussing theophylline likely to lead to discovery


of evidence?

Of course, theophylline is the primary ingredient of the medication that


allegedly caused the death of the plaintiffs decedent.

Fed. R. Civ. P. 26(c); if the defendant wanted to limit the discovery to information
about the particular medicine, not theophylline, it could do so NOT by simply objecting
that the inquiry was overly broad; the defendant needed to seek a protective order.

Note 1 following the problem.

This is essentially what the court in Fisons found.

Here, look at Fed. R. Civ. P. 37(a)(3) indicating that an evasive answer is


viewed as a non-answer.

So, should the defendant be sanctioned in Question (b); Look at Fed. R. Civ. P. 37.

If a party does not get answer, the party can move to compel and for
sanctions.

BUT, the motion must include a certification that the party seeking
discovery has made a good faith effort to resolve the disputethere is no evidence
that has been done here.

Again, the courts do not want to become embroiled in a discovery


dispute.

Similarly, in Fed. R. Civ. P. 37(a)(2)(B), if there was a request for


documents, there can be no sanction until a motion is filed that includes a
certification that there has been an attempt to resolve the dispute.

Finally, against whom can sanctions issue.

First, look at Fed. R. Civ. P. 26(g)that rule requires that all


responses to requests be signed and indicates that the signature constitutes a
certification very similar to those in Fed. R. Civ. P. 11Failure to sign, or
certifying falsely can subject the attorneys, the parties, or both, to sanctions.

Then, look at Fed. R. Civ. P. 37(a)(4) indicating that sanctions


can issue against a party, an attorney advising the conduct, or both.

In looking at Question (b), while all of the actors COULD be sanctioned,


none of them are likely to be because.

There is no evidence of an attempt to resolve the matter among


the parties.

There is no evidence of a pattern of discovery abuse.

What about disciplinary sanctions? Here, Rule 3.1 would apply because
it is applicable to any kind of paper; not just pleadings.

Rule 3.1 is specifically complimented by Rule 3.4(a), (c), and


(d).

As we know, however, the likelihood of discipline for violation


of the discovery rules is really quite small.

Note 5 following this section talks about disciplineand note


that a Florida case disciplined a lawyer for discovery abuseBUT, if you read
the case, you will find that the lawyer represented a client in a medical
malpractice action; that the client stole some of the clients fathers (the
patient) files directly from the hospital; that the client told the lawyer of this
theft; that the lawyer did not tell the hospital that the lawyer had these files;
that the lawyer then used the hospitals failure to produce records as a way
of getting expert testimony for the plaintiff that the hospital failed in a duty
to the plaintiffthere was thus a pattern of abuse, lying and fraudulent
conduct that produced the violation of Rule 3.4(c).

If the disciplinary rules are invoked, then, unlike the


procedural rules, only attorneys, not parties, can be disciplined.
Now look at Problem, Questions (c) and (d).

Both are variations on the theme and can be considered by students on their own.

Generally, is there the kind of abuse of discovery that should be sanctioned or disciplined?
Or is this one-time conduct that a judge can simply address either by motion to compel or
some less drastic sanction?

Remember that the court really does not want to become deeply embroiled in discovery
disputes.

Here, look at Note 4 following the problems; the perception of lawyers and judges is that
discovery abuse is epidemic; the findings by some scholars, however, is that the perception
of abuse is not always correct.

Look also at Malautea v. Suzuki, in Note 4.

This is the typical discovery abuse casea pattern of abuse throughout the
proceeding leads to severe sanctions, including a default judgment against the client.

That pattern of abuse is simply not present yet in our Problem.


Problem, Question (e); any document that contains, in any way, discussion of theophyllin,
Somophyllin or their ingredients.

Obviously, this could include a lot of documents.

Note 7 following the Problem talks of overly broad discovery.

It is not sanctionable to ask overly broad questionsat least not once.

Fed R. Civ. P. 26(b) and (c) allow the court, upon orderwhich will require a
motion as a court is not likely to do this on its ownto limit the extent of discovery or to
issue a protective order.

Here, if the discovery were limited to active ingredients, the information would
be both relevant and unprivileged and therefore should be available for discovery; As a
result, after the company offers to provide information regarding active ingredients, and
the plaintiff refuses, the company should seek some sort of protective order.

The company cannot simply refuse to provide any information.


What about under Rules of Professional Conduct.
Rule 3.1 requires non-frivolous conduct.
Rule 3.2 obligates the lawyer to expedite litigation.
Rule 3.4 has a number of provisions all relating to fairness.
Unless asking overly broad questions is seen as needlessly prolonging the
proceeding under Rule 3.2, there appears to be no disciplinary prohibition against asking
overly broad questionsagain at least oncewhen a pattern is used, and it is clear that the
pattern of abuse breaches local rule, then provisions like Rule 3.4(c) come into play.
6
Problem, Question (f); Deposition issues.

Fed. R. Civ. P. 30(c).

Even when there is an objection in a deposition, the deposition goes forwardthe


witness answers the question subject to the objection.

If the testimony is offered at trial, the court will have to deal with the objection.

If the party attempts to read the deposition into the record, as under Fed. R. Civ. P. 32(a)
(2), which allows a partys deposition to be used for any purpose, the party will be out of
luck if the judge at trial sustains the objection that was made at the deposition.

Fed. R. Civ. P. 30(d)(1).

All objections at deposition are to be stated succinctly, in a non-argumentative


and non-suggestive mannerthus a lawyer cannot object and tell the witness how to
answer in the objection.

The lawyer can only instruct a witness not to answer to preserve a privilege, or to
follow some earlier court order in the matter, or to preserve the right to stop the
deposition under Fed. R. Civ. P. 30(d)(4).

Fed. R. Civ. P. 30(d)(4).

At any time during the deposition, a deponent or counsel can demand the
deposition be stopped alleging that the deposition is being conducted in bad faith or for
the purpose of harassment.

Thereafter, the party stopping the deposition should move for an order to stop
the deposition.

At that point, the court will rule on whether the deposition is being improperly
being conducted.

Fed. R. Civ. P. 37(a)(4) is applicable to award the winning party the costs of any hearing on
the motion.

Almost all of this is detailed in Note 8 following the Problem.

What about under the Rules of Professional Conduct; In addition to Rules already
mentioned in this problem, as Note 8 following the Problem indicates, Comment [5] to
Rule 3.5(d) indicates that conduct disrupting a tribunal does apply to depositions.
1
Finally, look at Note 9 following the problemon delay; Here, the court uses another statute, 28
U.S.C. 1927 to discipline a lawyer who delayed proceedings by needlessly attempting to remove
a case from state to federal court.
Communication with Participants in the Litigation, Problem, Downs slips and falls in Ashton Foods,
hurting his hip and files suit; Lester Minard is hired by Ashton to defend; Sue Thatcher represents the
plaintiff; Witnesses include a customer, a mid-level manager, a part time employee, a former employee
and an expert.
1
Question (a)can defendants lawyer tell the witnesses not to make themselves available?

Note 8 following Air Crash.

Rule 3.4(f) makes it impermissible for the lawyer to request most non-clients to refrain
from voluntarily giving relevant information to other parties.

Here.

Rule would permit the lawyer to make this suggestion to mid-level manager and
part-time employee under Rule 3.4(f)(1)

Note that Comment [4] cross references Rule 4.2.

That Rule, under Comment [7] discusses current and former


employees.

As we learned in Upjohn back in Chapter 4, a lawyer for an


organization represents the organization, but might have privileged
communications with many people within the organization.

Under Rule 4.2 and Comment [7], however, once the employee becomes
a former employee, they can be contacted by anyone, subject to respecting their
rights under Rule 4.4and this is true EVEN IF the former employee has damaging
information that could cause liability upon the company.

Rule 3.4 would not permit a request to refrain from discussing the case addressed
to the customer or the former employee.

None of the Rules speak directly to the expertbut Rule 3.4(f)(1) allows the lawyer
to request that agents of the client refrain from speaking with others.

When an expert is hired by a litigant, isnt that person an agent of the


client?

If there were any question about it, I would certainly make them one in
any agreement hiring them.
Question (b)Can Thatcher, the plaintiffs lawyer, interview each of the witnesses without
notifying Minard, the lawyer for defendant?

This is In re Air Crash.

Facts

Following an airline crash and lawsuit over it, the airline defendants
became suspicious when their pilots received questionnaires from the Safety
Research Corporation of America about icing practices.

When they prepared to file a motion to take limited discovery to see if


plaintiff or plaintiffs counsel had caused the questionnaires to be sent, plaintiffs
attorney admitted that he had indirectly commissioned the questionnaires and
admitted that there was a consulting relationship between those conducting the
survey and the plaintiffs counsel.

Defendants then moved for sanctions for violating Rule 4.2.

Plaintiffs counsel seeks to excuse the conduct by talking about


unintentional conduct, as well as making some other excuses.

First, look at Rule 4.2; under this rule, it is never proper to make an ex parte contact with a
person known by you to be represented by counsel in a matter, about that matter, unless
you have permission of the adverse lawyer; the rule does not speak in terms of intent.

Under Comment [3], the rule applies even when the represented person
makes contact with the lawyerthe lawyer must break off the contact.

In that regard, the rule, as Comment [1] indicates, protects against


POSSIBLE overreachingnot actual overreaching.

The court in Air Crash sees two reasons for Rule 4.2.

Like Comment [1], the rule preserves the integrity of the attorney-client
relationship by preventing interference with it.

It also protects the rules of evidence, because the rule recognizes that in
many cases, the lawyer could obtain damaging concessions.

What about in dealing with corporationswho can the lawyer contact within a corporate
party on the other side?

The court prohibits contact with all managers and with any person
whose act or omission might be imputed to the corporation;thus in our problem
both the mid-level manager and the part-time employee.

Rule 4.2, Comment [7] is not as restrictiveprohibiting conduct only


with those constituents of the organization who supervise or direct or who
regularly consult with counselOR those who have authority to bind the
organizationOR those whose conduct (acts or omissions) can be imputed to the
organization); Thus, in our problem, that is not likely to include the part-time
employee, depending on his duties and actions at the time; Suppose you cant tell if
you can talk to this person?

The court recognizes that it is sometimes difficult to determine who


those people areso what should a party do?

The court says when permission to contact is unclear, counsel


should either ask the other attorneyor obtain permission from the court.

Comment [6] to Rule 4.2 makes the identical pointcan always


ask the court.

Please note, however, that Comment [7], as discussed in Note 5


following Air Crash indicates, the no contact rule does not apply to former
employees.
In Air Crash, the court also finds a violation of Rule 4.3that rule prohibits a lawyer from
telling a person not represented by counsel that the lawyer is disinterested when, in fact,
the lawyer represents a client.

Here, the questionnaires suggested that they were coming from a


neutral or disinterested party.

To participate in such a questionnaire, the plaintiffs lawyer needed to


disclose their interest.

The Rule 4.3 violation is still strange, however, in light of the Rule 4.2
violationbecause there cannot be a violation of both.

Rule 4.2 presumes that the person contacted IS represented by


counsel in the matter.

Rule 4.3 presumes that the person contacted IS NOT


represented by counsel in the matter.

As a person cannot be both represented and not represented,


finding a violation of both is a little strange.

The only way there can be both a Rule 4.2 and a Rule 4.3
violation in the same case is if some of the questionnaires were sent to pilots
who were employed by one of the parties (a rule 4.2 violation) and some of
the questionnaires were sent to pilots from non-party airlines (a rule 4.3
violation)this is probably the case here.
The court gives little attention to the issue, but what about the fact that it was not the
lawyer, but those hired by the lawyer, who committed the violation of both Rule 4.2 and
Rule 4.3?

This, because as the court points out, Rule 8.4(a) prohibits the lawyer
from violating a rule, either personally or through the acts of another.

Thus, the lawyer cannot escape the conduct simply because another,
under the lawyers direction, did the actual questionnaire.
In our problem, then:
Thatcher can contact the customer.
Thatcher cannot contact the manager.

Thatcher can probably contact the part-time employee under the 2002 version of
Rule 4.2 and Comment [7], but would not be able to do so under the prior version
Thatcher should seek court guidance.

Thatcher can contact the former employee.

What about the expertthis is Note 9 following Air Crash.

There is nothing in Rule 4.2 that prevents an attorney from contacting


the other sides expert witnesses.

As the Note indicates, the states are about equally divided on whether
to allow such contact.

Under Fed. R. Civ. P. 26(a)(2), parties must designate their


experts and must provide the kind of report detailed in that rule.

Under Fed. R. Civ. P. 26(b)(4)(a), any party may depose any


such expert.

Some jurisdictions with rules similar to the pre-1993 version of


the Federal Rules have read the right to depose such experts restrictively
that is, these experts may not be contacted other than by deposition.

At the end of Note 9, note the provisions of federal law on contacting a


treating physician for the opposite sideparticularly one for the plaintiff.

Finally, look at Note 10, A Final Note ... and look at the Sanctions, in the courts decision.

While the lawyer might have cost associated with retrieving the outstanding
questionnaires.

There is no direct sanction against the lawyer; Why?

The courts do not want to get involved in pretrial discovery abuse.

Having said that, however, do not underestimate the importance of Rule


4.2the ex parte contact ruleit is regularly enforced by disciplinary sanctioneven
if not enforced by procedural one.
Communications with Judges.

Note distinction between D.R. 7-110(B) detailing a prohibition against ex parte


communication with a judge and Model Rule 3.5(a) and (b) which leave the matter to local
substantive law.

In re Ragatz, there are a series of ex parte contacts that both the lawyer and judge later seek
to cover up.

Facts.

Ragatz represented a client in a contested will contest action.

Motions were made to dismiss documents filed by Ragatz.

Ragatz consented to dismissal but contested the imposition of


costs against his client.

Case assigned to Judge Aulik who met Ragatz casually and


suggested that the judges clerk had preliminarily concluded that Ragatzs
action might have been frivolous.

Ragatz received a preliminary decision in the mail making a


finding that the suit was frivolousthis was sent by the judge.

Ragatz had a response prepared, sent it to judge marked


confidential and did not send a copy to the adversary; Ragatz told judge
that neither side would likely appeal whatever judges ruling was on
frivolousness issue.

Adversary counsel saw reference to the exchange between


Ragatz and Aulik in the courts file, and then demanded a copy of the Ragatz
letter.

At this point, both Ragatz and Aulik denied having copies;


talked to each other on the phone about it (without adversary counsel being

present) and then had a conference in which adversary counsel was present,
but at which Ragatz and Aulik urged the adversary attorney to settle the
matter.

The extent and tenor of the ex parte contact in both these instances can overshadow the
main issue.

Failure to notify opposing counsel was in itself a violation; without regard to what
followed.

And that is a violation for the lawyer, Rule 3.5(b) and for the Judge, Canon 3(B)(7).

As the court indicates, it does not matter that the lawyer was responding to the judge
these contacts are not permittedand the lawyer has a duty to break off the contact.

Note that some ex parte contact is permitted for judges, when the judge
needs the kind of assistance authorized by the Codethis is Note 2 following
Ragatz.

But must ultimately be disclosed.

Further, judge can have other ex parte contact.

Provided the requirements of the rule are met.

None of this is an issue in Ragatzthe contact was impermissible.

What do Notes say:

Note 1 talks about ex parte contacts on the merits;

This is a former standard.

Today, under Rule 3.5, unless the contact is allowed by law, if it


is about a case in which the judge and the lawyer are participating, it is not
permissible.

Note 3 recognizes that some proceedings are designed to be ex parte


such as a request for some emergency temporary relief.

While the contact is permitted.

Rule 3.3(d) specifically requires the lawyer to advise the judge


of all material facts;, even those that are adverse.

What about the Rule 1.6 confidentiality implications of


revealing even adverse facts?

Neither Rule 3.3(d), nor the comments to the rule, mention the
fact that the obligation overrides Rule 1.6 confidentiality.

This is a little odd because Rule 3.3(c) makes it clear that


sections (a) and (b) of Rule 3.3 overrule Rule 1.6 confidentiality.

BUT, there is simply no way of providing the kind of


information required in Rule 3.3(d) WITHOUT revealing information
relating to the representation.

Therefore, Rule 3.3(d) must overrule Rule 1.6 to make any


common sense.
Contact with Jurors.

There is no problem on contact with jurors.

Under Rule 3.5.

The lawyer may not seek to influence a juror by any improper means.

While a proceeding is pending, a lawyer may not have ex parte communication


with a juror unless authorized by law or court order.

After the proceeding is over, the lawyer may contact a juror only

If the contact is authorized by law.

The juror has not made it clear that the juror does not want to be
contacted.

The contact does not involve harassment.

As a result, the issue of juror contact is generally left to local substantive law.

State v. Washington.
Facts.

While the defendant and his attorney were not present in the court, the
court clerk and members of the district attorneys office made comments to the trial
jury.

The DA did not make any substantive comments, but did welcome the
jury, and tell them they were the backbone of the system.

The court finds that unilateral discussion with the jury automatically skews the
jury and therefore interferes with the impartiality of the jury.

Here, the court reversed the defendants conviction because of these welcoming
remarks by the district attorney.

Notes following Washington.

Note 1contact with members of the jury pool is generally prohibited and is
specifically forbidden in Restatement.

Note 2contact with current jurorsthe cases give you some of the ways in which
such contact arises and it is almost always forbidden; for example, Florida Bar v. Peterson
involved a luncheon encounter wherein the lawyer and a witness in the case allowed
themselves to be seated at the same table as two of the jurors in their caseand the case
had to be mistried.

Note 3post trial contactthis is where there is most disagreementas Note


indicates, the courts generally are not in favor of such contact, but rules do not forbid them
unless the juror indicates they do not want contact or contact involves coercion; Many
courts compromise by telling the jury as they are dismissed that they are not required to
meet with the lawyers and can tell the lawyers no;
Presentation of Evidence and Arguments.

ProblemQuestion (a)--Public Defender meets with witnessa former jailmate of the


clientand explains exactly what she needs by way of testimony. The witness indicates an
understanding after first telling the Public Defender a story that does not help the client
but then changes the testimony at trial to show the kind of threat that the public defender
explained she needed; Was this proper?

Look at Note 5 following the problem.

Some witness coaching is clearly permitted.

Coaching a witness to lie is clearly not permitted.

The prohibition comes from Rule 3.3(a)(3) which we saw in


dealing with confidentiality.

That rule forbids the lawyer from offering evidence the lawyer
knows is false.

Knows, under Rule 1.0(f) means actual knowledge that can


be inferred from circumstances;

As Note 5 indicates, most lawyers will stay clear of actually planting false testimonybut
what about in our Problem, Question (a).

Here, I would appoint two students to argue the pros and cons of the issue.

As Note 3 indicates, Rule 3.4 also prohibits the lawyer from falsifying evidence, although
that does not appear to be what happened in our problem.

If you changed the problem and the public defender said: No, that
cannot be what happened. Here is what you need to sayand then told the witness
exactly what to say.

Then you are closer to a Rule 3.4 violation.

Problem, Question (b)After the testimony, does the lawyer have an obligation.

Obviously, if you concluded that the testimony was not false, there is no duty.

Assuming that you conclude that the lawyer has offered false testimony through
this witness, then what.

Rule 3.3(a)(3) and 3.3(c) control.

If the lawyer has offered false testimony , even if the lawyer


later learns of falsity, the lawyer MUST take reasonable remedial measures,
including telling the tribunal.

Even if doing so would otherwise violate Rule 1.6.

Please review Note 8 on Witness FeesRule 3.4(b).

The lawyer is allowed to pay experts.

The lawyer is allowed to pay permissible expenses and statutory witness


fees to witnesses.

But not allowed to compensate fact witnesses and not allowed to pay
expert witnesses on a contingent fee.

Problem, Question (c)public defender found unpublished opinion by another district


judge in the district which was unfavorable and did not cite it. Prosecutor did not cite it
either and public defender does not cite it in a reply brief.

Note 6 following the problem mentions Rule 3.3(a)(2) (incorrectly cited in the
Note as 3.3(a)(1)), and deals with legal authority that imposes the duty of disclosure only
with respect to decisions "directly adverse to the position of the client" which are "in the
controlling jurisdiction". Controlling jurisdiction does not have further definition in
Comment [4] which simply repeats the rule.

Intuitively, it seems that a decision by another district judge in the same


district would qualify and.

An old ABA opinion, ABA Formal Opinion 280 (1949), agrees asking:
"Would a reasonable judge properly feel that a lawyer who advanced, as the law, a
proposition adverse to the undisclosed decision, was lacking in candor and
fairness?"

As Note 6 indicates, many courts continue to expect lawyers to cite more


than required in the rules, while other cases follow the controlling jurisdiction
rule.

In answer to the problem, the decision by a coequal district judge is not


controlling law in a precedential sense, but a lawyer does not cite this decision at the
lawyers peril.
Other Trial Conduct.

Problem, Question (a)Newton is defending Knighton in a dog bite case; one of the key
witnesses is Nhorak, who testifies as to dogs aggressiveness; Newton knows witness is
from Blavobia, where some citizens eat dog meat; she cross-examines the witness about
coming from a culture that despises dogs.

In re Vincentia long case that is a very easy read.

Facts.

You can see that Vincenti is charged with violating most of the rules that
can be violated in litigation, including Rules 3.1, 3.2, 3.3, 3.4, 3.5, 4.4, and 8.4, in the
trial of an action designed to terminate the parental rights of Vincentis client.

The judge had removed Vincenti as trial counsel because of his


obstructive conduct and his real role was supposed to be as an observer.

Vincenti made numerous comments alleging bias by the trial judge.

Vincenti asked compound questions that witnesses could not answer


and then used sarcasm to deride the witnesses who could not answer them.

Vincenti made numerous derogatory comments about adversary


counsel, including calling her stupid and a liar.

Vincenti belittled witnesses, calling them names.

The court found Vincentis conduct abominable


In dealing with our Problem, Question (a), the court found Vincenti violated Rule
3.4(e) by alluding to matters the lawyer does not reasonably believe are relevant.

Understand the standard, then, is what a reasonable lawyer would find


relevant.

Thus, is the fact that citizens of Blavobia sometimes eat dogs relevant in
any way to impeach the credibility of a witness to a vicious dog?

Note 2 following Vincenti calls this the good faith rule.

Lawyers must have a good faith basis for believing that the
question will obtain admissible facts.

Please see the end of the Note making it clear that it is also a
violation of this rule to ask a wildly irrelevant question and then, upon
objection, to say withdrawn;

What is required is a good faith basis for asking the


questionnot for obtaining an answer.

Will the lawyer in our problem be disciplined for asking the Blavobia
question?

Return to Vincentiwhat happened?

The lawyer was disbarred.

BUT, there was a litany of violations in the same proceeding.

AND, this is the third reported decision regarding trial


violations for this lawyer.

As a result, the likelihood that one lawyer will be disciplined for one
violation is pretty small.

Although that lawyer could face sanctions from the judge.


Problem, Question (b)should Newton have released the witness from the subpoena.
Look at Note 3 following Vincenti.
Under Rule 4.4(a), it is impermissible for lawyers to use tactics that have no
purpose other than to harass, embarrass, delay or burden a third party.
Forcing a witness to remain after their testimony is over certainly sounds like it
does that.
In Vincenti, the lawyer was disciplined for violating this rule by his haranguing
cross-examinations.
But again, there was a series of actions, not just one.
By the same token, if Newton had a legitimate belief she might recall this
witness, then the conduct was not designed solely for the purpose of placing a burden
on the witness.
Note that Rule 4.4(a) is often used to prohibit use of religious, sexual, or racial
bias by attorneys in selecting jurorsthe Note gives you the cases on the topic.
Problem, Question (c)Were Newtons comments in closing argument permissibleeven
assuming she believed all of them?
Rule 3.4(e), as indicated by Note 4 following Vincenti, prohibits the lawyer from
asserting their personal opinion in the matter.
That includes closing argument; therefore, the statement I believe he was lying
violates the rule, in addition to being irrelevant.
Vincenti offers other examples of the lawyer offering personal opinion about
evidenceand the lawyer, as footnote 10 indicates, was wrong.
Is the lawyer likely to be disciplined for one violationnot likelyprovided the
lawyer moves off the topic, as the judge in our problem instructed.
A couple of the other Notes:
Note 6, the lawyer as witness.

A lawyer cannot serve in a case when the lawyer is a necessary


witness, unless the testimony is about the value of legal services or the client will be
prejudiced by disqualifying the lawyer.

If the lawyer is a witness, other lawyers in the firm can appear in most
circumstances.

If, however, the lawyer who will be a witness then poses a


conflict within the meaning of Rule 1.7, then the entire firm is disqualified.

When would this occur? If the lawyer is going to testify


adversely to the interests of the client.
Note 9Mitigation of the Sanction by Apology.

In many cases, trial errors by counsel can be eliminated by apology by


the attorney.

This because many errors by counsel are one-time things.

Even in Vincenti, the court seemed to encourage apology, but Vincenti


was not interested.

This points to the fact that the lawyers reputation is what is usually
tested in these kinds of cases.

If the court believes the lawyer is truly apologetic, and if the lawyers
reputation is one of honesty, these violations are often excused.
Fair Trial and Free Pressthe issue in this section is how can our adversary system, and
particularly the lawyers who are its officers, assure parties a fair trial, in criminal cases
within the meaning of the 6th Amendment without offending the right of the press and
public to know what is going on in their court system within the meaning of the First
Amendment?
Consider the Bernabe-Riefkohl article.

First draws the fair trial v. free press conflict.

Then talks about history, making the point that the issue really surfaced
after President Kennedy was assassinated and the press, for a brief time,
considered the possibility of trial for Lee Harvey Oswald.

In reality, the trial of Jack Ruby for the killing of Oswald


produced equal problems.

Those issues can be seen in Rubenstein v. State, 407 S.W.2d 793,


799-801 (Tex. Crim. App. 1966)(McDonald, J., concurring), which shows the
circus-like atmosphere in the Ruby trial.

Even before Rubys case, however, the issue of fair trial/free


press surfaced in the first Trial of the Century, the trial of Bruno Richard
Hauptmann for the kidnap/murder of the child of aviator Charles
Lindbergh; the issues surrounding this trial continued well after the
execution of Hauptmann, and were kept alive by his widow; One of the
better exposes of the atmosphere in that case is Hauptmann v. Wilentz, 570
F.Supp. 351 (D.N.J. 1983), aff'd., p.c., 770 F.2d 1070 (3d Cir. 1985), cert.
denied, 474 U.S. 1103, 106 S.Ct. 887, 88 L.Ed.2d 922 (1986).

The article then mentions Sheppard v. Maxwell, which has traditionally


been the poster-case for trials that are unfair.

Sheppard involved a medical doctor whose wife was


bludgeoned to death in the couples bedroom; she was allegedly found by
the doctor and suspicion immediately fell on him.

It may be worthwhile to recount the kind of fair/trial free press


issues that took place in the trial.

Sheppard, and its "carnival atmosphere," was tried when the rules were
virtually nonexistent, thus the Court's statements serve as a baseline for early rules.

As the article notes, the Court held that none of the persons involved,
including the lawyers, the witnesses, the court staff and the accused, can frustrate
the court's function of providing a fair trial.
Again, as the article notes, the ABA response to Sheppard created some
confusion in whether the applicable Constitutional standard could permit the trial
court to curtail activity otherwise protected by the First Amendment

If that activity posed a "serious and imminent threat" to a fair


trial (More First Amendment protective); or

If that activity posed a "reasonable likelihood of interference


with a fair trial" (More Fair Trial protective).

In 1983, ABA adopted an intermediate standard of "substantial


likelihood of" material prejudice.
As the article notes, these standards were challenged in Gentile.

You see that in that case, a defense lawyer held a press


conference after his client was indicted.

The lawyer and his associates had studied the rules and
thought they had to respond to the negative publicity against their client
caused by the prosecutor.

The client was tried and acquittednothing said by the defense


lawyer was raised by the state or the judge in the trial.

After the case, the Bar charged the defense lawyer with
violating the then-existing Rule 3.6.
Note the split on the Court on the issues.
Justices Kennedy, Marshall, Blackmun, and Stevens (Justice OConnor
concurred with this) found no need to reach the Constitutional issue of whether the
"reasonable likelihood" standard was constitutionally permissible.

They held portions of Model Rule 3.6, as adopted by Nevada,


Constitutionally vague.

This because part of the rule permitted certain statements


"notwithstanding" another part of the rule which might have forbidden
them.

The rule thus became a trap and was, therefore, vague.


Chief Justice Rehnquist and Justices White, Scalia, and Souter did reach
the Constitutional issue.

Standard of rule did not have to be "clear and present danger,"

Can be less stringent

Reasonable likelihood of materially limitation upon a fair


trial is permissible.
Justice O'Connor joined both of these opinions; as a result

The Rule's standard ("reasonable likelihood") would appear to


be constitutional.

But the structure of the rule remained a trap that renders it


constitutionally vague.
Now, look at Rule 3.6.
Rule 3.6(a) adopts the substantial likelihood of materially prejudicing a
fair trial standard.
Then, under Rule 3.6(b), even if a statement might have a substantial
likelihood of materially prejudicing a fair trial, a lawyer can make the statements
detailed in this rule.
Also, under Rule 3.6(c), even if a statement might have a substantial
likelihood of materially prejudicing a fair trial, a lawyer can make the kind of

statement made in Gentilethat is a statement a reasonable lawyer would believe


necessary to protect the client from substantial, undue, prejudicial publicity not
initiated by the lawyer; Such statements, however, must be limited to information
necessary to mitigate the harm done by the prior statement.

Finally, Rule 3.6(d) imputes the provisions of Rule 3.6(a)-(c) to all


member of a lawyers firm.
Much of the discussion of the current rule is contained in Notes 2 and 4 following
the article.
With all this in mind, lets look at our Problem, a highly public criminal case in which the
following interactions with the press are at issue.
The son of the victim states publicly that he suspects the defendant.
The son also tells the press about interactions between his now deceased family
members and the defendant.
The judge bans the public from pre-trial proceedings and refuses to allow
television in the courtroom during the trial.
The prosecutor mentions that the defendant had juvenile trouble, but wont
discuss it.
The police officer mentions arresting the defendant three times for violent
offenses while the defendant was a juvenile.
After this, the defense lawyer gives an interview and alleges that the defendant is
innocent.
After defendant is convicted, the defense lawyer gives an interview and basically
admits what the client did.
Question (a)are any of the statements by the prosecutor permitted?

These statements are almost certainly impermissible.

Juvenile statements are almost certainly inadmissible; a jury has not yet
been selected; and a reasonable lawyer would believe inadmissible statements
about the defendants prior record will have an impact.

Further, look at Rule 3.8(f), as discussed in Note 5 following the article.

The prosecutor must exercise care to prevent those employed


by the prosecutor from making the kind of statements the prosecutor is
prevented from making.

Simply telling the officer to cross that bridge when you get to
it seems to be an abandonment of that responsibility.
Question (b)were the statements of the defense counsel proper?

The pretrial statements may be partially permissible under Rule 3.6(c).

There was negative publicityabout the juvenile record.

The lawyer is entitled to counter that.

Did these statements go beyond what was necessary to mitigate those


statements; Comment [7] does not help with any definition of this standardbut
these statements do seem to match up favorably with those in Gentile.

What about the post trial statements?

Because the trial is over, it would seem that the rule is


inapplicable.

But what about appeal and possible retrial?

The Comments do not address this issue.

Certainly, however, these statements are made from


information relating to the representation and are covered by Rule 1.6.

The statements are almost certainly not in the interest of the


lawyers client.

Thus, in the absence of consent, which is not likely present,


these statements should be prohibited.

To the extent the lawyer is trying to look good, there are


conflict of interest aspects to these statements.

Both the confidentiality and conflict of interest issues are


discussed in Note 3 following the article.
Question (c)What other problems.

First, there is nothing that prohibits the son from talking to the press
and indicating suspicion against one or more parties to the proceeding.

The judge is required to refrain from public comment other than about
the process of the court, as discussed in Note 8 following the article.

The provisions of Canon 3(B)(9) mirror the standard for


lawyers and apply to all those within the supervision of the court.

The court must require the same of all court personnel.

Note 6 talks about gag orders issued against participants.

Note 7 discusses the issue of cameras in the courtroom.

It is clear that the Courts are not required to allow cameras in


the courtroom.

It is also clear, however, that the courts are generally prohibited


from closing the courtroom to all.
The Lawyer in Criminal Litigation--As prosecutor in a university town, you have a coed
alleging that she was raped by a famous alumnus of the university at a post-basketball
game fraternity party where the coed admits to having consumed 6 beers. A lawyer friend
of yours and campaign manager of your campaign represents the famous alum and wants
you to not charge (although agreeing that his client will do hundreds of hours of
voluntary community service after admitting to having consensual sex with the coed);
Your detectives and assistant prosecutors are split on whether there is enough to charge
and/or convict.
Berger v. United States.

Facts: Defendant was convicted of conspiracy after being charged with


conspiracy to counterfeit notes.

The court finds the conduct of the U.S. attorney/prosecutor to be


beyond the bounds of propriety in several respects.

The court finds that the prosecutor is not an advocate in the traditional
sense, but a representative of the sovereign whose obligations are to govern
impartially and for all.

While the prosecutor is obligated to argue vigorously.

The prosecutor must strike fair, not foul blows.

Improper suggestions, insinuations and personal comments are not


permitted.

The jury assumes that the prosecutor will be fair.


Does Berger mean that prosecutors have a standard of conduct above that of
ordinary lawyers? Or how do you translate Berger into a standard of conduct?
Look at Rule 3.8 as we look at out problem
ProblemQuestion (a)Should you charge?

First, look at Note 9 following Berger.

Prosecutors are subject to all of the rules of other attorneys.

In addition, prosecutors have additional responsibilities under


Rule 3.8.

Under Rule 3.8(a) and (c), a prosecutor is only allowed to charge when
the prosecutor knows there is probable cause.

Note 2 following Berger.


Probable cause is not defined in Rule 3.8 and is not mentioned
in the Comments.

The Note tells you the term allows a prosecutor to charge only
if there is a belief that the defendant committed the crime and there is
evidence to support that.

Is that more or less than is needed for a civil lawyer to file a


non-frivolous pleading under Rule 11?

The sense is that it is a greater obligation but could you file a


civil pleading with no evidence to support the claim?

In our problem, is there enough to charge?

Of course, you have the statement of the victim.

Is there enough to convict?

Maybe notbut that is a political, not ethical problem, as Note 2


points out.

Question (c) assumes that you do charge and there is an acquittal and
that you are overheard in a bar following the case indicating that one of your
assistant prosecutors was right and that you were not sure about whether to charge,
but thought the jury should decidewill you be disciplined?

In a practical world, almost certainly not; if the case was strong


enough to go to the jury, there was certainly probable cause and you should
escape a disciplinary violation.

Will that prevent the defense from making a chargeNo, but


the reality is that a prosecutor who charges without probable cause does not
last long in the court housethe trial judges will know there is no probable
cause and may start dismissing cases at probable cause hearings.

A prosecutor with this kind of reputation simply cannot


survive.
Problem, Question (b)If you charge, do you have an obligation to reveal a
conversation you had with your neighbor, who seemed to have a fair amount of
informationmostly hearsayabout the case?

Note 4 following Berger mentions Brady v. Maryland, the case that


inspired Rule 3.8(d).

Brady imposed a duty to disclose exculpatory evidence

And did so as a matter of Constitutional due process.

Rule 3.8(d) goes further than Brady and requires the


prosecutor to disclose any information that tends to negative guilt;
Additionally, at the sentencing phase, the prosecutor is obligated to disclose
all unprivileged information mitigating sanctionPLEASE NOTEthe
prosecution can always seek to protect information from disclosure by a
protective order, although such an order is rarely granted.

The Comments do not define what we mean by tends to


negative guilt and, as the Note indicates, there is widespread disagreement
as to what the term means.

BUT, if you put this obligation together with the comments by


the Court in Berger, the obligation upon the state ought to be a very high
one.

I would disclose the conversation with the neighbor.


Lets look at some of the Notes.
Note 3 following Berger discusses trial conduct and provides some interesting
insight into one significant prosecutorial mistake in trialthat of asking O.J. Simpson to try

on the bloody glove found at the scenethe prosecutor did not take into consideration the
fact that the defendant might sandbag this demonstration, which became the lynchpin of
the states failure to obtain a conviction.
Note 5 following Berger discusses contacts between the prosecution and defense.

First, again, the prosecutor is like any other lawyer and cannot make
contact with persons represented on the other side without the other partys
lawyers permission under Rule 4.2.

Rule 3.8(b) and (c), however go further.

The prosecutor has an affirmative obligation to assure that the


defendant has been advised of their right to counsel and how to take advantage of
that right.

The prosecutor cannot seek from an unrepresented accused a waiver of


any important pretrial rights.
Note 6 following Berger mentions Rule 3.8(e) which places special restrictions on
prosecutors from issuing a subpoena against defense counselexcept under very limited
circumstancessuch a subpoena clearly interferes with the attorney-client relationship
between the defense attorney and defendant.
Note 8 talks about Closing Argument and warns prosecutors against comments
suggesting that the jury can draw an inference from the defendants failure to testify, or
refusal to answer questions asked by police; these refusals are constitutionally permissible
and the defendant cannot be faulted for exercising Constitutional rights.
Finally, Note 7 raises a federal preemption issue.

In 1980, a Department of Justice memo indicated that federal


prosecutors were not subject to state rules of professional conduct.

The argument was that such rules limited federal prosecutors


on how they could meet with witnesses.

Additionally, the justice department investigating wrongdoing


across several state lines argued that it could not possibly comply with the
variation in rules in the several states.

As the Note indicates, legislation was passed requiring federal


prosecutors to comply with state ethics rules.
Criminal Defense Attorney; ProblemThrown out of the prosecuting attorneys office, you
become a public defender and take on the case of an accused serial murderer who has, as
yet, been accused of only one murder (but murders stopped when your client was
arrested); the client admits the assault to you.
Lets start with Question (b)you believe the victims identification of your client,
but have received a report from the state indicating that she is occasionally delusionalcan
you vigorously cross-examine her in an attempt to throw suspicion on her identification?

First, suppose you want to contact the victim either personally or


through one of your investigatorscan you do so in the absence of the prosecutor?

As Note 7 following the Babcock essay indicates, Rule 4.2


would not prohibit you from doing so.

Rule 4.2 applies only to those who are represented in an action


and the prosecutor does not represent the victim of crime.

A victim could always tell you she does not want to talk to you.

BUT, the prosecutor is bound by both Rule 3.4(a) and Rule


3.4(f); Thus, the prosecutor cannot unlawfully prevent you from access to
evidence and the prosecutor cannot ask another (including the victimwitness) to not make themselves available.

But what about the cross-examination issue?

Here, the Lawry essay helps.

He talks about the ABA Standards for Criminal Justice, the


prosecution and defense functionsand you should look at them.

These rules have been adopted by the ABA but have not been
adopted by states.

As a result, they have much the same status as advisory


opinions.

They assist in our understanding of the law, but they are not
the law.

Lawry traces the development of what was originally ABA


Defense Function 7.6(b).

Originally, in the 1970's, this Rule told the lawyer not to misuse
cross-examination to discredit a witness the lawyer knew was telling the
truth.

By the 1980's, the language eliminating cross of a truthful


witness was eliminated because cross examining that witness might be
necessary; There was, however, a comment encouraging the lawyer not to
destroy the credibility of a truthful witness if that could be done while still
providing effective assistance of counsel.

By the 1990's, any attempt at aspiration to the lawyer was gone;


the lawyer is told to cross-examine without regard to the lawyers knowledge
or belief in the credibility of the witness. The commentary mentions that the
lawyer should not try to demean the witness generally, but appears to give
permission to do so if such questioning is beneficial to the client.
What do the Rules say?

Rule 3.1 generally prevents frivolous conduct by attorneys.


When, however, the criminal defense lawyer is forcing the prosecution to
prove its case, the lawyer is exempt from the frivolous ban.

For the prosecution to prove its case requires that the jury
believe the credibility of the states witnesses; to believe that credibility
means the witness has to withstand tough cross-examination.

Thus, cross-examination of a state witness the lawyer knows is


telling the truth is permitted in a criminal case.

By the same token, however, the defense counsel could not put
on an affirmative defense in the criminal case that the defense lawyer knows
is frivolousbecause the lawyer is subject to the frivolousness requirement of
Rule 3.1except when defending the states case.
Thus, in strict answer to Question (b), yes you can cross-examine the
victim in an attempt to make her look bad in the eyes of the jury.
How do lawyers justify this?

Look at Note 1 following the Babcock essay; the lawyer in that


Note turns the tables on tradition.

That lawyer attempts to dehumanize the victim and the


prosecutions witnesses.

That way, the lawyer really likes the client.

And Note 6is withdrawal possible?

This cross-examination may be offensive.

But wont you have to get court approval? And wont a court
almost certainly deny the public defenders attempt to withdraw in light of
the courts obligation to provide counsel under the 6th Amendment.
Which then brings up Note 2can the public defender refuse cases?

If, under the 6th Amendment, every criminal defendant MUST


have effective representation, the only time a public defender can really
refuse is if taking the case will result in ineffective assistance, usually
because the defenders ability to make adequately considered decisions has
been compromised by the nature of the crime.

The same is likely true for appointed counselparticularly


where counsel is appointed because the public defender is either
disqualified or has been allowed to withdraw.
Look at Question (c)How do people do this kind of work? This is the Babcock

essay.

Babcock breaks the issue into multiple questions.


How can you do it when you know that when successful, your
now former client is free to commit more heinous crimes?

How can you defend when you know the client is guilty?

How can you move to suppress or cross-examine evidence you


know is truthful?

Then Babcock gives a litany of answers often used by criminal defense


attorneys.

The Garbage Collectorsomebody has to do it and it is required


under the 6th Amendment.

If we dont protect others, all our rights are eroded.

The Legalist or Positivist

I cant know if the client is guiltya jury actually decides that.

I cant know if the witness is telling the truththe jury has to


decide after hearing all the facts.

The Political Activist.

People who commit crimes are mostly victims of crimes.

Defending criminals then is merely putting all of society to the


test of its weaknesses.

The Social Worker.

I like to help people.

They will be helped by knowing they have a friendregardless


of the outcome.

The Egoist.

The work is interesting.

And better than doing routine business planning.

Do any of these appeal to you? What about others?


Now, go back to Question (a), a potpourri of difficult conduct for a criminal
defense attorney.

Can you get the clients Mom to get into a recorded conversation with
the victim in which the victim admits confusion? Because the clients Mom used to
care for the victim, the victim trusts the Mom and may readily consent.

Here look at Note 3 following Babcock.

Tape recording is not illegal2001 ABA opinion does not


prohibit secret recordings.

We already know that Rule 4.2 is not applicable so that sending


the clients Mom would not be the lawyer engaging in conduct that the
lawyer cannot do.

Can you avoid telling the client about the psychological report
so you can avoid the issue.

Notes 3 and 4 make a pretty compelling case that information


about the clients case is information the client is entitled to receive.

Particularly under Rule 1.3 where the client specifically asks for
information.

The best you can do is Note 5 which indicates that under Rule
1.2 and its Comments, tactical decision-making is left to the lawyer.

But, if you make a tactical decision that sandbags the clients


case, particularly in a criminal case, isnt that going to be ineffective
assistance.

Could it be argued that you have a personal conflict of interest


you are trying to preserve your own sanity while still defending and that
this conflict produced this resultand if that is true, the Strickland v.
Washington ineffective assistance test may be met.

If the victim really is confused, isnt this important?

The problem does not mention it, but finally look at Note 8 following
Babcocklawyer cannot ask jury to nullify the law, but can make indirect arguments
that suggest jury nullification is permissible.
Part C; the Lawyer as a Participant in Alternative Dispute Resolution.
A
First, does a lawyer have a duty to advise the client about possible non-litigation alternatives?
1
Rule 1.1 requires the lawyer to exercise the same competence as similarly situated lawyers
1
As the introductory material notes, the overwhelming majority of cases today (almost 98%
according to Yeazell) are settled without trial.
1
Additionally, many federal and state courts are using court-annexed mediationthereby forcing
litigants into at least one round of alternative dispute resolution before trial.
1
As a result, it seems likely that a reasonable lawyer would advise the client of alternatives to
litigation.
1
If that is true, then Rule 1.4(a)(2) and 1.4(b), which both require that the lawyer reasonably
explain the matter to the client so the client can make adequate decisions and confer with the
client on tactical decisions, probably mean that.
1
Under Rule 1.1 a lawyer DOES have an obligation to advise the client of these alternatives.
1
This analysis is probably the thinking of Rule 2.1's Comment [5] which indicates that it may be
necessary to advise the client of alternatives to litigation.
A
Now look at the Problemthe client lost an eye when a stick was thrown out of a lawn trimmer; the
client does not remember if warning tag was on the trimmerbut acknowledges that it may have been
and seems to recall reading the warning message about wearing goggles; You recommend trying for a
settlement of about $100,000, even though loss of an eye would normally bring closer to $250,000.
1
Question (a), in negotiating with the attorney for the manufacturer, can you say.
a
It will cost $500,000 to settle.
a
This is a blind in one eye case that is worth $500,000.
a
My client will testify there was no warning tag.
a
The warning tag was not on the product.
a
The warning tags were notoriously missing from these models, so it is logical to conclude
that there was none here?
a
Look at Note 1 following the Alfini article.

The Rules do not make specific reference to ethics in alternative dispute


resolution settings.

Rather, the Rules generally refer to litigation matters and everything else;

Thus, look at Rule 8.4(c)that Rule generally prohibits dishonesty, fraud, deceit
and misrepresentation; the Comments to this Rule do not discuss the obligation.

Rule 3.3 applies only to tribunals, but Rule 1.0(m) makes it clear that tribunal
would include binding arbitrationit would not therefore include negotiationor any
type of arbitration that is not binding.
That generally leaves Rule 4.1.

Rule 4.1(b) requires the lawyer to disclose material facts if doing so is


needed to avoid assisting the client in a criminal or fraudulent actunless disclosure
is prohibited by Rule 1.6.

Thus, if the lawyer knows the client is committing a fraudbut


the lawyer is not being used to assistthe lawyer has no obligation under this
Rule.

Similarly, if the only way the lawyer knows the true facts is as a
result of information relating to the representation, the lawyer may not
reveal this information.

Thus, even if the lawyer was being used to assist the client in
criminal or fraudulent activity, the lawyer would be prohibited from
revealing if the knowledge of the true facts came from information relating
to the representation.

As a result, Rule 4.1(b) is almost useless.

Thats why we focus on Rule 4.1(a), which prohibits the lawyer from
making a false statement of material fact or law to a third person.

Look at Comment [2]; As Alfini notes in his article, material


statements of fact DO NOT include estimates of price or value of the subject
of a transaction or the intentions of a party as to an acceptable settlement or
the fact that the party represents an undisclosed principal, unless failure to
reveal the undisclosed principal would amount to fraud.

Look also at Note 2 following the Alfini article on puffery in


negotiation.

Comment [2] virtually encourages puffery when it comes to


expectations in the settlement process.

As Alfini indicates, this Comment was not what was originally


envisioned in the Rules, but those in favor of puffing won the day.

Before answering this question, lets look at the Alfini article.

This article criticizes the current state of attorney conduct in


mediation.

It specifically mentions the notion of some that you cant trust


anything a lawyer says in mediation.

The article then points out the reason, by looking at the history
of the Rules of Professional Conduct.

As originally proposed, there was a rule that was far more


specific.

Rule 4.2 of the Discussion Draft of the Model Rules of


Professional Conduct (ABA Comm. on Eval. of Professional Standards,
1/30/1980), provided, in part, that the lawyer in negotiation had to be fair
in dealing with other participants; The Rule then provided that the lawyer
could not make a knowing misrepresentation of fact or law, or fail to
disclose a material fact known to the lawyer; As drafted, then, there were
two different requirements in dealing with factsNO fact could be
misrepresented and MATERIAL facts had to be disclosed.

This is the Rule to which Alfini refers and his article then
suggests that the comment to Rule 4.1 needs to be eliminated in favor of the
more specific requirements of current Rule 3.3.

What is interesting is that the same comments were part of the


original discussion draft, which first indicated that a party is not required
to apprise another party of background facts and then indicated no need to
disclose estimates of price of value that a party places on the subject of a
transaction, or a partys intentions as to an acceptable settlement of a claim;
Further, as in the current comment, there was no need to disclose an
undisclosed principal, unless nondisclosure amounted to fraud; See Rule 4.2
Comment (Disclosure) of the Discussion Draft of the Model Rules of
Professional Conduct (ABA Comm. on Eval. of Professional Standards,
1/30/1980)

Now, lets consider statements in Problem, Question (a) in light of Rule 4.1 and
the Alfini article.

It will cost $500,000 to settlethis is not a material statement of fact


under Rule 4.1 Comment [2]

This is a blind in one eye case that is worth $500,000neither is this


probably.

My client will testify there was no warning tagthe lawyer cannot offer
false testimony under Rule 3.3 and, because of discussions with the client, the
lawyer likely knows (or the circumstances certainly suggest) that this statement is
falseas a result, this statement would likely violate Rule 4.1

The warning tag was not on the product; this is a false statement of
material factthe client either thinks the tag was there or is not sure. It is prohibited
by Rule 4.1(a).

The warning tags were notoriously missing from these models, so it is


logical to conclude that there was none here? This is the kind of lying in
negotiation that the Alfini article addressesthere is not a definitive rule banning
this language and it is probably permitted.
Look at Problem, Question (b); the matter goes to mediation; are you allowed to make the same
statements in mediation?

Note 4 following the Alfini article answers this question for you.

Mediators are not part of a tribunal under the Rule 1.0(m) definition.

As a result, Rule 3.3 is not applicable to mediation.

Rule 4.1 is, however, applicable, so the answers to this question are the same as the
answers to question (a).
Problem, Question (c)the court practice is to use the United States Magistrate Judge as a
mediatorcan you make the statements?

Note 5 following the Alfini article.

Court-ordered or court-annexed arbitration and mediation is pretty commonin our


problem, there is a mediator.

As was clear in Note 4, mediation is not binding on the parties.

Therefore, mediation is not a proceeding before a tribunal, within the definition of Rule
1.0(m).

Therefore, Rule 3.3 is not likely applicable.

Here, look at Rule 2.4, Comment [5].

That comment makes pretty clear that while lawyers in alternative dispute
resolution are bound by the Rules of Professional Conduct.

Those lawyers are bound by Rule 3.3 only when appearing before a tribunal as
Rule 1.0(m0 envisions.

Otherwise, as the comment states, these lawyers are governed by Rule 4.1.

Thus, the technically correct answer to this question is the same as was the case for
Questions (a) and (b).

There is obviously the concern, however, about lying to a federal judgecould there be
contempt sanctions even if Rule 3.3 is not applicable?
1
Problem, Question (d)the parties agree on binding arbitration; can you make the statements?

As Note 3 following the Alfini article and our previous discussion demonstrate, once the
proceeding is binding, then, under Rule 1.0(m), the proceeding involves a tribunal and
Rule 3.3 is applicable.

Consider Comment [3] to Rule 3.3.

An assertion by a lawyer, purporting to state facts can only be made either with
knowledge, or based on reasonable inquiry.

Consider the statements, then, in light of that Rule and Comments.

This is a blind in one eye case that is worth $500,000; this statement is true
generally, but may not be true in this case depending on how the evidence comes in; as
part of an opening, it is probably still o.k., even under the more stringent standard of Rule
3.3.

My client will testify there was no warning tag; the lawyer cannot offer false
testimony under Rule 3.3(a)(3)this statement certainly cannot be made.

The warning tag was not on the product; the lawyer does not have any
independent basis for making this assertion; it is likely prohibited under Rule 3.3's
comment.

The warning tags were notoriously missing from these models, so it is logical to
conclude that there was none here? Part of this statement is truethe tags were often
missingIn this case, however, is the conclusion logical? Maybe not in light of what the
lawyer knows from the clientthis statement probably runs afoul of Rule 3.3(a)(1) and the
comments.
3
Before we leave the lawyer in Alternative Dispute Resolution, consider the Notes on Mediator and
Arbitrator Professional Responsibility.
1
Note 1there is no longer a rule on the Lawyer as Intermediary; Those states that have not
adopted the 2002 Rules, however, still have the prior Rule 2.2 and this Note describes how
difficult it was to comply with that rule, together with the fact that if compliance was not possible
after the lawyer started the mediation between clients, the lawyer could no longer represent
either of the parties.
2
Notes 2 and 3 mention new Rule 2.4Lawyer as neutral.

This rule recognizes the lawyers role as arbitrator or mediator when the parties are not
clients.

But then makes clear that the lawyer so serving is required to make sure the parties
understand the lawyer is not representing either of them and that the lawyer has an
obligation to the parties to assure that the parties understand the lawyers role.
1
Notes 4 and 5 then presuppose that the lawyer did serve as a third party neutral and now wants
to either represent one of those parties or be part of a firm that represents one of those parties.

Here, Rule 1.12 is applicable.

Rule 1.12 applies not only to judges, but to all other third party neutrals.

If the neutral participated personally and substantially in the matter between the parties;
the lawyer cannot represent any of the parties in that matter unless all parties provided
written, informed consent.

Other lawyers in the firm, however, are not disqualified provided there is screening of the
former party neutral and the parties are provided written notice.
1
Note 6 discusses other professional codes, most of which are voluntary, that apply to mediators,
or arbitrators, or other third party neutrals; they can be read by anyone who is interested.
Section D.Lawyer as Judge.
A
The Lawyer as Candidate for a Judgeship
1
First, look at Note 1 of this section and Rule 8.2(b).

Under Rule 8.2(b), the lawyer who is a candidate for a judgeship is required to comply
with the Code of Judicial Conduct applicable to judges political activity.
a
Thus, lawyers who are not yet judges, but are candidates, are required to comply with
Canon 5 of the Code of Judicial Conduct.
So, what are the rules regarding candidates for judge?
a
Under Canon 5(A), judges and candidates generally prohibited from

Acting as party leader or officer.

Making speeches or open endorsements of candidates for elected office.

Attending political gatherings.

Solicitation or contribution of money.

If the judge is already a judge, they have to resign as soon as they become a
candidate for a non-judicial elected office.

In addition, candidates for judge

Are required to maintain dignity in the campaign and encourage family


members to do the same.

Must prohibit others who work for the candidate from doing what the
candidate cannot do.

In campaigning, a judicial candidate.

Cannot make pledges other than for impartial performance of judicial


duties.

Cannot make statements that appear to commit the candidate to certain


positions in cases or controversies that might come before the judge.

Cannot knowingly misrepresent facts about the qualifications or


positions of the candidate or opposing candidate.

Knowingly in the Code of Judicial Conduct is the same as in


the Rules of Professional Conduct.

It means actual knowledge, which can be inferred from the


circumstances.

Can respond to personal attacks, but cannot do so in a way that violates


any of the other candidate rules.
a
In addition to these rules, there are separate rules for.

Candidates seeking APPOINTMENT to judicial officeCanon 5(B).

Candidates in Public Electionsand public elections includes not only regular


elections, but also retention electionsCanon 5(C).

Candidates in elections MAY purchase tickets and attend political


gatherings.

Candidates in elections MAY indicate they are members of the party.

Candidates in elections MAY contribute to parties.

Candidates in elections MAY distribute election brochures for


themselves and may appear in ads and at gatherings to campaign.

Candidates in elections MAY have their names listed with other


candidates of the same party for other political offices.

Candidates in elections MAY endorse other candidatesBUT ONLY for


the same judicial office.

Candidates in elections have significant limitations on their ability to


raise funds.

The candidate MAY NOT personally solicit funds.

The candidate MAY create a committee to solicit funds.

Such a committee MAY NOT, however solicit funds sooner


than one year before the election nor later than 90 days after the election.

No funds received can be used for the candidates personal


use.
With these rules in mind, look at our Problem,Friends on vacation, all male friends, one of
whom is a lawyer. The others ask the lawyer what he would do in a certain highly public case if
he were a judge. The lawyer says that if he were a judge, the litigant would receive a maximum
sentence, as would almost every criminal who appeared in front of this lawyer/judge.

Question (a) assumes that the lawyer had announced candidacy for judge the week before
this trip.

Note 2 following the problem.

If the judicial candidate simply announces his position on contested


political issues, the Supreme Court has indicated that this is protected speech
under the First Amendment.

How would you phrase what Tom Calvin said to make it a


protected announcement?

I believe judges should be tough on crime and I pledge to you


that if elected, I will generally sentence criminal defendants to the maximum
sentence allowed by law;

If, however, the judicial candidate commits or appears to commit to a


position in a certain case, that is prohibited and its First Amendment implications
are outweighed by the need for judicial impartiality.

By saying SHE would get the max, Tom here appears to be


committing to a particular result in a particular case.

And this is a commitment violation.

Note 3 following the problem.

Were there knowing misstatements about Toms opponent?

It is not known what is the position of the opponent.

Therefore, we cannot know if this is a misstatement.

What about the Nurture Nature roast?

The judicial candidate is required to maintain dignity.

The entire tenor of this dinner appears to run afoul of that


standard.

Certainly, off color comments about women run afoul of that.

Note 5 following the problem deals with the solicitation of campaign contributions.

If Tom had already announced his candidacy, and IF it was less than a
year before the election, then the formation of the committee by his friends was not
a problem.

If, however, it was more than a year before the election, there is a
violation.

The problem here is that Tom appears to know who is writing the
checks.

As the Note indicates, Comment [1] to Canon 5(C)(2) raises the


possibility of disqualification when a judge knows that lawyers who come before
the judge are making contributions; this may seriously affect judicial impartiality.

What about the gift of the fishing equipment.

This looks like a violation of Canon 5(C)(2).

Political contributions are being used to benefit the judicial


candidate personally.

Now look quickly at Question (d)suppose Toms friends are all lawyershow does that
change any answers?

This is partially answered by Note 6 following the problem; as lawyers, they are
subject to the provisions of Rule 8.2, which prohibits knowing or recklessly false
statements about the qualifications of a judge or judicial candidate.

In this problem, while the lawyers wrote letters, because we do not know how
weak the opponent would be on crime, we cannot determine whether there is a Rule 8.2
violation.

Now look at Question (c)Suppose Tom were already a judge and was running for
reelection.

First, all of the considerations that were part of Question (a) are applicable.

More than that, because we have a sitting judge, there are also considerations
applicable under Canon 2.

Under Canon 2(C), the judge may not be a member of any social
organization that discriminates on the basis of race, sex, religion or national origin.

Comment [1] to Canon 2(C) tells you that such membership gives
credence to the lack of impartiality.

Consider in light of this problem.

Here, the judge belongs to an all-mens social organization


even if it is just a bunch of fishing buddies.

That then combines with the judge telling sexually


inappropriate jokes at a political gathering.

Certainly leading one to conclude that this judge has either a


bias against women or is unwilling to treat women with the degree of
equality that we expect of any judge.

Also look at Canon 3(B)(5) which requires the judge to conduct all
judicial duties without bias.

It is unlikely that appearing at the Nurture Nature Roast is a


judicial duty;

But appearance at other events could trigger this rule.

Now look at Question (b)suppose Tom was seeking an appointed judgeship?

Note 7 following the Problem.

These candidates cannot raise funds at all.

They are subject to the no commitment rules.

They may not engage in most political activity other than


communicating with those who make the political appointments.

Thus, in our problem, virtually all of the activity of Tom would violate the Code
of Judicial Conduct if he were a candidate for an appointed position.
Lawyers Judicial Conduct.
1
In re Schapiro is a pretty typical judicial discipline case in that contains a tremendous amount of
impermissible activity; Most judicial discipline cases contain more than one violation of the Code
of Judicial Conduct.
2
Facts:

The judge demeaned lawyers in the courtroom by telling the lawyer he had to treat him
like a child, by yelling at lawyers, by refusing reasonable requests for continuances,
including a request by a pregnant prosecutor who had to check herself out of a hospital to
avoid a contempt charge; by berating and embarrassing attorneys, by swearing at
attorneys, by berating guests in the court, like the mother of a deceased child who wished
to speak at the bail hearing of the motorcyclist who allegedly ran down the womans child,
by rudely and needlessly interjecting himself into counsels questioning and arguments,
and by using a sound like a flushing toilet to show disapproval of attorney arguments, all
in violation of Canon 1, Canon 2A, and Canon 3(B)(4).

Also in violation of these Canons and Canon 3(B)(5), took a female lawyer into a back
room and told her she needed to act more like male attorneys and not become so
emotional.
You can see the courts concern in disciplining this judge.

The overriding concern of virtually all of the Code of Judicial Conduct is the integrity of
the judicial system.

When the judge undermines public confidence in the judiciary, the judge strikes at the
very root of an effective judiciary;
But note what the court did to the judge.

Public reprimand.

Treatment program.

Write apology letters.

Here, recall the two lawyer cases in this Chapter, Giangrasso and In re Vincenti; Both
those lawyers engaged in a similar pattern of disrespect for the judicial process and were,
in Giangrasso fined $100,000 and enjoined from filing lawsuits against the school district
and, in Vincenti, disbarred; Why does the judge here get a reprimand and some public
service only?

Certainly, we have a right to expect scrupulous conduct from our judges.

When they fail, shouldnt we punish them.

If we dont punish the judge who acts like this judge, wont respect for judges and
for our system decrease?

Yet, at the end of the day, we do not have a good answer to this question.
The Notes following the case describe some of the other areas of concern in dealing with judicial
conduct.

Note 1 indicates that the judge is required to act in a way that promotes respect for the
judiciary and for judicial impartiality.

Like a lawyer, a judge is a judge, both on and off the bench

Thus, can commit acts which bring disrespect on the judiciary generally; Acts like.

Sexual harassment

Use of judicial position to advance self--as by using judicial stationary


for personal business.

Receipt of gifts, directly or indirectly from persons either parties or


likely to become parties before the judge

For example, Bank makes favorable loans

For example, Law Firm lets judge use home at the lake

Violation of law generally

Criticism of other judges--makes entire institution of judging look bad

The same rule holds true for family members residing in the judge's household.

Note 2 again points out the requirement that a judge avoid prejudice and bias, such as
treating women with disrespect, as was the case in Schapiro

This includes membership, or public approval of an organization discriminates


on the basis of race, sex, religion or national origin.

This includes using such an organization.

As in does not belong to the organization but lunches in such a club frequently

Note 3 deals with Public Comments.

This includes making sure that others in the judges employment do the same.

Note that the judge cannot comment on a case,

But can advise of the procedures that will take place in a case.

Judge cannot compliment or criticize a jury for their verdict, but can thank jury
for their service.

Note 4 deals with courtroom conduct.

Conduct which is inappropriate to the courtroom again suggests either that the
judge is not impartial or which brings disrespect for the court or judges generally.

Profanity and other injudicious language

Disparaging Remarks about one firm of attorneys or one class of


attorneys, such as prosecutors generally

Intemperate judicial criticism


Other cases involve denial of rights--constitutional, federal, state, local.
Abuse of Power

Like contempt

Forcing litigants to do certain things either to get a hearing or to get a


particular result

Using judicial power to get persons hired, fired or otherwise taken care
of.
Showing Favoritism; for example, special rules for certain lawyers, firms or
against pro se litigant
Note 5 indicates that a judge is under a requirement to decide matters promptly.
Note 6 talks about limitations on other activities by the judge.
Judge can be involved in business but, cannot be an officer, director, manager,
general partner, advisor or employee of that business unless it is owned by the judges
family or manages investments for the judge.
Any business interest must not reflect adversely on judicial impartiality, nor
involve frequent transactions with lawyers or litigants likely to come before the judge.
It is permissible for judge to speak, teach, or write on vocational topics and to
earn money therefrom.
A judge can be an officer of the bar association
A full time judge cannot practice law
In all cases where judge earns income outside of judging, the judge is required to
file with the clerk a report, once a year, detailing that income.
A judge is severely limited as to their interaction with executive or legislative
bodies as this Note indicates.
Judge can be an officer and can generally work on behalf of a fraternal
organization; BUT, be careful of those organizations that discriminate against members by
race, sex or national origin.
Note 7 reviews the judges duty to report lawyers and allows the judge to take
appropriate action when the conduct is wrong but does not rise to the level of truly
reportable misconduct.
Finally, look at Note 10.
Why does it take so long to discipline the lawyers in Giangrasso and Vincenti and
the judge in Schapiro?
The Note suggests several possibilities.

Trial activity is inherent competitive and, as a result, we may allow both


lawyers and judges to get a little more worked up before imposing discipline.

We want to respect both the strategy decisions by lawyers and the


judicial decisions by the courts.

It is simply difficult to turn in both trial lawyers and judges.


But these three cases all demonstrate the same thing.

At the end of the day, what our practice is about is respect.

When respect is lost by lawyers representing clients.

When respect is lost by judges in dealing with lawyers and litigants.

Our entire system breaks down.

And that, we will not tolerate.

S-ar putea să vă placă și